You are on page 1of 177

Revalida Midyear (BASICS SET A,B, C)

December 6 2020

Color text RED and BOLD for answer

POOL PARTY:
PART 1

1. Where is the lesion located for a patient with The superior temporal gyrus contains the auditory cortex, which is
inability to determine sounds heard in the absence responsible for processing sounds. Specific sound frequencies map precisely
of ear abnormalities? onto the auditory cortex. This auditory (or tonotopic) map is similar to the
a. Temporal lobe homunculus map of the primary motor cortex
b. Superior temporal gyrus
c. Occipital lobe
d. Superior parietal lobe

2. Where are the nicotinic receptors in the myoneural The sarcolemma at the junction has invaginations called postjunctional folds,
junction located? which increase its surface area facing the synaptic cleft. These postjunctional
a. Terminal ends of the motor neuron folds form the motor endplate, which is studded with nicotinic acetylcholine
b. T-tubule receptors (nAChRs) at a density of 10,000 receptors/micrometer2.
c. Epimysium
d. Motor end-plate

3. Student X has normal defecation which is mostly due to the


activity of the:
a. Pelvic Splanchnic Nerves
b. Sympathetic Nerves
c. Vagus Nerves
d. Prevertebral ganglia

4. Seeing Korean food on your Grab app made your mouth


watery and is due to the activation of which preganglionic
Inferior salivatory nucleus
neuron?
a. Otic ganglion The inferior salivatory nucleus (or nucleus salivatorius inferior) is a cluster
b. Inferior salivatory nucleus of neurons in the pontine tegmentum (dorsal part of the pons), just above
c. Submandibular ganglion its junction with the medulla. It is the general visceral efferent (GVE)

“Courage is found in unlikely places.”


― J.R.R. Tolkien 1
d. Superior salivatory nucleus component of the glossopharyngeal nerve supplying the parasympathetic
input to the parotid gland for salivation.
It lies immediately caudal to the superior salivatory nucleus and just above
the upper end of the dorsal nucleus of the vagus nerve in the medulla.
The preganglionic parasympathetic fibres originate in the inferior salivatory
nucleus of the glossopharyngeal nerve. They leave the glossopharngeal
nerve by its tympanic branch and then pass via the tympanic plexus and the
lesser petrosal nerve to the otic ganglion. Here, the fibres synapse, and the
postganglionic fibers pass by communicating branches to the
auriculotemporal nerve, which conveys them to the parotid gland. They
produce vasodilators and secretomotor effects.

Function

Parasympathetic input from fibers of the inferior salivatory nucleus


stimulates the parotid gland to produce vasodilation and secrete saliva.

Superior salivatory nucleus

The superior salivatory nucleus (or nucleus salivatorius superior) of the facial
nerve is a visceromotor cranial nerve nucleus located in the pontine
tegmentum. It is one of the salivatory nuclei.
Parasympathetic efferent fibers of the facial nerve (preganglionic fibers)
arise according to some authors from the small cells of the facial nucleus, or
according to others from a special nucleus of cells scattered in the reticular
formation, dorso-medial to the facial nucleus – the superior salivatory
nucleus.
Some of the preganglionic fibers travel along the greater petrosal nerve
through the pterygoid canal, where they join the postsynaptic fibers of the
deep petrosal nerve to become the nerve of the pterygoid canal. These fibers
synapse in the pterygopalatine ganglion, whereupon the postganglionic,
postsynaptic, efferent fibers travel to innervate the lacrimal gland and the
mucosal glands of the nose, palate, and pharynx.
Preganglionic parasympathetic fibers are also distributed partly via the
chorda tympani and lingual nerves to the submandibular ganglion, thence by
postganglionic (vasodilator) fibers to the submandibular and sublingual
salivary glands.
The term "lacrimal nucleus" is sometimes used to refer to a portion of the
superior salivatory nucleus

“Courage is found in unlikely places.”


― J.R.R. Tolkien 2
5. While explaining anemia to a 32 year old patient, you
explain that this bone is no longer a site for hematopoiesis:

a. Ribs
b. Sternum
c. Vertebra
d. Femur

Parang lahat naman sila pwede???????

“Courage is found in unlikely places.”


― J.R.R. Tolkien 3
6. As part of the neurologic exam, the cranial nerves were
tested and you elicited the corneal reflex which helps you
access which region of the CNS?
a. Mesencephalon
b. Myelencephalon
c. Metencephalon
d. Diencephalon

7. A newly wed male argues that the newborn baby with blood
type O is not his. The father is a type B homozygous and the
mother is a type AB, what is the probability that this newborn
will have that blood type?
a. 0%
b. 25%
c. 50%
d. 75%

8. Why would tetany not happen in cardiac muscles? The refractory period of cardiac muscle is dramatically longer than that of
a. They have a more calcium ions skeletal muscle. This prevents tetanus from occurring and ensures that each
b. They have a long absolute refractory contraction is followed by enough time to allow the heart chamber to refill
period with blood before the next contraction.
c. They have more mitochondria than
skeletal muscles
d. They have a long hyperpolarized period
e.

“Courage is found in unlikely places.”


― J.R.R. Tolkien 4
9. For a left-handed person, an ischemic event to this area
presents a problem in the regulation of speaking: left inferior frontal gyrus houses the broca’s area
a. Left superior temporal gyrus broca’s area is responsible in expressing spoken and written language
b. Left inferior frontal gyrus
c. Right superior temporal gyrus
d. Right inferior frontal gyrus

10. A patient complaining of frequent bruising followed up


with a bleeding time result that is prolonged, this may be due
to:

a. Vit. K deficiency
b. Thrombocytopenia
c. Hemophilia
d. Protein C deficiency

11. After eating a heavy breakfast, the distention of the The gastrocolic reflex is a physiological reflex that controls the motility of
stomach will: the lower gastrointestinal tract following a meal. As a result of the
a. Trigger gastrocolic reflex gastrocolic reflex, the colon has increased motility in response to the stretch
b. Inhibit gastric emptying of the stomach with the ingestion of food.
c. Decrease salivary secretion
d. Cause glucose absorption

12. While playing basketball, a blunt injury to the neck


occurred after which a tingling sensation in the clavicular area
was felt. The most probable level of injury is at:

a. C1
b. C3
c. C5
d. C7

“Courage is found in unlikely places.”


― J.R.R. Tolkien 5
13. A patient with a failing heart was prescribed cardiac
glycosides in order to:

1. Increase the excretion of fluids


2. Increase myocardial contractile force
3. Increase the storage of fluids in the
venous system
4. Decrease the absorption of sodium

14. Checking a patient’s arterial blood gas, you take note that
this factor increases the affinity of hemoglobin to oxygen:
a. Increase in pH (or decreased [H+])
b. Increase in pCO2
c. Increase in 2,3 DPG
d. Increase in temperature

“Courage is found in unlikely places.”


― J.R.R. Tolkien 6
15. A patient complains of palpitations. While examining him,
you place the diaphragm of the stethoscope at which best
location if you want to hear the sound produced by closing off
the mitral valve?

a. 4th ICS, LMCL


b. 4th ICS, LPSM
c. 5th ICS, LPSM
d. 5th ICS, LMCL

16. Lymph from the skin of the lateral side of the foot drains
into ____?
a. Internal iliac node
b. Inguinal node
c. Subsartorial nodes
d. Popliteal node

17. Most fractured carpal bone to suspect if one will fall from A scaphoid (navicular) fracture is a break in one of the small bones of the
an outstretched hand? wrist. This type of fracture occurs most often after a fall onto an
a. Pisiform outstretched hand. Symptoms of a scaphoid fracture typically include pain
b. Capitate and tenderness in the area just below the base of the thumb.
c. Hamate
d. Scaphoid

“Courage is found in unlikely places.”


― J.R.R. Tolkien 7
18. The penile urethra is embedded in what specific structure?
a. Corpus spongiosum
b. Corpus cavernosum
c. Corpus striatum
d. Corpus spinosum

The pendulous urethra is invested in the corpus spongiosum of the penis in


the pendulous portion of the penis. The urethra is located concentrically
within the corpus spongiosum. In the distal urethra lies the fossa navicularis,
a small dilation of the urethra just proximal to the urethral meatus.

19. Which part of the intestines is movable? The transverse colon is the longest and most movable part of the colon. It
a. Rectum crosses the abdomen from the ascending colon at the hepatic or right colic
b. Sigmoid flexure with a downward convexity to the descending colon where it curves
c. Transverse colon sharply on itself beneath the lower end of the spleen forming the splenic or
d. Ascending colon left colic flexure.

20. Largest and strongest bone of the foot. Calcaneus – the largest bone of the foot, which lies beneath the talus to
a. Calcaneus form the heel bone.
b. Cuboid
c. Cuneiform
d. Talus

21. Children aged 12 months through age 12 years can be It has been postulated that these B cells either persist throughout the
given 2 doses of varicella vaccine subcutaneously, separated
lifetime of the host84 or are renewed constantly through either
by at least 3 months and results in faster and higher antibody
production brought by which attribute of B cells? nonspecific85 or antigen-specific stimulation.
a. Memory
b. Clonal proliferation Memory B cells circulate throughout the body in a quiescent state until
c. Antigenicity
specific antigen is re-encountered and triggers a potent secondary immune
d. Specificity
response. Memory cells respond to antigen much faster, require lower
amounts of antigen, and can even be induced in its absence by soluble
mediators such as IL-2 or IL-15, in part because the BCR is already localized
to lipid rafts. Subsequently, just like naïve B cells, memory B cells ingest
antigen and express peptides–MHC class II fragments. After antigen
presentation of peptides to helper T cells, memory B cells undergo expansion
and may differentiate to plasma cells.i

“Courage is found in unlikely places.”


― J.R.R. Tolkien 8
22. A patient with acute gastroenteritis with no signs of
dehydration was given an oral rehydration solution. Glucose
with sodium in the ORS when absorbed in the GIT exemplifies
what type of transport process?
a. Primary active transport
b. Passive transport
c. Secondary active transport
d. Vesicular transport

Glucose is absorbed by sodium glucose co-transport mechanism. SGLT1


In secondary active transport, also known as coupled transport or
cotransport, energy is used to transport molecules across a membrane;
however, in contrast to primary active transport, there is no direct coupling
of ATP.

23. Patient X obtained abrasions from falling off his bicycle


after avoiding a pedestrian. Which is NOT expected from the
resulting inflammatory process?
a. Migration of large number of granulocytes and
monocytes into the injured tissue
b. Vasoconstriction of local blood vessels decreasing local
blood flow
c. Clotting of fluid in the interstitial space due to leakage of
fibrinogen
d. Increased permeability of the capillaries with leakage of
fluid into the interstitium

24. A patient with bilious vomiting showed this compensatory Pancreatic and biliary drainage most often occurs after surgery, and usually
mechanism: the volume is low, so, despite loss of a HCO3−-rich fluid, significant metabolic
a. Increased secretion of H+ ion by the proximal tubules acidosis does not occur. In the rare setting in which drainage volume exceeds
b. Increased generation of hydrogen ions from food 1 to 2 L/d, however, metabolic acidosis will develop and be maintained by
c. Increased activity of beta intercalated cells of collecting concomitant volume depletion
ducts
d. Decreased rate and depth of breathing

“Courage is found in unlikely places.”


― J.R.R. Tolkien 9
25. Which of the following factors decrease the cardiac output
in normal physiologic states?
a. Venous Return
b. Afterload
c. Effective filling time
d. Effective filling pressure

Venous return is thus directly dependent on cardiac output. Central venous


pressure by contrast is essentially an independent variable and can regulate
the stroke volume. Venous return 'controls' cardiac output only in the sense
that transient inequalities between the two alter the CVP.

“Courage is found in unlikely places.”


― J.R.R. Tolkien 10
“Courage is found in unlikely places.”
― J.R.R. Tolkien 11
26. The presence of this manifestation in your patient made
you pinpoint the location of his brain tumor in the medial side
of the medulla oblongata:

a. Impaired gag reflex


b. Inability to close an eye
c. Deviation of the neck on one side
d. Paralysis of the tongue

Medial medullary syndrome is caused by a lesion in the medial part of the


medulla, which is due to an infraction of vertebral arteries and/or
paramedian branches of the anterior spinal artery. So, medial medullary
structures, including the lateral corticospinal tract, medial lemniscus, and
hypoglossal nerve, are commonly damaged in medial medullary syndrome.

The lateral corticospinal tract controls the voluntary movement of


contralateral limbs of the body. The medial lemniscus receives sensory
(vibration, fine touch, and proprioception) input from contralateral nucleus
cuneatus or nucleus gracilis and sends this signal to the sensory cortex of the
brain. Due to the involvement of caudal medulla, hypoglossal nerve (twelfth
cranial nerve) damage may occur. The hypoglossal nerve supplies the
intrinsic and extrinsic muscles of the tongue except for palatoglossus.

The function of each muscle supplied by the hypoglossal nerve are described
below:

● Genioglossus- Protrude the tongue forward from the root


● Hyoglossus- Retracts and depresses the tongue
● Styloglossus- Draws the tongue upward
● Intrinsic muscles (transversus, verticalis, superior/inferior
longitudinalis)- By curving, shortening, narrowing the tongue, they
change the shape of the tongue.

27. Hypokalemia was found in a hypertensive patient on A decrease in extracellular K+ concentration leads to hyperpolarization of
prolonged hydrochlorothiazide who had difficulty ambulating. the cell membrane and can result in muscle weakness occasionally severe
This finding can explain the muscle weakness due to: enough to cause paralysis, as occurs in patients with hypokalemic dRTA.

a. Hyperpolarization of the resting membrane potential Hyperpolarization is a change in a cell's membrane potential that makes it
requiring greater stimulus intensity to initiate an action more negative. It is the opposite of a depolarization. It inhibits action
potential potentials by increasing the stimulus required to move the membrane
b. Hyperpolarization of the resting membrane potential potential to the action potential threshold.
requiring lesser stimulus intensity to initiate an action
potential
c. Hypopolarization of the resting membrane potential
requiring greater stimulus intensity to initiate an action
potential

“Courage is found in unlikely places.”


― J.R.R. Tolkien 12
d. Hypopolarization of the resting membrane potential
requiring lesser stimulus intensity to initiate an action
potential

28. Which of the following will lead to an increase in stroke


volume?
a. An increase in aortic pressure
b. A decrease in venous return
c. A highly tachycardic heart
d. A more negative intrathoracic pressure

Reduces SV (googled):
1. Decrease in venous return
2. Tachycardia

Increased pulmonary venous return to the left atrium leads to increased


filling (preload) of the left ventricle, which in turn increases left ventricular
stroke volume by the Frank-Starling mechanism.

Tachycardia of atrial or ventricular origin reduces stroke volume and cardiac


output particularly when the ventricular rate is greater than 160 beats/min.

As the left ventricle ejects blood into the aorta, the aortic pressure increases.
The greater the stroke volume, the greater the change in aortic pressure
during ejection. (Yung SV nagprprpoduce ng aortic pressure).

29. Which of the following is NOT associated with the Endothelium, the inner most single layer of cells lining the blood vessels,
development of a thromboembolic condition? provides a surface for thrombosis formation and critically regulates blood
fluidity and homeostasis. As a barrier, endothelium separates blood clotting
a. Normal bleeding time and clotting time factors from exposure to subendothelial prothrombotic extracellular matrix

“Courage is found in unlikely places.”


― J.R.R. Tolkien 13
b. Platelet count of 150,000-300,000/ul components. Endothelium also secretes or expresses vasoactive factors that
c. Smooth endothelial surface modulate platelet reactivity, coagulation, fibrinolysis, and vascular
d. Adequate levels of calcium on the blood contractility, all of which contribute to thrombotic formation. Such factors
include nitric oxide, prostacyclin, Von Willebrand factor (VWF),
thrombomodulin, endothelin, etc. Accumulating evidence show that
endothelial cells (ECs) play a pivotal role in modulating thrombosis,
highlighting ECs as a potential target for thrombosis control.

30. After eating a hamburger, most of its chemical digestion Protein digestion occurs in the stomach and the duodenum through the
will occur where in the GI tract? action of three main enzymes: pepsin, secreted by the stomach, and trypsin
and chymotrypsin, secreted by the pancreas. During carbohydrate digestion
a. Esophagus the bonds between glucose molecules are broken by salivary and pancreatic
b. Stomach amylase.
c. Small intestines
d. Large intestines

31. Weakest area or part of the shoulder joint.


inferior capsular aspect, this is the point where the capsule is the weakest.

a. Anterior
b. Posterior
c. Superior
d. Inferior

32. Part of the brain that separates frontal from parietal lobe The frontal lobe is separated from the parietal lobe by a space called the
of cerebrum: central sulcus, and from the temporal lobe by the lateral sulcus. The frontal
a. Precentral gyrus lobe is generally where higher executive functions including emotional
b. Central sulcus of Rolando regulation, planning, reasoning and problem solving occur.
c. Lateral Sylvian fissure
d. Longitudinal fissure

33. As the facial nerve emerges from the stylomastoid The facial nerve exits skull via stylomastoid foramen; nearby, it gives off the
foramen, it immediately gives out a branch that goes behind posterior auricular nerve which is meant to supply the occipital belly of the
the ear to innervate the occipitalis and auricularis posterior occipitofrontalis muscle and some of the auricular muscles, and nerves to
muscles. What do you call that nerve branch? the posterior belly of the digastric and the stylohyoid.
a. Great auricular nerve
b. Posterior auricular nerve
c. Marginal mandibular nerve
d. Auriculotemporal nerve

“Courage is found in unlikely places.”


― J.R.R. Tolkien 14
34. What vertebra that is identified by the presence of the
mammillary process?
a. Atlas (ribs)
b. Cervical
c. Thoracic
d. Lumbar

35. The antero-posterior diameter of pelvic inlet that can be


measured through vaginal exam: “11cm=true conjugate”
sagittal conjugate = pelvic outlet?
a. Diagonal conjugate Diagonal conjugate = 12.5 cm
b. Obstetrics conjugate Obstetrics conjugate = 10.5 cm
c. Sagittal conjugate Transverse conjugate = 13 cm
d. Transverse conjugate

Diagonal conjugate is measured through vaginal exam.


Shortest diameter is obstetrics conjugate

“Courage is found in unlikely places.”


― J.R.R. Tolkien 15
36. Which of the following nerves is NOT a branch of CN VII:

a. Buccal
b. Mental
c. Mandibular
d. Temporal

37. The following are unpaired cranial bones, EXCEPT:

a. Frontal
b. Temporal
c. Sphenoid
d. Occipital

*FESO
Frontal
Ethmoid
Sphenoid
Occipital

38. Which of the following cranial nerves does NOT originate


from the pons?

a. Trochlear
b. Trigeminal
c. Abducens
d. Facial

“Courage is found in unlikely places.”


― J.R.R. Tolkien 16
39. This muscle flexes the metatarsophalangeal joint of the big FLEXOR HALLUCIS LONGUS
toe and also supports the medial longitudinal arch ORIGIN
Lower two thirds of posterior fibula between median crest and posterior
a. Flexor hallucis longus border, lower intermuscular septum and aponeurosis of flexor digitorum
b. Flexor hallucis brevis longus
c. Adductor hallucis Oblique head INSERTION
d. Lumbricals Base of distal phalanx of big toe and slips to medial two tendons of flexor
digitorum longus
ACTION
Flexes distal phalanx of big toe, flexes foot at ankle, supports medial
longitudinal arch
NERVE
Tibial nerve (S2, 3)

40. Which of the following is not part of the Levator ani?

a. Puborectalis
b. Coccygeus
c. Iliococcygeus
d. Pubococcygeus

41. A patient followed up for the reading of his tuberculin skin The reaction to intracutaneously injected tuberculin is the classic example of
test, which you interpret as a positive response. This is due to a delayed (cellular) hypersensitivity reaction.
which type of hypersensitivity reaction?

a. Immediate
b. Delayed
c. Antibody-mediated
d. Immune-complex

“Courage is found in unlikely places.”


― J.R.R. Tolkien 17
42. When testosterone produced by the Leydig cells of the The synthesis of both products is regulated by endocrine hormones
testes acts on adjacent seminiferous tubules to stimulate produced in the hypothalamus and pituitary, as well as locally within the
spermatogenesis, what type of chemical signaling is testis. Testosterone is indispensable for sperm production, however both
exemplified? testosterone and Follicle Stimulating Hormone (FSH) are needed for optimal
testicular development and maximal sperm production. Sperms are
a. Autocrine produced via the extraordinarily complex and dynamic process of
b. Endocrine spermatogenesis that requires cooperation between multiple testicular cell
c. Juxtacrine types. While it has long been known that testosterone and FSH regulate
d. Paracrine spermatogenesis, years of research has shed light on many of the intricate
mechanisms by which spermatogonial stem cells develop into highly
specialized, motile spermatozoa. Spermatogenesis involves the concerted
interactions of endocrine hormones, but also many paracrine and growth
factors

43. Under normal circumstances, which neurotransmitter Glutamine is an energy source for intestinal and immune cells. It also helps
provides inhibition of muscle tone that balances the excitation maintain the barrier between the intestines and the rest of your body and
of muscle tone provided by other neurotransmitters? aids with proper growth of intestinal cells.
Glutamate is a powerful excitatory neurotransmitter that is released by
a. Glycine nerve cells in the brain. It is responsible for sending signals between nerve
b. Glutamine cells, and under normal conditions it plays an important role in learning and
c. Glutamate memory
d. Glucagon Glucagon is used along with emergency medical treatment to treat very low
blood sugar.
-used in diagnostic testing of the stomach and other digestive organs.
-glycogenolytic agents.
-It works by causing the liver to release stored sugar to the blood

44. Which electrolyte can cause cardiac arrest in diastole when The entry of sodium and calcium play a key effect on myocytes subjected to
it is in excess? cardiac arrest by hyperkalemia. They cause cell swelling, acidosis,
consumption of adenosine triphosphate and trigger programmed cell death.
a. Calcium
b. Chloride
c. Potassium
d. Sodium

45. Motor fibers that form the corticospinal tract are mostly
coming from the:

a. Premotor area
b. Primary motor area
c. Primary somatosensory area
d. Supplementary motor area

“Courage is found in unlikely places.”


― J.R.R. Tolkien 18
46. Educating your patient why he is experiencing heartburn
after eating meals, you explained that this is because of the
inability of the lower esophageal sphincter to contract
effectively when food enters the stomach, also called as:

a. Dysphagia
b. Achalasia
c. Gastric reflux
d. Hirschsprung disease

47. This blocks the active sites on actin for myosin cross-bridge When a muscle is in a resting state, actin and myosin are separated. To keep
formation when calcium is not present: actin from binding to the active site on myosin, regulatory proteins block the
molecular binding sites. Tropomyosin blocks myosin binding sites on actin
a. Actinin and titin molecules, preventing cross-bridge formation and preventing contraction in
b. Troponin-tropomyosin complex a muscle without nervous input. Troponin binds to tropomyosin and helps to
c. Sarcolipin and myoregulin position it on the actin molecule; it also binds calcium ions.
d. Sarcoplasmic reticulum and the T-tubules

48. Your asthmatic patient complained that every time he uses


his reliever medication he experiences a feeling of incomplete
voiding. What kind of drug is his reliever medication?

a. Sympathomimetic
b. Parasympathomimetic
c. Parasympatholytic
d. Sympathomimetic and parasympatholytic

49. A patient with which condition will NOT exhibit an increase


in hematocrit?

a. Pregnancy
b. Dehydration
c. Polycythemia Vera
d. High altitude dwelling

“Courage is found in unlikely places.”


― J.R.R. Tolkien 19
50. Among water soluble hormones, which of the following is
NOT a 2nd messenger?

a. Cyclic AMP
b. Cyclic GMP
c. Calcium
d. Acetylcholine

PART 2

1. What is the major end- product of beta oxidation of fatty In biochemistry and metabolism, beta-oxidation is the catabolic process
acid? by which fatty acid molecules are broken down in the cytosol in
prokaryotes and in the mitochondria in eukaryotes to generate acetyl-
CoA, which enters the citric acid cycle, and NADH and FADH2, which are
o Priopionyl CoA co-enzymes used in the electron transport chain.
o Butyroyl CoA
o Acetoacetyl CoA
o Acetyl CoA

2. Which of the following is NOT a steroid hormone? Thyroid hormones are two hormones produced and released by the
thyroid gland, namely triiodothyronine (T3) and thyroxine (T4). They are
tyrosine-based hormones that are primarily responsible for regulation of
o Tetraiodothyronine (T4) metabolism. T3 and T4 are partially composed of iodine.
o Aldosterone
o Estriol
o Dihydrotestosterone

3. What is the primary biochemical lesion in homozygotes Familial hypercholesterolemia is caused by a reduction or defect in the
with familial hypercholesterolemia? LDL receptor, which is also called the apo B/E receptor because it binds
both apolipoprotein B and apolipoprotein E.

o Loss of feedback inhibition of HG reductase


o Loss of apolipoprotein B
o Increased production of LDL from VLDL
o Functional deficiency of plasma membrane
receptors for LDL

4. Which hormone is produced using two mitochondrial Mitochondria are essential sites for steroid hormone biosynthesis. ...
enzymes? Several other steroidogenic enzymes, including 3β-hydroxysteroid
dehydrogenase, 11β-hydroxylase and aldosterone synthase also reside
in mitochondria.
o Testosterone
o Estradiol
o Aldosterone

“Courage is found in unlikely places.”


― J.R.R. Tolkien 20
o Cortisol

5. What is the complication of excessive ketogenesis? ketoacidosis is a high anion gap metabolic acidosis due to an excessive
blood concentration of ketone bodies (keto-anions). ketone bodies
(acetoacetate, beta-hydroxybutyrate, acetone) are released into the
o Metabolic acidosis blood from the liver when hepatic lipid metabolism has changed to a
o Metabolic alkalosis state of increased ketogenesis
o Respiratory acidosis
o Respiratory alkalosis

6. What happens during each cycle of β-oxidation? Briefly, each cycle of this process begins with an acyl-CoA chain and ends
with one acetyl-CoA, one FADH2, one NADH and water, and the acyl-CoA
chain becomes two carbons shorter.
o One carbon atom is removed from carbonyl end of
the fatty acid
o One carbon atom is removed from the methyl end of
the fatty acid
o Two carbon atoms are removed from carbonyl end
of the fatty acid
o Two carbon atoms are removed from the methyl
end of the fatty acid

7. The “B” gene in ABO blood group codes for what enzyme? The ABO gene is located on chromosome 9 and has three alleles, A, B and
O. The A allele encodes a glycosyltransferase that adds N-
acetylgalactosamine to the glycoprotein H antigen that is expressed on
o G-6-PD all normal red cells. The B allele encodes a different glycosyltransferase
o Gal transferase that adds d-galactose
o Fucosyl transferase
o GalNAc transferase

8. Deficiency of this/these enzyme(s) can lead to Primary hyperammonemia is caused by several inborn errors of
hyperammonemia: metabolism that are characterised by reduced activity of any of the
enzymes in the urea cycle. The most common example is ornithine
transcarbamylase deficiency, which is inherited in an X-linked fashion.
o Carbamoyl phosphate synthetase 2
o Cystathionine synthetase
o Saccharopine dehydrogenase
o No correct answer among the choices

9. In fibrinolysis, plasminogen is activated to plasmin and Utilizing both clotting and chromogenic assays, the fibrinolysis protease
inhibits what factor? plasmin was found to irreversibly inhibit the pivotal function of factor X
(FX) in coagulation. This was due to cleavage at several sites, the location
of which were altered by association of FX with procoagulant
o Factor X phospholipid (proPL).
o Factor XI
o Factor XII

“Courage is found in unlikely places.”


― J.R.R. Tolkien 21
o Factor XIII

Factor 13 is fribrin. Plasmin inactivates factor 13

10. Which of the following post-translational modifications is Protein phosphorylation is the most common post-translational
involved in regulation of the activity of covalently modification. In eukaryotes, the most commonly phosphorylated amino
modulated enzymes? acid residues are serine, threonine and tyrosine.

o Phosphorylation of serine
o Carboxylation of glutamic acid
o Glycosylation of tyrosine
o Removal of leader peptide

11. The binding of epinephrine to which receptor inhibits The beta-adrenergic receptor is coupled to adenylyl cyclase via Gs and
adenylate cyclase from producing cAMP? binding of epinephrine to this receptor leads to increased cyclic AMP
o Vasopressin-V2 synthesis. Also, when epinephrine binds to alpha-2 adrenergic receptors,
o Vasopressin-V1 adenylyl cyclase activity is inhibited, because that receptor is coupled to
o α2 adrenergic via Gi, an inhibitory G protein.
o α1 adrenergic

12. Which of the following enzymes catalyzes the conversion Cholic acid is a primary bile acid.
of cholesterol to cholic acid? Cholesterol 7α-hydroxylase is the rate-limiting enzyme in the conversion
o 7-alpha hydroxylase of cholesterol to bile acids and is critical to regulation of bile acid pool
o 9-beta hydroxylase size.
o 17-alpha hydroxylase
o 19-beta hydroxylase

13. The product of the reversed glutamate dehydrogenase Glutamate dehydrogenase (GDH) is a hexameric enzyme that catalyzes
reaction: the reversible conversion of glutamate to α-ketoglutarate and ammonia
while reducing NAD(P)+ to NAD(P)H.
o α-ketoglutarate
o oxaloacetate
o ammonia
o glutamic acid

“Courage is found in unlikely places.”


― J.R.R. Tolkien 22
14. There could be premature of termination of protein Termination of translation occurs when a nonsense codon (UAA, UAG,
synthesis when this codon is encountered by the or UGA) is encountered.
ribosome:
o UAG
o UCC
o CUG
o GAU

15. Which of these steps in beta oxidation is rate-limiting? Carnitine palmitoyltransferase 1 (CPT 1), located on the outer
o Carnitine transport of the fatty acid mitochondrial membrane, transfers the acyl group from CoA to carnitine.
o Thiolytic cleavage of ketoacyl CoA This is the rate limiting step in beta-oxidation.
o Oxidation of fatty acyl CoA
o Activation of free fatty acid to fatty acyl CoA

16. What happens if the energy requirement of the cell is Not sure.
increased?

o Citrate synthase is stimulated


o G-6-P-D is stimulated
o Phosphofructokinase 2 is stimulated
o All choices

17. Action of the levator ani muscles when defecating


o Are completely inactive
o Relax portion of puborectalis with the anal
sphincters
o Do not support the sigmoid colon
o Do not support the uterus and vagina

18. The committed step in extramitochondrial lipogenesis Fatty acid synthesis starts with the carboxylation of acetyl CoA to
involves the formation of: malonyl CoA. This irreversible reaction is the committed step in fatty acid
synthesis.
Acetyl CoA carboxylase can be regarded as the first truly committed step
o Acetyl CoA leading specifically to fatty acid synthesis because the product, malonyl
o Propionyl CoA CoA, has no other known metabolic fate. The importance of this enzyme
o Succinyl CoA in the regulation of lipogenesis has been recognized for many years.
o Malonyl CoA

19. The artery that supplies the gallbladder usually arises from the The arterial supply to the gallbladder is via the cystic artery – a branch of
_____ the right hepatic artery (which itself is derived from the common hepatic
artery, one of the three major branches of the celiac trunk/celiac artery).
o Celiac artery
o Gastroduodenal artery
o Inferior mesenteric artery
o Superior mesenteric artery

“Courage is found in unlikely places.”


― J.R.R. Tolkien 23
20. These are the two starting materials needed in the pathway in Glycine and succinyl-CoA are the starting materials; succinyl-CoA is an
order for heme to be synthesized: intermediate of the Krebs cycle, and glycine can be derived from the diet
or from serine. The first step in heme synthesis is catalyzed by
o Succinyl-CoA and leucine aminolevulinic acid synthase, a vitamin B6-requiring enzyme.
o Succinyl-CoA and lysine
o Succinyl-CoA and glycine
o Succinyl-CoA and tyrosine

21. The uterus receives its blood supply from what artery? The ovarian artery supplies the oxygenated blood to the ovary, fallopian
tube, and uterus. In the myometrium, the uterine artery further
branches out to the arcuate artery, radial artery, spiral artery and basal
o Uterine artery artery.
o Uterine and ovarian arteries
o Middle rectal artery
o Superior vesical artery

22. All of the following would cause a shift to the right on the Shift os the oxygen dissociation curve to the right
oxygen dissociation curve, EXCEPT ● Increase in partial pressure of carbon dioxide (PC02)
● Decrease in pH
o Hypothermia ● Increased 2,3 DPG
o Hyperthermia
o Decreased pH
o Increased 2,3 DPG Hypothermia produced a left shift of the oxyhemoglobin dissociation
curve

23. The following are 1st layer muscles of the sole, EXCEPT: First layer muscles
● Abductor Hallucis M.
o Abductor hallucis muscle ● Flexor Digitorum Brevis
o Abductor digiti minimi muscle ● Abductor Digiti Minimi
o Flexor digitorum brevis muscle
o Quadratus plantae muscle 2nd Layer
- Quadratus Plantae

24. Anemia secondary to malaria is classified under? The most common extrinsic causes of hemolytic anemia includes:
● Hyperactive immune system (autoimmune hemolytic anemia)
E3 ● Infections
o Extrinsic hemolysis ● Certain medications
o Acute blood loss ● Abnormal blood vessels
o Intrinsic hemolysis ● Trapping in the spleen
o Disorder of erythropoiesis ● Some cancers, including leukemia and lymphoma

25. TRUE of the Subtalar joint: The subtalar joint (talocalcaneal joint) is anatomically described as the
articulation formed between posterior articular surfaces of two tarsal
o Consists of calcaneocuboid bones; the talus and calcaneus. The joints move in a triaxial plane, which
o Also known as “Chopart’s joint” allows for the motions of flexion/extension, inversion/eversion, and
o Synovial joint that can evert/invert foot adduction/abduction.

“Courage is found in unlikely places.”


― J.R.R. Tolkien 24
o Transverse tarsal joint

26. Which is true regarding degeneracy of the genetic code? Degeneracy of codons is the redundancy of the genetic code, exhibited
as the multiplicity of three-base pair codon combinations that specify an
o Involves the 1st base of the codon amino acid.
o Involves the 3rd base of the codon
o Involves the N-terminal end of the polypeptide
o Involves the C-terminal end of the polypeptide

27. Common action of Gastrocnemius, Soleus and Plantaris muscles Along with the soleus muscle, the gastrocnemius forms half of the calf
on the foot muscle. Its function is plantar flexing the foot at the ankle joint and
flexing the leg at the knee joint
o Dorsiflexion
o Inversion
o Eversion
o Plantarflexion

28. Which of the following regarding functions of immune cells Naive lymphocytes expressing the chemokine receptor CCR7
under natural immunity is CORRECT? continuously migrate into lymph nodes (LN) from the blood via
specialized high endothelial venules (HEV), while antigen-bearing
o Basophils produce reactive oxygen species to kill pathogens dendritic cells (DC), also expressing CCR7, carry antigen from the
it phagocytosed periphery into the LN via afferent lymphatics.
o Dendritic cells secrete chemokines that orchestrate the
migration of other immune cells within the lymph nodes
o Neutrophils during inflammation potentiates activation of
B and T cells by the expression of co-stimulatory molecules
o Natural Killer cells produce and release cationic content of
granules to induce inflammation

29. The levator ani muscle receives its innervation from the Female levator ani muscle is not innervated by the pudendal nerve but
rather by innervation that originates the sacral nerve roots (S3-S5) that
o Inferior rectal nerve travels on the superior surface of the pelvic floor (levator ani nerve)
o Obturator nerve
o Hypogastric plexuses
o Pudendal nerve and the perineal branch of S4

30. If G-6-P-D is deficient: Not sure

o Hydrogen peroxide is detoxified to water


o RBC membrane is intact
o Glutathione remains in its reduced form
o Glutathione is oxidized

deficiency leads to an impaired regeneration of reduced


glutathione. , an important antioxidant, which makes. RBCs.

“Courage is found in unlikely places.”


― J.R.R. Tolkien 25
31. Cancer of the prostate can metastasize to the skull via the

o Inferior vena cava


o Portal vein
o Vertebral venous plexus
o Pampiniform plexus

o Vertebral venous plexus also known as Batson’s plexus.


The predilection of prostate cancer to the skull may be due to Batson's
venous plexus that connects the prostate with the skull through epidural
and dural veins. Therefore, there should be a deliberate attempt at
assessing the possibility of skull metastasis in all patients with advanced
prostate cancer.

32. Formation of the peptide bond is inhibited by this antibiotic: Chloramphenicol. Binds to 50S subunit and reversibly inhibits the
formation of peptide bonds by peptidyl transferase associated with the
bacterial ribosome.
o Penicillin
o Cephalexin
o Erythromycin
o Chloramphenicol

33. Ejaculation is the response to the activity of the The sympathetic neurons play the predominant role in the ejaculation
process. Their nerve terminals secrete primarily norepinephrine,

“Courage is found in unlikely places.”


― J.R.R. Tolkien 26
o Obturator nerves although other neurotransmitters such as acetylcholine and
o Sympathetic nerves nonadrenergic/noncholinergic also play important roles.
o Parasympathetic nerves
o Perineal branch of S4

34. Hereditary spherocytosis is secondary to a defect in what red The most common cause of HS is ankyrin deficiency accounting for
cell membrane protein? approximately 50%-60% of cases with varying degrees of anemia ranging
from mild to severe.22 Ankyrin mutations cause HS in association with
both dominant and recessive forms of inheritance.
o Ankyrin
o Anion exchange protein
o Glycophorin
o Spectrin

35. True regarding the biliary tree? Bile is initially secreted from hepatocytes and drains from both lobes of
the liver via canaliculi, intralobular ducts and collecting ducts into the left
o Union of the major and minor pancreatic ducts will form the and right hepatic ducts. These ducts amalgamate to form the common
Ampulla of Vater hepatic duct, which runs alongside the hepatic vein.
o Union of the right and left hepatic ducts will form the
common bile duct As the common hepatic duct descends, it is joined by the cystic duct –
o Union of the cystic duct and common bile duct will form the which allows bile to flow in and out of the gallbladder for storage and
release. At this point, the common hepatic duct and cystic duct combine
common hepatic duct
to form the common bile duct.
o Union of the common hepatic duct and cystic duct will form
the common bile duct
The common bile duct descends and passes posteriorly to the first part
of the duodenum and head of the pancreas. Here, it is joined by the main
pancreatic duct, forming the hepatopancreatic ampulla (commonly
known as the ampulla of Vater) – which then empties into the duodenum
via the major duodenal papilla. This papilla is regulated by a muscular
valve, the sphincter of Oddi.

36. Proteins with garbled amino acid sequence are produced when Transition: this occurs when a purine is substituted with another purine
there is: or when a pyrimidine is substituted with another pyrimidine.

Transversion: when a purine is substituted for a pyrimidine or a


o Missense mutation pyrimidine replaces a purine.
o Deletion frameshift mutation
o Transition point mutation Missense: When base substitution results in the generation of a codon
o Transversion point mutation that specifies a different amino acid and hence leads to a different
polypeptide sequence. Depending on the type of amino acid substitution
the missense mutation is either conservative or nonconservative. For
example if the structure and properties of the substituted amino acid are
very similar to the original amino acid the mutation is said to be
conservative and will most likely have little effect on the resultant
proteins structure / function. If the substitution leads to an amino acid
with very different structure and properties the mutation is
nonconservative and will probably be deleterious (bad) for the resultant
proteins structure / function (i.e. the sickle cell point mutation).

A deletion, resulting in a frameshift, results when one or more base pairs


are lost from the DNA (see Figure above). If one or two bases are deleted
the translational frame is altered resulting in a garbled message and

“Courage is found in unlikely places.”


― J.R.R. Tolkien 27
nonfunctional product. A deletion of three or more bases leaves the
reading frame intact. A deletion of one or more codons results in a
protein missing one or more amino acids. This may be deleterious or not.

37. Which of the following is NOT a boundary of the Calot triangle?

o Cystic artery
o Gastroduodenal artery
o Common hepatic duct
o Inferior surface of the liver

The triangle of Calot is an important landmark whose boundaries include


the common hepatic duct medially, the cystic duct laterally, and the
inferior edge of the liver superiorly.

38. Which structural feature is found in a single stranded DNA


molecule?

o Each purine and pyrimidine pairs with its complementary


base
o Each 3’-5’ phosphodiester bond contains one phosphate
group linking 2 deoxyribose sugars.
o It can have one end with a 5’-phosphate group and the other
end has a 2’ -hydroxyl group
o With sugar and phosphate as backbone and nitrogen bases in
the center which hold the strands together

39. Erection of the penis is a response to the activity of the Erections occur in response to tactile, olfactory, and visual stimuli. The
ability to achieve and maintain a full erection depends not only on the
penile portion of the process but also on the status of the peripheral
o Sympathetic nerves nerves, the integrity of the vascular supply, and biochemical events
o Parasympathetic nerves within the corpora. The autonomic nervous system is involved in
o Ilioinguinal nerves erection, orgasm, and tumescence. The parasympathetic nervous system
o Perineal branch of S4 is primarily involved in sustaining and maintaining an erection, which is
derived from S2-S4 nerve roots.

40. A 35-year-old man with severe hypercholesterolemia has a Familial hypercholesterolaemia (FH), defined as the heritable occurrence
family history of deaths at a young age from heart disease and of severe hypercholesterolaemia with cholesterol deposits in tendons
stroke. Which of the following genes is likely to be defective? and premature heart disease, is caused by at least four genes in sterol
and lipoprotein pathways and displays varying gene-dose effects. The
genes are the low-density lipoprotein (LDL) receptor, apolipoprotein
o LDL receptor (apo) B, proprotein convertase subtilisin/kexin 9, and the autosomal
o Lipoprotein lipase recessive hypercholesterolaemia (ARH) adaptor protein. All of these
o Apolipoprotein E disorders have in common defective clearance of LDL within a complex
o LCAT system of lipid and lipoprotein metabolism and regulation.

“Courage is found in unlikely places.”


― J.R.R. Tolkien 28
41. The antero-posterior diameter of pelvic inlet that can be ● This diameter extends from the sacral promontory to the upper
measured through vaginal exam inner border of the symphysis pubis and measures approximately
11cms. It is also called the obstetrical conjugate. It is the most
important of the pelvic measurements as it is the first boney strait
o Obstetrics conjugate through which the fetus has to pass.
o Diagonal conjugate
o Sagittal conjugate ● The diagonal conjugate is measured antero-posteriorly from the
o Transverse conjugate apex of the pubic arch of the symphysis to the sacral promontory.
It measures about 1.25cm more than the obstetric conjugate and
may be felt during a vaginal examination as part of a pelvic
assessment.

http://learntech.uwe.ac.uk/pelvicanatomy/default.aspx?pageid=2092#
:~:text=The%20diagonal%20conjugate%20is%20measured,part%2
0of%20a%20pelvic%20assessment.

42. A 7-year-old child was brought to the clinic for nutritional


assessment. He has winged scapula and old man facie which can be
found among patients with:

o Kwashiorkor
o Marasmus
o Obesity
o 1° malnutrition

43. The internal anal sphincter receives its nerve supply from the Sympathetic fibers from the superior rectal and hypogastric plexuses
stimulate and maintain internal anal sphincter contraction. The internal
o Obturator nerve anal sphincter is not innervated by the pudendal nerve, which carries
o Hypogastric plexuses somatic (motor and sensory) fibers that provide the innervation to the
o Vagus nerve external anal sphincter.
o Pudendal nerve and the perineal branch of S4

44. How is the DNA template read during transcription?


Transcription is the first step in gene expression, in which information
from a gene is used to construct a functional product such as a protein.
o From 5’ to 3’ end The goal of transcription is to make a RNA copy of a gene's DNA
o From 3’ to 5’ end sequence.
o From C-terminal to N-terminal
o From N-terminal to C-terminal RNA polymerase

The main enzyme involved in transcription is RNA polymerase, which


uses a single-stranded DNA template to synthesize a complementary

“Courage is found in unlikely places.”


― J.R.R. Tolkien 29
strand of RNA. Specifically, RNA polymerase builds an RNA strand in the
5' to 3' direction, adding each new nucleotide to the 3' end of the strand

45. A deep laceration at the anterior axillary fold will traumatize this The anterior axillary fold is formed by the lateral edge of the pectoralis
structure: major muscle;

o Pectoralis major
o Latissimus dorsi
o Coracobrachialis
o Subscapularis muscle

46. This is a hemoglobinopathy whereby the iron in ferrous (Fe 2+) a similar reaction also occurs during the spontaneous oxidation of
state is converted to the ferric (Fe3+) state: haemoglobin to methaemoglobin

o HbS
o α Thalassemia
o Beta Thalassemia
o Methemoglobinemia

“Courage is found in unlikely places.”


― J.R.R. Tolkien 30
47. Insulin lowers the blood glucose level by all of the following
mechanisms, EXCEPT:

o Increasing lipogenesis
o Increasing the synthesis of proteins
o Promoting glycogenolysis
o Promoting the mobilization of glucose transporters

48. What happens if an amino acid undergoes catabolism? - Aspartic acid is converted to oxaloacetate thru TRANSAMINATION.
Transamination reactions combine reversible amination and
o Aspartic acid is converted to oxaloacetate thru non- deamination, and they mediate redistribution of amino groups
oxidative deamination- FALSE among amino acids. It is a biologically important process by which
o Serine is converted to pyruvate thru oxidative living cells reversibly transfer the amino group from an amine (e.g.,
deamination- FALSE γ-aminobutyrate) or α-amino acid (e.g., aspartate) to an α-keto
o Decarboxylation converts glutamic acid to GABA carboxylic acid (e.g., α-ketoglutarate).
o All choices - Serine is converted to pyruvate thru DEHYDRATION deamination.
(serine dehydratase)These enzymes are called dehydratases
REFERENCE: because dehydration precedes deamination. Serine loses a
https://www.sciencedirect.com/topics/neuroscience/transaminati hydrogen ion from its α-carbon atom and a hydroxide ion group
on from its β-carbon atom to yield aminoacrylate.
https://chem.libretexts.org/Bookshelves/Organic_Chemistry/Map
%3A_Organic_Chemistry_(Bruice)/26%3A_The_Organic_Chemistr
y_of_Metabolic_Pathways/26.09%3A_The_Catabolism_of_Protein
s

https://www.sciencedirect.com/topics/pharmacology-toxicology-
and-pharmaceutical-science/glutamate-decarboxylase

49. Which of the following hormones is not a polypeptide? Aldosterone is a steroid.

o ACTH
o FSH
o hCG
o Aldosterone

“Courage is found in unlikely places.”


― J.R.R. Tolkien 31
50. If one mole of Fructose-1,6-bisphosphate undergoes complete
oxidation using the malate shuttle, how many moles of ATP will be
produced (NET)?

o 22
o 28
o 30
o 34

REFERENCE:
https://chem.libretexts.org/Bookshelves/Organic_Chemistry/Ma
p%3A_Organic_Chemistry_(Bruice)/26%3A_The_Organic_Chemi
stry_of_Metabolic_Pathways/26.07%3A_The_Catabolism_of_Ca
rbohydrates

PART 3

1. A 44 year old woman notes a lump in her left breast while taking The lack of metastases suggests a lower stage and a better
a shower. The physician palpates a 3 cm firm, irregular, non- prognosis.
movable mass in the upper outer quadrant of her left breast on
physical examination. A fine needle aspiration of this mass is
performed, and cytologically the cells are consistent with
infiltrating ductal carcinoma. The mass is removed with
lumpectomy along with an axillary lymph node dissection.
Which of the following findings will best predict a better
prognosis for this patient?

● No metastases in the sampled lymph nodes


● Tumor cells strongly estrogen receptor positive
● One relative who had similar type of breast cancer
● Flow cytometry analysis with aneuploidy and a high S-
phase

2. Amoebic dysentery: Rule out Odorless and Bright red blood on stool (Bacillary
Dysentery)
● Odorless
● Bright red blood on the stool Either Few pus cells on the stool or non-adherent stool
● Few pus cells on the stool consistency (Amoebic dysentery)
● Non-adherent stool consistency

3. Biological vector: A biological vector has the pathogens within its body, and delivers
them to new hosts in an active manner, usually a bite.
● Phlebotomus Mosquitoes, ticks, fleas and lice are examples of biological
● Glossina vectors and are often responsible for serious blood-borne
● Musca domestica diseases, such as malaria.
● Anopheles

“Courage is found in unlikely places.”


― J.R.R. Tolkien 32
4. In malaria, the infective stage to man from the insect vector The stage infective for humans is the uninucleate, lancet-
is: shaped sporozoite (approximately 1 × 7 μm). Sporozoites are
produced by sexual reproduction in the midgut of vector
● Gametocytes anopheline mosquitoes and migrate to the salivary gland.
● Schizonts
● Sporozoites
● Merozoites

5. In primary wound healing: Macrophages


The macrophage is the one cell that is truly crucial to wound
● Neovascularization is maximal at day five healing by orchestrating the release of cytokines and stimulating
● Macrophage infiltration occurs at 24 hours many subsequent processes in wound healing. Tissue
● Type I collagen is replaced by type III collagen macrophages are derived from chemotaxis of migrating
● Wound strength is 25% of normal at the end of first monocytes and appear within 24 to 48 hours of injury.
week

6. A substance that is not known to accumulate intracellularly


is:

● Pus
● Melanin
● Hemosiderin
● Tattoo ink
Endogenous pigments:
Hemosiderin-multiple transfusions (end product of erythrocyte
degradation)
Lipofuscin-aging pigments (ROS cause lipid)
Melanin-skin and neurotransmission

7. A host harboring sexually immature or larval stage of a Types of Hosts:-


parasite and in which no fertilization takes place in it: 1. Definitive host:- Depending on the parasitic species, it is either
a host which harbors the adult stage of a parasite or most highly
● Carrier host developed form of the parasite occurs; or sexually mature stages
● Intermediate host of a parasite and fertilization takes place in it, e.g., man is the
● Reservoir host definitive host of Taenia saginata. When the mature or most highly
● Definitive host developed form is not obvious the definitive host is the
mammalian host, e.g., humans are the definitive host for
trypanosomes that cause African trypanosomiasis.
2. Intermediate host:- Is a host harboring sexually immature or
larval stage of a parasite and in which no fertilization takes place
in it. E.g. Cow is the intermediate host for Taenia saginata
Parasitology 14 Amplifier host- Intermediate hosts in which
parasites undergo multiplication.
3. Reservoir host:- A wild or domestic animal which harbors a
parasite and acts as sources of infection to humans.
4. Carrier host:- A host harboring and disseminating a parasite but
exhibiting no clinical sign. 5. Accidental (Incidental) host:- Infection
of a host other than the normal host species. A parasite may or
may not continue full development in this host.

8. A host harboring and disseminating a parasite but exhibiting


no clinical sign:

“Courage is found in unlikely places.”


― J.R.R. Tolkien 33
RATIO BASE ABOVE.
● Reservoir host
● Carrier host
● Intermediate host
● Definitive host

9. Regarding acute inflammation:


2 phase:
● Increased permeability leads to protein depleted
plasma leaking into the tissue 1. Vascular Phase
● Initial vasoconstriction is the result of histamine and
nitric oxide In the vascular phase, small blood vessels adjacent to the injury
● Initial formation of endothelial gaps lasts for only 15- dilate (vasodilatation) and blood flow to the area increases. The
30 minutes endothelial cells initially swell, then contract to increase the
● Stasis occurs due to vasodilation and the larger caliber space between them, thereby increasing the permeability of
of vessels the vascular barrier. This process is regulated by chemical
mediators.

Exudation of fluid leads to a net loss of fluid from the vascular


space into the interstitial space, resulting in oedema (tumour).

The formation of increased tissue fluid acts as a medium for


which inflammatory proteins (such as complement and
immunoglobulins) can migrate through. It may also help to
remove pathogens and cell debris in the area through lymphatic
drainage.

2. Cellular Phase

The predominant cell of acute inflammation is the neutrophil.


They are attracted to the site of injury by the presence of
chemotaxins, the mediators released into the blood
immediately after the insult.

The migration of neutrophils occurs in four stages:


● Margination – cells line up against the endothelium
● Rolling – close contact with and roll along the endothelium
● Adhesion – connecting to the endothelial wall
● Emigration – cells move through the vessel wall to the
affected area

Once in the region, neutrophils recognise the foreign body and


begin phagocytosis, the process whereby the pathogen is
engulfed and contained with a phagosome. The phagosome is
then destroyed via oxygen-independent (e.g. lysozymes) or
oxygen-dependent (e.g. free radical formation) mechanisms.

10. All are true about karyotyping, EXCEPT: The study of chromosomes—karyotyping—is the basic tool of the
cytogeneticist. The usual procedure to examine chromosomes is to
● G-banding is most common staining method employed arrest dividing cells in metaphase with mitotic spindle inhibitors
using Giemsa (e.g., N-diacetyl-N-methylcolchicine [Colcemid]) and then to stain
● Chromosomes are arranged in pairs and increasing the chromosomes. In a metaphase spread, the individual
order of length chromosomes take the form of two chromatids connected at the

“Courage is found in unlikely places.”


― J.R.R. Tolkien 34
● After staining, chromosomes are analyzed under a centromere. A karyotype is obtained by arranging each pair of
microscope and photographed autosomes according to length, followed
● Dividing cells are arrested in metaphase by sex chromosomes. A variety of staining methods have been
developed that allow identification of individual chromosomes on
the basis of a distinctive and reliable pattern of alternating light and
dark bands. The one most commonly used involves a Giemsa stain
and is hence called G banding. A normal male karyotype with G
banding is illustrated in Figure 5-17. With standard G banding,
approximately 400 to 800 bands per haploid set can be detected.
The resolution obtained by banding can be markedly improved by
obtaining the cells in prophase. The individual chromosomes appear
markedly elongated, and as many as 1500 bands

Robbins and Cotran Pathologic Basis of Disease, p.158

11. Klinefelter’s syndrome is characterized by: Klinefelter syndrome can rarely be diagnosed before puberty,
particularly because the testicular abnormality does not develop
● Webbed hands before early puberty. Most patients have a distinctive body habitus
● Short stature with an increase in length between the soles and the pubic bone,
● Cardiac defects which creates the appearance of an elongated body. Also
● Subnormal intelligence characteristic are eunuchoid body habitus with abnormally long legs;
small atrophic testes often associated with a small penis; and lack of
such secondary male characteristics as deep voice, beard, and male
distribution of pubic hair. Gynecomastia may be present. The mean
IQ is somewhat lower than normal, but
mental retardation is uncommon. There is increased incidence of
type 2 diabetes and the metabolic syndrome that gives rise to insulin
resistance. Curiously, mitral valve prolapse is seen in about 50% of
adults with Klinefelter syndrome. There is also an increased
incidence of osteoporosis and fractures due to sex hormonal
imbalance.

Robbins and Cotran Pathologic Basis of Disease, p.165

12. Myelogenous leukemias are caused by the cancerous production Many diseases alter the production of blood cells. The marrow is the
of innate (non-specific) immune system cells: in which tissue is such ultimate source of most cells of the innate and adaptive immune
production most likely to occur? system and responds to infectious or inflammatory challenges by
increasing its output of granulocytes under the direction of specific
● Thymus growth factors and cytokines. By contrast, many other disorders are
● Spleen associated
● Lymph nodes with defects in hematopoiesis that lead to deficiencies of one or
● Bone marrow more types of blood cells. Primary tumors of hematopoietic cells are
among the most important diseases that interfere with marrow
function, but certain genetic diseases, infections, toxins, and
nutritional deficiencies, as well as chronic inflammation from any
cause, can also decrease the production of blood cells by the
marrow.

Robbins and Cotran Pathologic Basis of Disease, p.581

13. Histamine is involved in acute inflammatory responses and is “The response consists of changes in blood flow, an increase in
released from mast cells. Which of the following statements is permeability of blood vessels, and the migration of fluid, proteins,
incorrect? and WBC(leukocytes) from the circulation to the site of tissue
damage.”
● It causes increased permeability of arterioles “Most histamine in the body is generated in granules in mast cells
● It causes constriction of large vessels and in white blood cells called basophils and eosinophils”

“Courage is found in unlikely places.”


― J.R.R. Tolkien 35
● It may be released by physical trauma “When tissue is first injured, the small blood vessels in the
● It is found in blood basophils, platelets and mast cells damaged area constrict momentarily, a process called
vasoconstriction.”

Source:
Role of Histamine in Acute Inflammation (jpsr.pharmainfo.in)

14. Regarding mediators of inflammation: “In cancer cachexia condition, there is alteration in
carbohydrate, protein and fat metabolism. TNF alpha is
● TNF contributes to cachexia of disease responsible for the increase in gluconeogenesis, loss of
● TNF and IL-1 are produced mainly by activated adipose tissue and proteolysis, while causing decrease in
leukocytes protein, lipid and glycogen” (pubmed.ncbi.nlm.nih.gov)
● TNF is a chemokine with chemoattractant properties
● The systemic acute phase response is induced by MIP-
1 and RANTES chemokines

15. Leishmania species are transmitted by: “Leishmaniasis is a vector-borne disease that is transmitted by
sandflies and caused by obligate intracellular protozoa of the
● Tsetse flies genus Leishmania.”
● Sandflies
● Triatomid bugs Source: www.cdc.gov
● House flies

16. Transduction may occur due to the following, EXCEPT:


B. Inflammation
● Lymphatic obstruction
● Inflammation Reception: A cell detects a signaling molecule from the outside of
● Increased intravascular hydrostatic pressure the cell. A signal is detected when the chemical signal (also known
● Decreased intravascular oncotic pressure as a ligand) binds to a receptor protein on the surface of the cell or
inside the cell.

2. Transduction: When the signaling molecule binds the receptor it


changes the receptor protein in some way. This change initiates the
process of transduction. Signal transduction is usually a pathway of
several steps. Each relay molecule in the signal transduction
pathway changes the next molecule in the pathway.

3. Response: Finally, the signal triggers a specific cellular response

17. Complement: Complement pathway is activated by C3 cleavage

● Pathway is inhibited by C3-cleavage Complement proteins are present in inactive forms in the plasma,
● C3 is the most abundant protein in the complement and many of them are activated to become proteolytic enzymes that
family degrade other complement proteins, thus forming an enzymatic
● Activation by the classical pathway involves microbial cascade capable of tremendous amplification. The critical step in
surface antigens complement activation is the proteolysis of the third (and most
● Proteins are usually stored in an active state in abundant) component, C3. Cleavage of C3 can occur by one of
lysosomal molecules three pathways: The classical, alternative and lectin pathway.

Pathway

“Courage is found in unlikely places.”


― J.R.R. Tolkien 36
1. Classical which is triggered by fixation of C1 to
antibody (IgM or IgG) that has
combined with antigen

2. which can be triggered by


Alternative microbial surface molecules (e.g.,
endotoxin, or LPS),
complex polysaccharides, cobra
venom, and other substances, in the
absence of antibody

3. Lectin plasma mannose-binding


lectin binds to Carbohydrates on
microbes and directly activates C1.

Robbin’s 9th ed. Chapter 3 Inflammation and Repair p 84

18. Which of the following changes is associated with irreversible cell


damage?

● Cellular swelling
● Loss of microvilli
● Glycogen depletion
● Flocculent densities in mitochondria

Flocculent densities in the matrix of mitochondria have become quite


important in cell pathology since, when prominent, they indicate
irreversible cell injury. The morphology and chemical nature of these
flocculent densities have been studied in Kidney after various periods
of autolysis in vitro in whole tissue samples and in isolated
mitochondria. After 30 to 60 min of ischemia, flocculent densities
were seen only occasionally and they were most prominent in
samples subjected to mechanical damage during isolation.

19. Common risk factors for arterial thrombo-emboli include the Dilated cardiomyopathy can cause your heart to suddenly
following, EXCEPT: stop beating. Blood clots (emboli). Pooling of blood (stasis) in
the left ventricle can lead to blood clots, which may enter the
● AMI bloodstream, cut off the blood supply to vital organs, and
● Rheumatic heart disease cause stroke, heart attack or damage to other organs
● Paradoxical embolism from venous thrombi
● Cardiomyopathy

20. When the parasite benefited from the host while the host neither Symbiosis is a close relationship between two species in which
benefited or harmed: at least one species benefits. For the other species, the
relationship may be positive, negative, or neutral
● Symbiosis Mutualism is a symbiotic relationship in which both species
● Commensalism benefit.Commensalism is a symbiotic relationship in which
● Mutualism one species benefits while the other species is not
● Parasitism affected.Parasitism is a symbiotic relationship in which one
species (the parasite) benefits while the other species (the
host) is harmed.

“Courage is found in unlikely places.”


― J.R.R. Tolkien 37
21. Monocytes move from the systemic circulatory system into macrophages
general connective tissues, where they differentiate into what
phagocytic cell type? Monocytes, which constitute between 4 and 8 percent of the
total number of white blood cells in the blood, move from the
● T cell blood to sites of infection, where they differentiate further
● B cell into macrophages
● Neutrophil
● Macrophage

22. Regarding collagen: Collagen consists of amino acids bound together to form a
triple helix of elongated fibril known as a collagen helix.
● Non-fibrillar collagen is the most ubiquitous protein in
the body
● Collagen is formed in a triple helix structure
● Collagen is formed in cells as long fibrils
● Type V, non-fibrillar collagen, is seen in basement
membranes

23. A 48 year old woman has a routine physical examination. A 4 cm


diameter non tender mass is palpated in her right breast. The mass
appears fixed to the chest wall. Another 2 cm non-tender mass is
palpable in the left axilla. A chest radiograph reveals multiple 0.5 to 2
cm nodules in both lungs. Which of the following TNM classification
best indicated the stage of her disease?

● T1 N0 M1
● T2 N1 M0
● T3 N0 M0
● T4 N1 M1

“Courage is found in unlikely places.”


― J.R.R. Tolkien 38
24. Habitat of Giardia lamblia:

● Distal part of the large intestine


● Liver and biliary tree
● Colon and rectum
● Mainly in the duodenum and jejunum

25. Angelman syndrome is due to:

● Inversion
● Uniparental disomy
● Mitochondrial disorder
● Digenic inheritance

Reference: BRS pediatrics 2nd edition

26. Apoptosis occurs in all BUT one of the following situations:

● Tumors
● Menstruation
● Bacterial infection
● Embryogenesis

Reference: BRS Embryology 6th edition

Reference: https://embryo.asu.edu/pages/apoptosis-
embryonic-development

“Courage is found in unlikely places.”


― J.R.R. Tolkien 39
27. Histamine exerts its effect during inflammation by:

● Vasoconstriction of post capillary sphincters


● Acting on H2 receptors on mast cells
● Constriction of large arteries
● Causing venular endothelial contraction

Reference:
https://webpath.med.utah.edu/INFLHTML/INFL071.html

28. Metastatic calcification:

● Is encountered in areas of necrosis


● Can be associated with metastatic malignancy
● Can occur with normal serum levels of calcium
● Commonly develops in aging or damaged heart valves

Reference: BRS pathology 5th edition

29. Concerning vascular leakage with acute inflammation: Leakage is restricted to venules of 20 - 60um in diameter

● Chemically mediated endothelial contraction


● Immediate sustained response is due to widening of
intercellular gaps
● Endothelial cell contraction preferentially occurs in arterioles
20-60um in diameter
● Tumor necrosis factor (TNF) affects leakage by direct
endothelial cell necrosis

30. In the mononuclear phagocyte system: Unlike neutrophils, which are short-lived, macrophages can live
for months to years.
● The life span of tissue macrophages is about three days
● Aggregation of macrophages in the kidney are known as Kupffer Kupffer cells phagocytic cell which forms the lining of the
cells sinusoids of the liver and is involved in the breakdown of red
● The half-life of blood monocytes is about one month blood cells
● Chemotactic stimuli for monocytes include C5a and certain
growth factors The half-life of blood monocytes is about 1 day, whereas the
life span of tissue macrophages is several months or years.

“Courage is found in unlikely places.”


― J.R.R. Tolkien 40
31. The parasite derives food and protection from the host without Commensalism is a symbiotic relationship in which one species
causing harm to the host: benefits while the other species is not affected. One species
typically uses the other for a purpose other than food. For
● Opportunistic parasite example, mites attach themselves to larger flying insects to get
● Commensal and opportunistic parasite a “free ride.” Hermit crabs use the shells of dead snails for
● Commensal homes.
● Pathogenic parasite

32. The following are all actions of bradykinin, EXCEPT: Bradykinin is a potent endothelium-dependent vasodilator
and mild diuretic, which may cause a lowering of the blood
● Dilation of blood vessels pressure. It also causes contraction of non-vascular smooth
● Increased vascular permeability muscle in the bronchus and gut, increases vascular
● Smooth muscle contraction permeability and is also involved in the mechanism of pain.
● Activation of complement pathway

33. Type of specimen used for parasitological examination of


Paragonimus westermani:

● Sputum
● Bone marrow aspirate
● Rectal scraping
● Stool

34. Most common cause of calcium oxalate stone formation:

● Hyperuricosuria
● Hyperoxaluria
● Hypercalciuria without hypercalcemia
● Hypercalcemia and hypercalciuria

35. A diabetic patient has an ulcer on their great toe. Which statement
is false regarding it’s healing?

● It will heal by secondary intention


● The amount of acute inflammation is small, slowing
the healing

“Courage is found in unlikely places.”


― J.R.R. Tolkien 41
● Wound contraction required actin containing
fibroblasts
● Substantial scar formation is likely

36. Parasite that can complete its life cycle in a single host:

● Taenia
● Leishmania
● Plasmodium
● Giardia

37. The route of infection is through inhalation of contaminated air: Pinworm infection is spread by the fecal-oral route, that is by the
transfer of infective pinworm eggs from the anus to someone’s
● Strongyloides stercoralis mouth, either directly by hand or indirectly through contaminated
● Trichomonas vaginalis clothing, bedding, food, or other articles.
● Toxoplasma gondii Because of their small size, pinworm eggs sometimes can become
● Enterobius vermicularis airborne and ingested while breathing. (CDC)

38. Which of the following regarding arterial thrombi is TRUE? Mural thrombi can be seen in large vessels such as the heart and
aorta and can restrict blood flow. They are mostly located in the
● Mural thrombi is most commonly found on the femoral descending aorta.
artery
● The tail of thrombi often breaks off and embolized to
distal sites
● Most common sites are cerebral, coronary and
femoral arteries
● It typically creates a long cast of vessel lumen with a
propagational long tail

39. Regarding cutaneous wound healing which statement is correct? Vitamin C is the essential cofactor for the two enzymes required for
collagen synthesis.
● Vitamin C deficiency impairs collagen synthesis in Glucocorticoids seem to hind the wound healing process, causing a
healing tissues decrease in cellular proliferation.
● Early movement of wounds can stimulate the process
of angiogenesis and healing
● Glucocorticoids promote collagen synthesis and
fibrosis of tissues

“Courage is found in unlikely places.”


― J.R.R. Tolkien 42
● Complication from wound contraction are most
commonly seen on the face and neck

40. Source of exposure to cercarial signs of human blood fluke:

● Contaminated water
● Contaminated soil
● Blood sucking arthropods
● Insufficiently cooked meat of pork or beef

41. Which of the following is the least important risk factor for the
development of thrombus?

● Acute myocardial infarction


● Smoking
● Antithrombin III deficiency
● Disseminated malignancy

“Courage is found in unlikely places.”


― J.R.R. Tolkien 43
42. A 15 year old healthy girl with no major medical problems notes Food allergies are typically a type I hypersensitivity reaction.
blotchy areas of erythema that are pruritic over the skin of her arms,
legs and trunk within an hour every time she eats seafood, followed
by diarrhea. These problems abate within 3 hours, and then physical
examination reveals no abnormal findings. Which of the following
immunologic abnormalities is she most likely to have?

● Complement activation
● Hypergammaglobulinemia
● Cell-mediated hypersensitivity
● Localized anaphylaxis

43. Permanent association between two different organisms so


dependent on each other that their life part is impossible: Symbiosis is a term describing any relationship or interaction
between two dissimilar organisms.
● Symbiosis
● Parasitism
● Commensalism
● Mutualism

44. Which of the following statements regarding disseminated


intravascular coagulation (DIC) is FALSE?

● Bleeding diathesis is solely due to thrombocytopenia


● Rapid consumption of platelets, prothrombin and
fibrinogen occurs
● Fibrinogen degradation products (FDPs) increases due
to rapid fibrinolysis
● Multiple thrombi develop in the microcirculation

AC is common in the Ull-rich-Turner syndrome, an aneuploidy


45. Coarctation of the aorta is common in which syndrome: disorder and not a mendelian syndrome.

● Turner’s
● Noonan’s - rare
● Down’s
● Klinefelter’s

“Courage is found in unlikely places.”


― J.R.R. Tolkien 44
46. With regard to mechanisms of cellular adaptation,
which of the following statements is TRUE?

● TNF always acts as an inhibitor of hyperplasia


● Barrett’s metaplasia is squamous to columnar
epithelium in the distal esophagus
● Hyperplasia refers to an increase in cell size
● The myocardium adapts to increased workload by
hypertrophy and hyperplasia

47. Type of specimen used in parasitological examination of


Naegleria fowleri:

● Urine
● Cerebrospinal fluid
● Perianal swab
● Liver aspirate

48. A 51 year old man has worked for 10 years in a factory producing Vinyl chloride monomer is a known cause of angiosarcoma of
plastic pipe but not following safety standards. He has noted weight the liver. It also has other toxic effects on the liver, and it has
loss, nausea, and vomiting worsening over the past 5 months. On recently been suggested that exposure to vinyl chloride also
examination, he is afebrile. There is generalized muscle wasting. causes hepatocellular carcinoma. However, the data on which
Laboratory studies show the serum alkaline phosphatase is 405 U/L this conclusion is based is incomplete.
with AST 47 U/L. ALT 35 U/L, and total bilirubin 1.2 mg/dL. An
abdominal CT scan reveals a 12 cm right liver lobe mass. Liver biopsy
reveals a neoplasm composed of spindle cells forming irregular
vascular channels. With immunohistochemical staining the cells
demonstrate vimentin positivity and cytokeratin negativity.
Exposure to which of the following substances most likely led to
development of this neoplasm

● Cyclophosphamide
● Benzene
● Radon
● Vinyl chloride

49. Review of a series of surgical pathology reports indicates that a


certain type of neoplasm is diagnosed as grade I on a scale of I to IV.
Clinically, some of the patients with this neoplasm are found to have
stage I disease. Which of the following is the best interpretation of
a neoplasm with these designations?

● Arising from epithelium


● Well-differentiated and localized
● May spread via lymphatics and bloodstream
● Unlikely to be malignant

“Courage is found in unlikely places.”


― J.R.R. Tolkien 45
50. Most common site of lodgment of arterial thromboemboli is:

● Kidney
● Brain
● Lower limb
● Upper limb

The presentation of arterial emboli depends on the arterial bed that


is affected. The most common manifestations are strokes and acute
lower limb ischemia. Less frequently, emboli target the upper
extremities, mesenteric or renal arteries.

PART 4

SET B:
1. Non-selective beta adrenergic blockers:

▪ Atenolol
▪ Bisoprolol
▪ Metoprolol
▪ Propranolol

2. Atypical lymphocytosis is most likely to be found in which one Infectious Mononucleosis


of the following diseases? These atypical lymphocytes (ie, Downey cells) actually are
CD8 cytotoxic T cells. B-cell infection caused by EBV leads to
▪ Chronic hepatitis C the transformation of the B cells to immortal plasmacytoid
▪ Rotavirus gastroenteritis cells, which secrete a wide variety of immunoglobulins (eg,
▪ Mononucleosis induced Epstein-Barr virus heterophile antibodies).
▪ Parvovirus infection

3. A child develops impetigo on the right thigh. He was given an Treatment for impetigo
antibiotic that will inhibit cell wall synthesis. Which is the most Options include mupirocin cream or ointment and
likely antibiotics this child would have received? retapamulin ointment
Severe impetigo or widespread prescribe oral antibiotics such
▪ Cephalexin as amoxicillin/ clavulanate, certain cephalosporin or
▪ Erythromycin clindamycin
▪ Ciprofloxacin
▪ Ampicillin Ciprofloxacin (Xacin or acid-Quinolones)
Erythromycin (Mycin- Macrolides)
Ciprofloxacine- 2nd generation Cephalosporin
Ampicillin (Aminopenicillin- for non purulent cellulitis)

“Courage is found in unlikely places.”


― J.R.R. Tolkien 46
4. Vaccination for this hepatic disease is through a viral surface Study confirms that transient HBsAg positivity can occur in
antigen and usually provides immunity patients following HBV vaccination, usually not extending
beyond 14 days post vaccination
▪ Hepatitis A
▪ Hepatitis B Reference : NCBI
▪ Hepatitis C
▪ Hepatitis D

5. Action of acetylcholine: Actions of Acetylcholine


*Muscarinic actions
▪ Miosis -Heart
▪ Tachycardia -Depresses the SA node and thereby reduces the heart rate &
▪ Vasoconstriction force of contraction
▪ High blood pressure Decreases rate of diastolic depolarization and increase
-Intestinal-peristalsis is enhanced, sphincters are relaxed, resulting in refractory period ( through M2 receptors)
rapid forward propulsion of intestinal contents (M1 and M3 receptors) Blood vessels
Urinary bladder-detrusor contracts and trigonal sphincter relaxes thus -Ach relaxes the vascular smooth muscles and dilates the blood
promotes voiding of urine. (M3 receptors) vessels of the skin and mucous membrane
-Bronchial smooth muscle-contraction of bronchial muscle resulting in -The BP falls due to a fall in total peripheral resistance and reflex
bronchospasm (M3 receptors) tachycardia (arteries have no parasympathetic innervations, but
Secretory glands have M3 receptors which release EDRF)
-Enhances the secretion of all glands: saliva, lacrimal, nasopharyngeal, Muscarinic Actions
trachea-bronchial, gastric ( M1 receptor) and intestinal secretions are -Smooth muscle Ach increases the tone of all other (non-vascular)
increased. ( M3 receptors) smooth muscle tone
Eye
-Constriction of pupil (miosis) by contracting the circular muscles of the
iris
-Drainage of aqueous humor is facilitated and intraocular pressure falls.
-Ciliary muscle contracts resulting in spasm of accommodation

6. Reduces smooth muscle and secretory activity of gut: The alkaloid L-(-)-scopolamine [L-(-)-hyoscine] competitively
inhibits muscarinic receptors for acetylcholine and acts as a
▪ Atropine nonselective muscarinic antagonist, producing both peripheral
▪ Scopolamine antimuscarinic properties and central sedative, antiemetic, and
▪ Ipratropium amnestic effects.
▪ Dicyclomine
Anticholinergic agents such as dicyclomine (Bentyl, Axcan Scandipharm) Ipratropium is an acetylcholine antagonist via blockade of
and hyoscyamine (Levsin, Levbid, NuLev, all by Schwarz Pharma) are muscarinic cholinergic receptors. Blocking cholinergic receptors
commonly used in the United States to treat pain associated with decreases the production of cyclic guanosine monophosphate
functional intestinal disorders. These agents are smooth muscle (cGMP). This decrease in the lung airways will lead to decreased
relaxants that block the muscarinic effects of acetylcholine on the
contraction of the smooth muscles.
gastrointestinal tract, thereby relaxing smooth muscle and potentially
reducing spasm and abdominal pain, slowing intestinal motility, and
decreasing diarrhea

Atropine reduces secretions in the mouth and respiratory passages,


relieves the constriction and spasm of the respiratory passages, and may
reduce the paralysis of respiration, which results from actions of the toxic
agent on the central nervous system.

“Courage is found in unlikely places.”


― J.R.R. Tolkien 47
7. Thayer-Martin and Martin-Lewis media are used to isolate and The most widely used N. gonorrhoeae culture media (
identify modified Thayer Martin Medium and Martin-Lewis use
commercial peptone, hemoglobin, Isovitalex or Supplement B.
▪ Neisseria
▪ Salmonella
▪ E. coli
▪ Mycobacterium

8. Which of the following bacterial agents has the lowest The infectious dose for different serovars of Salmonella and
infective dose for producing gastrointestinal disease in the strains of E.coli was quite large ( > 10 ^5 organisms), while the
human host? infectious dose for some Shigella spp. Seemed to be as low as
less than 10 organisms.
▪ Shigella flexneri
▪ Enteropathogenic Escherichia coli
▪ Shigella flexneri
▪ Enterotoxigenic Escherichia coli

9. Beta 1 agonist: Dobutamine - Beta 1 agonist (positive inotropic effect)


Dopamine - Dopamine receptor agonist
▪ Dobutamine Clonidine - Alpha 2 agonist (lowers BP)
▪ Dopamine Propranolol - Non selective beta antagonist
▪ Clonidine
▪ Propranolol

10. Prototype of muscarinic antagonists: Atropine - nonselective competitive antagonist at all


muscarinic receptors in CNS and periphery, prototype of
▪ Atropine muscarinic receptor blocking drugs
▪ Hyoscine Hyoscine (Scopolamine) - unknown mechanism in CNS
▪ Physostigmine Physostigmine - intermediate acting cholinesterase inhibitors
▪ Neostigmine Neostigmine - intermediate acting cholinesterase inhibitors

11. The primary transmitter at ANS ganglia, at the somatic Acetylcholine - primary transmitter at ANS ganglia, st the
neuromuscular junction and at parasympathetic somatic neuromuscular junction, and at parasympathetic
postganglionic nerve endings: postganglionic nerve endings.

▪ Acetylcholine
▪ Dopamine
▪ Cholecystokinin
▪ Adenosine triphosphate

12. Adrenaline is conjugated and oxidized in the: Liver is the principal site of metabolism. For many drugs,
metabolism occurs in 2 phases.
▪ Spleen Phase I reactions involve formation of a new or modified functional
▪ Small intestine group or cleavage (oxidation, reduction, hydrolysis); these
▪ Pancreas reactions are nonsynthetic. Phase II reactions involve conjugation
▪ Liver with an endogenous substance (eg, glucuronic acid, sulfate,
glycine); these reactions are synthetic.

“Courage is found in unlikely places.”


― J.R.R. Tolkien 48
13. Contraindication/s for the use of adrenaline:

▪ Hypertension
▪ Coronary artery disease
▪ Hyperthyroidism
▪ Hypertension and coronary artery disease
▪ Hypertension, hyperthyroidism and coronary artery
disease

14. Alpha 1 agonist Prazosin, a specific α1-adrenergic receptor antagonis

▪ Prazosin
▪ Phenylephrine
▪ Propranolol
▪ Dobutamine

15. Which of the following is a leading cause of pneumonia Rhinovirus has more than 100 serotypes; it is unlikely that a
primarily in infants? unifying vaccine will be developed. However, the use of antivirals
as chemoprophylaxis may have practical value. Topical application
▪ Mumps of interferon in the nose has been shown to be effective in reducing
▪ Rhinovirus the incidence of colds in people who are exposed to others with a
▪ Respiratory syncytial virus fresh cold.This strategy reduced the overall risk of cold by 40% and
▪ Cytomegalovirus almost eliminated proven rhinovirus colds in contacts.

16. The science of substances used to prevent, diagnose, and treat Pharmacology is the study of how a drug affects a biological
disease: system and how the body responds to the drug. The discipline
encompasses the sources, chemical properties, biological
▪ Pharmacology effects and therapeutic uses of drugs. These effects can be
▪ Pharmacogenomics therapeutic or toxic, depending on many factors.
▪ Medical pharmacology
▪ Toxicology

17. Direct mode of action of adrenergic drugs: Adrenergic receptors, otherwise known as adreno-receptors,
are classified as either alpha or beta receptors. Those two
▪ Displacement of stored catecholamines from the classes further subdivide into alpha-1, alpha-2, beta-1, beta-2,
adrenergic nerve endings and beta-3. Alpha-1 and alpha-2 receptors both have three
▪ Interact and active adrenoreceptors subtypes. These receptors are all G-protein-coupled
▪ Inhibit the enzyme cholinesterase receptors.
▪ Inhibition of reuptake of catecholamines already releases

“Courage is found in unlikely places.”


― J.R.R. Tolkien 49
18. Lower blood pressure by depleting the body sodium and
reducing blood volume:

▪ ACE inhibitor
▪ Calcium channel blockers
▪ DIURETICS
▪ Beta blocker

Most diuretics produce diuresis by inhibiting the reabsorption


of sodium at different segments of the renal tubular system

19.

20. Which one of the following represents the mechanism of The generally accepted mode of action of azoles is the
action of the antifungal azoles? inhibition of 14α-lanosterol demethylase, a key enzyme in
ergosterol biosynthesis, resulting in depletion of ergosterol
▪ Inhibition of GTP hydrolysis carried out by elongation of and accumulation of toxic 14α-methylated sterols in
factor 2 (EF-2) membranes of susceptible yeast species.
▪ Inhibition of the activity of enzyme, ß-glucan synthase
▪ Inhibition of the activity of the enzyme, 14-a-
demethylase
▪ Inhibition of the activity of the enzyme, chitin synthase

21. Full scale evaluation of treatment comparing it with standard


treatment is done:

▪ Phase 1 Clinical Trial


▪ Phase 2 Clinical Trial
▪ Phase 3 Clinical Trial
▪ Phase 4 Clinical Trial

22. The relation of the individual’s genetic make-up to his/her PHARMACOGENOMICS (pharmocogenetics): is a field of
response to specific drugs: research that studies fow a person’s genes affect how he or
she responds to medications
▪ Toxicology
▪ Pharmacology
▪ Pharmacogenomics
▪ Medical Pharmacology

23. Side effect/s of Methyldopa


(Katzung page 180)
▪ Sedation
▪ Vertigo
▪ Nausea
▪ Sedation and Vertigo
▪ Sedation, Vertigo, and nausea

“Courage is found in unlikely places.”


― J.R.R. Tolkien 50
Common s/e: nausea

24. Which of the following hepatitis viruses is known as defective, Hepatitis D is a liver disease in both acute and chronic forms
for it cannot replicate independently without the presence of caused by the hepatitis D virus (HDV) that requires HBV for its
hepatitis B virus? replication. Hepatitis D infection cannot occur in the absence
of hepatitis B virus.- WHO
▪ Hepatitis A
▪ Hepatitis B
▪ Hepatitis C
▪ Hepatitis D

25. Angiotensin converting enzyme inhibitor Examples of ACE inhibitors include:

▪ Enalapril Benazepril (Lotensin)


▪ Losartan Captopril
▪ Amlodipine Enalapril (Vasotec)
▪ Metoprolol Fosinopril
Lisinopril (Prinivil, Zestril)

26. Non-pharmacological approaches to therapy of hypertension: The American Heart Association/American College of Cardiology
lifestyle management guideline recommends a diet emphasizing
▪ Weight reduction vegetables, fruits, and whole grains; limiting sodium intake to
▪ Cessation of smoking less than 2,400 mg per day; and exercising three or four times
▪ Low sodium chloride diet per week for an average of 40 minutes per session. Other
▪ Low sodium diet and weight reduction nonpharmacologic strategies include weight loss, tobacco
▪ Low sodium diet, weight reduction and cessation of smoking cessation, decreased alcohol consumption, biofeedback, and
self-measured blood pressure monitoring. For patients with
obstructive sleep apnea, the use of continuous positive airway
pressure has been shown to improve blood pressure.

27. Main action of Propranolol, EXCEPT: Bronchoconstriction: may exacerbate asthma/COPD


Vasoconstriction: avoid in patients with peripheral vascular disease
▪ Bronchodilation
▪ Bradycardia Hypoglycemia and hyperglycemia
▪ Hyperglycemia Bradycardia and syncope
▪ Increase renin secretion

28. The principal action of alpha adrenergic blocking agent is: Alpha-1 blockers (also called alpha-adrenergic blocking agents)
constitute a variety of drugs that block the effect of alpha-1-
▪ Central vasodilation adrenergic receptors. ... When alpha blockers bind to these
▪ Central vasoconstriction receptors in vascular smooth muscle, they cause vasodilation.
▪ Peripheral vasodilation

“Courage is found in unlikely places.”


― J.R.R. Tolkien 51
▪ Peripheral vasoconstriction-samplex

29. The action of the body on the drug:

▪ Pharmacogenomic
▪ Pharmacodynamic
▪ Pharmacokinetic
▪ Pharmacogenetic

30. Effect of atropine

▪ Tachycardia
▪ Bronchoconstriction
▪ High doses produce sedation
▪ Increased GUT motility and secretions

31. The presence of Negri inclusion bodies in host cells is


characteristic of:

▪ Mumps
▪ Rabies
▪ Aseptic meningitis
▪ Infectious mononucleosis

32. Formed by decarboxylation of histidine:

▪ Histamine
▪ Prostaglandin
▪ Cromolyn sodium
▪ Ipratropium

“Courage is found in unlikely places.”


― J.R.R. Tolkien 52
33. Shingles, a disease caused by varicella, belongs to which Varicella Zoster Virus is the causative agent of chicken pox and
member of the viral families? shingles and is one of eight herpes virus that have humans as
natural host.
▪ Herpesvirus
▪ Paramyxovirus
▪ Adenovirus
▪ Poxvirus

34. It blocks sodium channel so that there is an increase in the Quinidine acts as a blocker of voltage gated Na channels. The
threshold of excitability: quinidine effects on Na channels is known as a use of
dependent block. This means the higher rates, the block
▪ Hydrochlorothiazide increases, while at lower rates the block decreases.
▪ Propranolol
▪ Quinidine
▪ Verapamil

35. Comprises small scale trials on patients used to determine Phase 1 Clinical Trial - this phase aims to figure out the highest
dose level and establish that the treatment offers some dose humans can take without serious side effects.
benefit: Investigators also look the best way to administer the drug.

▪ Phase 1 Clinical Trial


▪ Phase 2 Clinical Trial
▪ Phase 3 Clinical Trial
▪ Phase 4 Clinical Trial

36. Classified as a calicivirus this hepatitis and is acquired via fecal- Hepatitis E - is acquired in contamination of water supply such
oral route. Common reservoirs are pigs and humans. as after monsoon flooding, but sporadic isolated cases occur.
Reports suggest a zoonotic reservoir for HEV in swine.
▪ Hepatitis B Transmitted via fecal oral route.
▪ Hepatitis C
▪ Hepatitis D
▪ Hepatitis E

37. Which of the following causes a fatal encephalitis for which a


vaccine is available? Encephalitis, or inflammation of the brain, is a treatable but
potentially-serious medical condition that can result from immune
▪ Rabies
▪ Mumps system disorders. This includes disorders triggered by the MMR
▪ Rhinovirus vaccine. According to the CDC:
▪ Cytomegalovirus
“Measles inclusion body encephalitis, or severe brain swelling
caused by the measles virus, is a complication of getting infected
with the wild-type measles virus. While rare, this disorder almost
always happens in patients with weakened immune systems. The
illness usually develops within 1 year after initial measles infection
and has a high death rate.”

38. Cholinesterase regenerator: Pralidoxime (2-PAM)


Drug Class: CHOLINESTERASE REGENERATOR, “ OXIMES”

“Courage is found in unlikely places.”


― J.R.R. Tolkien 53
▪ Ipratropium MOA: regenerates cholinesterase activity that has become
▪ Atropine inhibited by organophosphate compounds .
▪ Pralidoxime
▪ Scopolamine

39. Antitussive: DEXTROMETHORPHAN


Drug Class: ANTITUSSIVE a cough suppressant.
▪ Salbutamol
▪ Carbocysteine
▪ Guaifenesin
▪ Dextromethorphan

40. Orchitis, which may cause sterility, is a possible manifestation Mumps


of which of the following? Orchitis is an inflammation of the testicles. The most common
cause of viral orchitis is MUMPS.
▪ Mumps
▪ Rabies
▪ Cytomegalovirus
▪ Rhinovirus

41. Indication for physostigmine: Physostigmine salicylate has FDA approval for use in the
treatment of glaucoma, as well as the treatment of
▪ Paralytic ileus anticholinergic toxicity.
▪ Postoperative retention of urine
▪ Coronary insufficiency
▪ Glaucoma

42. Exert antimicrobial action against H. pylori: Clarithromycin is one of the most useful antimicrobials
against H. pylori. It is an acid-stable macrolide with a broad
▪ Cimetidine spectrum of antibacterial activity, well absorbed with a wide
▪ Clarithromycin tissue distribution and with mild side effects
▪ Magnesium hydroxide
▪ Sucralfate

43. A 8 year old boy developed bloody diarrhea after eating a E. coli 0157: H7 or STC or Shiga toxin producing E.coli was first
poorly cooked hamburger. A few days later, he was recognized as a cause of illness in 1982 during an outbreak of
hospitalized with oliguria and hypertension. With which severe bloody diarrhea; the outbreak was traced to
organism is he most likely infected? contaminated hamburgers. Since then, most infections have
come from eating undercooked ground beef.
▪ Shiga toxin-producing E. coli
▪ Enteroinvasive E. coli
▪ Salmonella typhi
▪ Clostridium perfringens

44. Adverse effect of propranolol: Principal toxicities of propranolol result from blockade of cardiac,
vascular, or bronchial B receptors .
▪ Bronchodilation In mild to moderate hypertension, propranolol produces a
▪ Heart block significant reduction in blood pressure without prominent postural
▪ Hypotension hypotension.

“Courage is found in unlikely places.”


― J.R.R. Tolkien 54
▪ Heart block and hypotension
▪ Heart block, hypotension and bronchodilation Worsen heart block

45. Coarctation of the aorta is common in which syndrome: Coarctation of the aorta is the most common cardiac defect
associated with Turner syndrome.
▪ Turner’s
▪ Klinefelter’s
▪ Noonan’s
▪ Down’s

46. Reduce gastric acid secretion: Examples of H2 antagonists commonly used to suppress
gastric acid secretion include cimetidine, Ranitidine,
▪ Sucralfate famotidine, and nizatidine.
▪ Cimetidine
▪ Clarithromycin
▪ Magnesium sulfate

47. Traditional vaccination for the common cold is virtually Rhinovirus has more than 100 serotypes; it is unlikely that a
impossible because there are multiple serotypes of which one unifying vaccine will be developed. However, the use of antivirals
of the following viruses? as chemoprophylaxis may have practical value. Topical application
of interferon in the nose has been shown to be effective in reducing
▪ Mumps the incidence of colds in people who are exposed to others with a
▪ Cytomegalovirus fresh cold [79]. This strategy reduced the overall risk of cold by 40%
▪ Rhinovirus and almost eliminated proven rhinovirus colds in contacts.
▪ Respiratory syncytial virus

48. The action of the drug on the body: Body to drugs pharmacokinetic

▪ Pharmacokinetic
▪ Pharmacodynamic
▪ Pharmacogenetic
▪ Pharmacogenomic

49. Drugs that mimic the action of acetylcholine A parasympathomimetic drug, sometimes called a
cholinomimetic drug or cholinergic receptor stimulating
▪ Sympatholytic drugs agent, is a substance that stimulates the parasympathetic
▪ Sympathomimetic drugs nervous system (PSNS). These chemicals are also called
▪ Parasympatholytic drugs cholinergic drugs because acetylcholine (ACh) is the
▪ Parasympathomimetic drugs neurotransmitter used by the PSNS
Sympathomimetic drugs are agents which in general mimic responses
due to stimulation of sympathetic nerves. These agents are able to ▪ A parasympatholytic agent is a substance or activity
directly activate adrenergic receptors or to indirectly activate them by that reduces the activity of the parasympathetic
increasing norepinephrine and epinephrine (mediators of the nervous system. (The parasympathetic nervous
sympathoadrenal system) levels. system is often colloquially described as the "Feed
Sympathomimetic drugs (also known as adrenergic drugs and and Breed" or "Rest and Digest" portion of the
adrenergic amines) are stimulant compounds which mimic the effects of autonomic nervous system.
endogenous agonists of the sympathetic nervous system. ▪ Parasympatholytic (anticholinergic, or
antimuscarinic) agents that are given by aerosol
A sympatholytic (or sympathoplegic) drug is a medication that opposes include ipratropium, a combination of ipratropium
the downstream effects of postganglionic nerve firing in effector organs and albuterol, and tiotropium.
innervated by the sympathetic nervous system (SNS). They are indicated .

“Courage is found in unlikely places.”


― J.R.R. Tolkien 55
for various functions; for example, they may be used as
antihypertensives
Drugs that inhibit the actions of the sympathetic nervous system by any
mechanism. The most common of these are the ADRENERGIC
ANTAGONIST

50. Adrenoceptor antagonist, EXCEPT: Prazosin, a specific α1-adrenergic receptor antagonist, has been
used alone or with other medications to treat cardiovascular
▪ Labetalol diseases such as hypertension and congestive heart failure.
▪ Methyldopa
An adrenergic antagonist is a drug that inhibits the function of
▪ Propranolol
adrenergic receptors. There are five adrenergic receptors, which
▪ Prazosin
are divided into two groups. The first group of receptors are the
Methyldopa is in the alpha-2 adrenergic receptor agonist family of
beta (β) adrenergic receptors. There are β1, β2, and β3 receptors.
medication. It works by stimulating the brain to decrease the activity of The second group contains the alpha (α) adrenoreceptors. There
the sympathetic nervous system are only α1 and α2 receptors. Adrenergic receptors are located
near the heart, kidneys, lungs, and gastrointestinal tract.[1] There
are also α-adreno receptors that are located on vascular smooth
muscle.[2]
Antagonists reduce or block the signals of agonists. They can be
drugs, which are added to the body for therapeutic reasons, or
endogenous ligands. The α-adrenergic antagonists have different
effects from the β-adrenergic antagonists.

Alpha blockers

● Phentolamine
● phenoxybenzamine
● Tamsulosin

Beta blockers

● Propranolol
● Nebivolol
● Atenolol
● Oxprenolol
● Metoprolol
● Timolol
● Pindolol
● Nadolol
● Pindolol
● Esmolol
● Acebutolol
● Sotalol
● Talinolol
● Betaxolol

Mixed action

● Labetalol
● Carvedilol

SET C:

“Courage is found in unlikely places.”


― J.R.R. Tolkien 56
REVALIDA BASIC PART 4
December 6, 2020

1. Formed by decarboxylation of histidine: Histamine is formed by decarboxylation of histidine


A. Ipratropium catalyzed by histidine decarboxylase. It is stored preformed
B. Prostaglandin in cytoplasmic granules of mast cells and basophils, and within
C. Cromolyn sodium platelets of some species. Synthesis occurs also in other cells
D. Histamine within the gastric mucosa, nervous tissue, and lymphatic

2. Non-selective beta-adrenergic blockers:


A. Propranolol
B. Metoprolol
C. Atenolol
D. Bisoprolol

3. Action of acetylcholine -Intestinal-peristalsis is enhanced, sphincters are relaxed,


A. Miosis resulting in rapid forward propulsion of intestinal contents
B. Tachycardia (M1 and M3 receptors)
C. Vasoconstriction Urinary bladder-detrusor contracts and trigonal sphincter
D. High blood pressure relaxes thus promotes voiding of urine. (M3 receptors)
-Bronchial smooth muscle-contraction of bronchial muscle
resulting in bronchospasm (M3 receptors)
Secretory glands
-Enhances the secretion of all glands: saliva, lacrimal,
nasopharyngeal, trachea-bronchial, gastric ( M1 receptor) and
intestinal secretions are increased. ( M3 receptors)
Eye
-Constriction of pupil (miosis) by contracting the circular
muscles of the iris
-Drainage of aqueous humor is facilitated and intraocular
pressure falls.
-Ciliary muscle contracts resulting in spasm of accommodation

4. Direct mode of action of adrenergic drugs: Drugs that mimic the actions of epinephrine or
A. Inhibit the enzyme cholinesterase norepinephrine have traditionally been termed
B. Interact and activate adrenoreceptors sympathomimetic drugs. The sympathomimetics can be
C. Displacement of stored catecholamines from grouped by mode of action and by the spectrum of receptors
the adrenergic nerve endings that they activate. Some of these drugs (eg, norepinephrine
D. Inhibition of reuptake of catecholamines and epinephrine) are direct agonists; that is, they directly
interact with and activate adrenoceptors. Others are indirect
already released
agonists because their actions are dependent on their ability
Sympathomimetic drugs (also known as adrenergic drugs to enhance the actions of endogenous catecholamines. These
and adrenergic amines) are stimulant compounds which indirect agents may have either of two different mechanisms:

“Courage is found in unlikely places.”


― J.R.R. Tolkien 57
mimic the effects of endogenous agonists of the sympathetic (1) they may displace stored catecholamines from the
nervous system. adrenergic nerve ending (eg, the mechanism of action of
tyramine), or they may decrease the clearance of released
norepinephrine either by (2a) inhibiting reuptake of
catecholamines already released (eg, the mechanism of action
of cocaine and tricyclic

5. Adrenoreceptor antagonist, EXCEPT:


A. Methyldopa Prazosin, a specific α1-adrenergic receptor antagonist, has
B. Labetalol been used alone or with other medications to treat
C. Prazosin cardiovascular diseases such as hypertension and congestive
D. Propranolol heart failure.
An adrenergic antagonist is a drug that inhibits the function
of adrenergic receptors. There are five adrenergic receptors,
which are divided into two groups. The first group of receptors
Methyldopa is in the alpha-2 adrenergic receptor agonist
are the beta (β) adrenergic receptors. There are β1, β2, and
family of medication. It works by stimulating the brain to
β3 receptors. The second group contains the alpha (α)
decrease the activity of the sympathetic nervous system adrenoreceptors. There are only α1 and α2 receptors.
Adrenergic receptors are located near the heart, kidneys,
lungs, and gastrointestinal tract.[1] There are also α-adreno
receptors that are located on vascular smooth muscle.[2]
Antagonists reduce or block the signals of agonists. They can
be drugs, which are added to the body for therapeutic
reasons, or endogenous ligands. The α-adrenergic antagonists
have different effects from the β-adrenergic antagonists.

Alpha blockers

● Phentolamine
● phenoxybenzamine
● Tamsulosin

Beta blockers

● Propranolol
● Nebivolol
● Atenolol
● Oxprenolol
● Metoprolol
● Timolol
● Pindolol
● Nadolol
● Pindolol
● Esmolol
● Acebutolol
● Sotalol
● Talinolol
● Betaxolol

Mixed action

● Labetalol
● Carvedilol

“Courage is found in unlikely places.”


― J.R.R. Tolkien 58
6. Reduces smooth muscle and secretory activity of
the gut: Anticholinergic agents such as dicyclomine (Bentyl, Axcan
A. Dicyclomine Scandipharm) and hyoscyamine (Levsin, Levbid, NuLev, all by
B. Atropine Schwarz Pharma) are commonly used in the United States to
C. Scopolamine treat pain associated with functional intestinal disorders.
D. Ipratropium These agents are smooth muscle relaxants that block the
muscarinic effects of acetylcholine on the gastrointestinal
The alkaloid L-(-)-scopolamine [L-(-)-hyoscine] competitively tract, thereby relaxing smooth muscle and potentially
inhibits muscarinic receptors for acetylcholine and acts as a reducing spasm and abdominal pain, slowing intestinal
nonselective muscarinic antagonist, producing both motility, and decreasing diarrhea
peripheral antimuscarinic properties and central sedative,
Atropine reduces secretions in the mouth and respiratory
antiemetic, and amnestic effects.
passages, relieves the constriction and spasm of the
Ipratropium is an acetylcholine antagonist via blockade of respiratory passages, and may reduce the paralysis of
muscarinic cholinergic receptors. Blocking cholinergic respiration, which results from actions of the toxic agent on
the central nervous system.
receptors decreases the production of cyclic guanosine
monophosphate (cGMP). This decrease in the lung airways
will lead to decreased contraction of the smooth muscles.

7. Angiotensin converting enzyme inhibitor: Ace Inhibitor- PRILs


A. Enalapril ARBs- TANs
B. Metoprolol Beta blocker- LOL
C. Losartan
D. Amlodipine Calcium channel blockers (VERAPan Found a LAND):
VERAPAm- Verapamil
F-felodipine
L-lacidipine
A-amlodipine
N-nifedipine, natindipine
D-diltiazem

8. Exert antimicrobial action against H. pylori Clarithromycin is resistant to gastric acid, penetrates in high
A. Magnesium hydroxide concentrations into gastric tissue and mucus, shows excellent
B. Sucralfate antimicrobial activity against H. pylori, results in a high cure
C. Clarithromycin rate when used in two- and three-drug combinations, is
D. Cimetidine associated with a low incidence of acquired.

9. Effect of atropine Atropine -Dilates pupil, increase heart rate and reduce
A. Tachycardia salivation and other secretions
B. Bronchoconstriction
C. High doses produce sedation
D. Increased GIT motility and secretions

“Courage is found in unlikely places.”


― J.R.R. Tolkien 59
10. Full scale evaluation of treatment comparing it with Phase 0
standard treatment is done:
A. Phase 4 clinical trial Phase 0 trials are the first clinical trials done among people.
B. Phase 1 clinical trial They aim to learn how a drug is processed in the body and how
C. Phase 2 clinical trial it affects the body. In these trials, a very small dose of a drug
D. Phase 3 clinical trial is given to about 10 to 15 people.

Phase III Phase I

Phase III trials compare a new drug to the standard-of-care Phase I trials aim to find the best dose of a new drug with the
drug. These trials assess the side effects of each drug and fewest side effects. The drug will be tested in a small group of
which drug works better. Phase III trials enroll 100 or more 15 to 30 patients.
patients.
Phase II
Phase III clinical trials compare the safety and effectiveness Phase II trials further assess safety as well as if a drug works.
of the new treatment against the current standard The drug is often tested among patients with a specific type of
treatment.
cancer. Phase II trials are done in larger groups of patients
compared to Phase I trials.

11. Neutralize gastric acid: Magnesium hydroxide- increases peristaltic activity causing
A. Sucralfate osmotic retention of fluids. It reduces stomach acid by
B. Magnesium hydroxide reacting with hydrochloric acid to form Mg Chloride.
C. Clarithromycin
D. Cimetidine

12. Beta 1 agonist:


A. Dopamine
B. Propranolol
C. Clonidine
D. Dobutamine

“Courage is found in unlikely places.”


― J.R.R. Tolkien 60
13. Reduce gastric acid secretion:
A. Cimetidine H2 Receptor Antagonists (TIDINE)
B. Sucralfate
C. Clarithromycin Histamine is clearly one of the primary regulators of acid
D. Magnesium hydroxide secretion, and the parietal cell receptor for histamine is of the
H2 type. Evidence of histamine's role in acid secretion is
Clarithromycin is resistant to gastric acid, penetrates in high strongly supported by finding that H2 receptor antagonists
concentrations into gastric tissue and mucus, shows excellent are quite effective in inhibiting acid secretion. Examples of H2
antimicrobial activity against H. pylori, results in a high cure antagonists commonly used to suppress gastric acid secretion
rate when used in two- and three-drug combinations, is include cimetidine (Tagamet HB), ranitidine (Zantac 75),
associated with a low incidence of acquired. famotidine (Pepcid AC) and nizatidine (Axid AR).

It was believed that the formation of a physical protective These drugs, particularly cimetidine, are among the most
barrier was the major mechanism by which sucralfate assisted widely prescribed drugs in man. They are also useful for
ulcer healing. However, it is now believed that the major drug management of certain gastric diseases in dogs and horses.
actions of sucralfate are related to stimulation of mucosal Antihistamines that engage H1 receptors (e.g. those used to
defense and reparative mechanisms, possibly related to treat colds) have no effect on acid secretion.
stimulation of local PGE2 and PGI2 production. Sucralfate also
inactivates pepsin, adsorbs bile acids and is believed to be Proton Pump Inhibitors
cytoprotective by stimulating prostaglandin synthesis. It does
not significantly affect gastric acid output but may slow Acid secretion is absolutely dependent on the function of the
gastric emptying appreciably. H+/K+ ATPase or proton pump located in the cannilicular
membrane of the parietal cell. Several drugs have been
Sucralfate suppresses Helicobacter pylori infection and developed that non-competively bind and inactivate the
reduces gastric acid secretion by 50% in patients with ATPase, resulting in strong inhibition of acid secretion.
duodenal ulcer. Omeprazole (Prilosec) is an acid-activated prodrug that binds
covalently to two cysteines on the ATPase, resulting in its
As an antacid, magnesium hydroxide is dosed at irreversible inactivation. Other inhibitors, including
approximately 0.5–1.5 g in adults and works by simple lansoprazole (Prevacid), esomeprazole (Nexium), rabeprazole
neutralization, where the hydroxide ions from the Mg(OH)2 (Aciphex) and pantoprazole (Protonix) have similar modes of
combine with acidic H+ ions produced in the form of action.
hydrochloric acid by parietal cells in the stomach to produce
water References and Reviews

14. The actions of the body on the drug: PharmacoKINETICS is the study of what the body does to a
A. Pharmacokinetic drug (Kinetic-to move-Remember Galaw galaw KATAWAN for
B. Pharmacogenetic Kinetic- katawan sa Drug)

“Courage is found in unlikely places.”


― J.R.R. Tolkien 61
C. Pharmacogenomic PharmacoDynamics is the study of what the drug does to the
D. Pharmacodynamic body (Dynamic-Drug sa Katawan)

Pharmacogenetics- study of how genetic differences in a Pharmacogenomics- study how genetic (genome) differences
SINGLE gene influence variability in drug response (efficacy & in MULTIPLE genes influence variability in drug response (
toxicity) efficacy & toxicity) (MIXs_ MADAMI-Multiple)

15. Produce localized vasoconstriction on the small


blood vessel of the nasal membrane:
A. Phenylpropanolamine
B. Terbutaline
C. Dextromethorphan
D. Guaifenesin
Phenylpropanolamine is a sympathomimetic agent that
acts as a nonselective adrenergic receptor agonist and
norepinephrine reuptake inhibitor. It has been used as a
decongestant and appetite suppressant

16. Cholinesterase regenerator: Pralidoxime (2-PAM)


A. Atropine Mechanism of Action: regenerates cholinesterase activity
B. Pralidoxime that has become inhibited by an organophosphate compound
C. Ipratropium (but is most effective when given before chemical “aging”
D. Scopolamine between the organophosphate & cholinesterase enzyme has
occurred)

17. Adverse effect of propranolol Principal toxicities of propranolol result from blockade of
A. Heart block cardiac, vascular, or bronchial B receptors .
B. Heart block, hypertension and bronchodilation In mild to moderate hypertension, propranolol produces a
C. Hypertension significant reduction in blood pressure without prominent
D. Heart block and hypertension postural hypotension.
E. Bronchodilation

18. Antitussive: Albuterol (also known as salbutamol) is used to treat


A. Salbutamol wheezing and shortness of breath caused by breathing
B. Dextromethorphan problems such as asthma. It is a quick-relief medication.
C. Guaifenesin Albuterol belongs to a class of drugs known as
D. Carbocysteine bronchodilators.

Dextromethorphan is in a class of medications called


antitussives.

Guaifenesin is an expectorant that helps thin and loosen


mucus in the lungs, making it easier to cough up the mucus

Carbocysteine -A mucolytic

19. The primary transmitter at ANS ganglia, at the In the autonomic nervous system, acetylcholine (ACh) is the
somatic neuromuscular junction and at neurotransmitter in the preganglionic sympathetic and
parasympathetic postganglionic nerve endings: parasympathetic neurons. These are shown in Figure 11.2 as

“Courage is found in unlikely places.”


― J.R.R. Tolkien 62
A. Dopamine the red ACh in the ganglion. ACh is also the neurotransmitter
B. Cholecystokinin at the adrenal medulla and serves as the neurotransmitter at
C. Adenosine triphosphate all the parasympathetic innervated organs. ACh is also the
D. Acetylcholine neurotransmitter at the sweat glands, and at the piloerector
muscle of the sympathetic ANS

20. Comprises small scale trial on patients used to Phase 0


determine dose level and establish that the
Phase 0 trials are the first clinical trials done among people.
treatment offers some benefit:
They aim to learn how a drug is processed in the body and how
A. Phase 2 Clinical Trial it affects the body. In these trials, a very small dose of a drug
B. Phase 1 Clinical Trial is given to about 10 to 15 people.
C. Phase 4 Clinical Trial
D. Phase 3 Clinical Trial Phase I
Phase I trials aim to find the best dose of a new drug with the
fewest side effects. The drug will be tested in a small group
of 15 to 30 patients.

Phase II
Phase II trials further assess safety as well as if a drug works.
The drug is often tested among patients with a specific type of
cancer. Phase II trials are done in larger groups of patients
compared to Phase I trials.

Phase III
Phase III trials compare a new drug to the standard-of-care
drug. These trials assess the side effects of each drug and
which drug works better. Phase III trials enroll 100 or more
patients.

“Courage is found in unlikely places.”


― J.R.R. Tolkien 63
21. Loop Diuretics:
A. Furosemide
B. Spironolactone
C. Thiazides
D. Clonidine

22. Cholinergic drugs are also called:


A. Sympatholytic drugs
B. Sympathomimetic drugs
C. Parasympathomimetic drugs
D. Parasympatholytic drugs

23. Alpha 1 adrenoreceptor action, EXCEPT:


A. Contraction of most vascular smooth muscle
B. Mydriasis
C. Increase force of contraction of the heart
D. Glycogenolysis

“Courage is found in unlikely places.”


― J.R.R. Tolkien 64
24. Alpha 2 antagonist:
A. Oxymetazoline
B. Phentolamine
C. Isoproterenol
D. Clonidine

Oxymetazoline- decongestant
Isoproterenol - non selective beta adrenergic agonist
Clonidine- Alpha2 agonist

25. Indication for Physostigmine Physostigmine salicylate has FDA approval for use in the
A. Glaucoma treatment of glaucoma, as well as the treatment of
B. Paralytic ileus anticholinergic toxicity. It is useful to treat the central nervous
C. Post-operative retention of urine system effects of anticholinergic toxicity due to its ability to
D. Coronary insufficiency cross the blood-brain-barrier.

26. Contraindication/s for the use of adrenaline:


A. Hyperthyroidism
B. Hypertension
C. Hyperthyroidism, hypertension, and coronary
artery disease
D. Coronary artery disease
E. Hypertension and coronary artery disease

27. The new drug is studied in patients with the target Phase 0
disease to determine its efficacy and the doses to
Phase 0 trials are the first clinical trials done among people.
be used in any follow-on trials:
They aim to learn how a drug is processed in the body and how
A. Phase 3 Clinical Trials it affects the body. In these trials, a very small dose of a drug
B. Phase 4 Clinical Trials is given to about 10 to 15 people.
C. Phase 2 Clinical Trials
D. Phase 1 Clinical Trials Phase I
Phase I trials aim to find the best dose of a new drug with the
fewest side effects. The drug will be tested in a small group of
15 to 30 patients.

Phase II
Phase II trials further assess safety as well as if a drug works.
The drug is often tested among patients with a specific type of

“Courage is found in unlikely places.”


― J.R.R. Tolkien 65
cancer. Phase II trials are done in larger groups of patients
compared to Phase I trials.

Phase III
Phase III trials compare a new drug to the standard-of-care
drug. These trials assess the side effects of each drug and
which drug works better. Phase III trials enroll 100 or more
patients.

28. Enhance mucosal defenses: sucralfate is protective against acid injury in rabbit esophagus
A. Magnesium hydroxide by enhancing mucosal defenses through binding of pepsin and
B. Cimetidine bile salts, neutralization of hydrogen ions by its content of
C. Sucralfate aluminum hydroxide, and reduction in the permeability of
D. Clarithromycin esophageal mucosa

29. Drugs that mimic the action of acetylcholine: Drugs that activate muscarinic receptors in the peripheral
A. Parasympathomimetic drugs nervous system are called parasympathomimetic drugs
B. Sympatholytic drugs because they mimic the effects of acetylcholine on the
C. Parasympatholytic drugs parasympathetic nervous system. An example of a
D. Sympathomimetic drugs parasympathomimetic drug is pilocarpine, which is a
nonspecific muscarinic agonist.

30. Alpha 1 agonist: Phenylephrine is a selective α1-adrenergic receptor agonist


A. Dobutamine that increases blood pressure mainly by increasing systemic
B. Phenylephrine vascular resistance, without an associated increase in
C. Propranolol myocardial contractility.
D. Prazosin

31. The action of the drug on the body: pharmacogenetic- it is the science of understanding how
A. Pharmacogenetic genetic variability influences drug treatment outcomes
B. Pharmacodynamic
C. Pharmacokinetic pharmacokinetics- actions of body on the drug
D. Pharmacogenomic
pharmacogenomics-study on the role of genome in drug
response

32. Non-pharmacological approaches to therapy of 1Non-Pharmacologic Management : Lifestyle Management for

hypertension: hypertension
A. Low sodium chloride diet, weight reduction,
and cessation of smoking Weight reduction Attain & maintain BMI < 25
B. Low sodium chloride diet and weight kg/m2
reduction
C. Cessation of smoking Salt and potassium in diet -For adults, reducation of
D. Low sodium chloride diet Na+ intake by 1 g/ day
E. Weight reduction lowers SBP by 3-4 mmHg
(aim to consume no more
than 2.4 g/day of Na+)
-Increased K+ intake my
lower BP

“Courage is found in unlikely places.”


― J.R.R. Tolkien 66
Adapt Dietary Approaches -Diet rich in fruits,
to Stop Hypertension vegetables, low fat dairy
(DASH) type dietary plan products, whole grains,
poultry , fish , nuts
Diet low in sweet , red meat
& saturated / total fat

Moderation of Alcohol For those who drink alcohol


consumption ,consume:
</=2 standard drinks/ day
in men
</=1standard drink/day in
women

Physical activity Regular aerobic activity ( 3-


4 session a week lasting 40
mins per session)
Aerobic physical activity
reduces SBP by up to 5
mmHg

33. The relation of the individual’s genetic make-up to medical pharmacology-it is the science that study how
his/her response to specific drugs: medicines drugs work in health and disease and how they are
A. Medical Pharmacology processed by the bodies
B. Pharmacology
C. Toxicology pharmacology- it is a branch of medicine and pharmaceutical
sciences which is concerned with the study of drug medication
D. Pharmacogenomics
actions

toxicology- it is the study of adverse effects that occur in living


organisms due to chemicals

34. Side effect/s of Methyldopa:


A. Sedation
B. Sedation, vertigo, and nausea
C. Nausea
D. Sedation and vertigo
E. Vertigo

35. Lower blood pressure by depleting the body sodium


and reducing blood volume:
A. Diuretic
B. Calcium channel blocker
C. Beta blocker
D. ACE inhibitor

“Courage is found in unlikely places.”


― J.R.R. Tolkien 67
36. A boy is being worked up for severe acute
rhinosinusitis. Culture from material obtained from
maxillary sinus grew aerobic, oxidase-positive and
gram-negative diplococci.

How should this infection be treated?


A. Cephalexin
B. Vancomycin
C. Ampicillin
D. Amoxicillin-clavulanate

37. Which of the following bacterial agents has the The infectious dose for different serovars of Salmonella
lowest infective dose for producing gastrointestinal and strains of E.coli was quite large ( > 10 ^5 organisms),
disease in the human host? while the infectious dose for some Shigella spp. Seemed
A. Enterotoxigenic Escherichia coli to be as low as less than 10 organisms.
B. Shigella flexneri
C. Shigella flexneri
D. Enteropathogenic Escherichia coli

38. Shingles, a disease caused by varicella, belongs to Varicella-zoster virus (VZV) is an enveloped double-stranded
which member of the viral families? DNA virus belonging to the family Herpesviridae.VZV causes
A. Adenovirus chickenpox as a primary infection and shingles
B. Paramyxovirus
C. Herpesvirus
D. Poxvirus

39. A 8-year-old boy developed bloody diarrhea after E. coli 0157: H7 or STC or Shiga toxin producing E.coli was first
eating a poorly cooked hamburger. A few days later, recognized as a cause of illness in 1982 during an outbreak of
he was hospitalized with oliguria and hypertension. severe bloody diarrhea; the outbreak was traced to
With which organism is he most likely infected? contaminated hamburgers. Since then, most infections have
A. Enteroinvasive E. coli come from eating undercooked ground beef.
B. Salmonella typhi
C. Shiga toxin-producing E. coli
D. Clostridium perfringens

40. Thayer-Martin and Martin-Lewis media are used to The most widely used N. gonorrhoeae culture media (
isolate and identify: modified Thayer Martin Medium and Martin-Lewis use
A. Mycobacteria commercial peptone, hemoglobin, Isovitalex or Supplement
B. Neisseria B.
C. Salmonella
D. E. coli

41. Which one of the following represents the Azole antifungals consist of two primary classes: imidazoles
mechanism of action of the anti fugal azoles? and triazoles. Both classes are fungistatic agents and share
A. Inhibition of the activity of the enzyme, chitin similar mechanisms of action. The azoles interfere with the
synthase synthesis and permeability of fungal cell membranes by
B. Inhibition of GTP hydrolysis carried out by inhibiting cytochrome P450-dependent 14-alpha-sterol
elongation factor-2 (EF-2) demethylase

“Courage is found in unlikely places.”


― J.R.R. Tolkien 68
C. Inhibition of the activity of the enzyme, 14-α-
demethylase
D. Inhibition of the activity of the enzyme, β-
glucan synthase

42. Which of the following is a leading cause of Pneumonia in infants aged three weeks to three months is
pneumonia primarily in infants? most often bacterial; Streptococcus pneumoniae is the most
A. Rhinovirus common pathogen. In infants older than four months and in
B. Respiratory syncytial virus preschool-aged children, viruses are the most frequent cause
C. Mumps of CAP; respiratory syncytial virus (RSV) is the most common
D. Cytomegalovirus

43. Which of the following causes a fatal encephalitis


for which a vaccine is available? Encephalitis, or inflammation of the brain, is a treatable but
A. Rhinovirus potentially-serious medical condition that can result from
B. Rabies
immune system disorders. This includes disorders triggered
C. Cytomegalovirus
D. Mumps by the MMR vaccine. According to the CDC:

“Measles inclusion body encephalitis, or severe brain


swelling caused by the measles virus, is a complication of
getting infected with the wild-type measles virus. While rare,
this disorder almost always happens in patients with
weakened immune systems. The illness usually develops
within 1 year after initial measles infection and has a high
death rate.”

44. How does Pneumonic plague infection Pneumonic plague occurs when Y. pestis infects the lungs.
transmitted? This type of plague can spread from person to person through
A. Ingestion of contaminated water the air. Transmission can take place if someone breathes in
B. Inhalation of aerosolized bacteria aerosolized bacteria, which could happen in a bioterrorist
C. Direct skin contact with bubos attack
D. Bite of infected rat

45. A child develops impetigo on the right thigh. He was Penicillins and cephalosporins are the major antibiotics that
given an antibiotic that will inhibit cell wall inhibit bacterial cell wall synthesis.
synthesis. Which is the most likely antibiotic this
child would have received?
A. Erythromycin
B. Cephalexin
C. Ciprofloxacin
D. Ampicillin

46. Traditional vaccination for the common cold is Rhinovirus has more than 100 serotypes; it is unlikely that a
virtually impossible because there are multiple unifying vaccine will be developed. However, the use of
serotypes of which one of the following viruses? antivirals as chemoprophylaxis may have practical value.
A. Respiratory syncytial virus Topical application of interferon in the nose has been shown
to be effective in reducing the incidence of colds in people

“Courage is found in unlikely places.”


― J.R.R. Tolkien 69
B. Mumps who are exposed to others with a fresh cold.This strategy
C. Cytomegalovirus reduced the overall risk of cold by 40% and almost eliminated
D. Rhinovirus proven rhinovirus colds in contacts.

47. What is a mass of fungal filaments called? Mycelium, plural mycelia, the mass of branched, tubular
A. Mycelium filaments (hyphae) of fungi. The mycelium makes up the
B. Hyphae thallus, or undifferentiated body, of a typical fungus.
C. Yeast
D. Septum Hyphae each of the branching filaments that make up the
mycelium of a fungus.

Yeasts are eukaryotic, single-celled microorganisms classified


as members of the fungus kingdom.

48. The presence of Negri inclusion bodies in host cells Infectious Mononucleosis inclusion body:Owl’s eye
is characteristic of Rabies inclusion body:Negri bodies
A. Infectious mononucleosis
B. Rabies
C. Mumps
D. Aseptic meningitis

49. Classifies as a calicivirus this hepatitis and is acquire Fecal oral ( TAE) -Hepatitis A and E
via the fecal-oral route. Common reservoir are pigs
and humans
A. Hepatitis E
B. Hepatitis D
C. Hepatitis C
D. Hepatitis B

“Courage is found in unlikely places.”


― J.R.R. Tolkien 70
50. Orchitis, which may cause sterility, is possible Mumps-orchitis
manifestation of which of the following? Rabies-rabies
A. Mumps CMV-Infectious Mononucleosis
B. Rhinovirus Rhinovirus-Common colds
C. Rabies
D. Cytomegalovirus

AVENGERS:
Part 1 (1-25)
Questions Ratio

1. While working up a patient complaining of frequent bruising, you Specialized tests will include screening coagulation
included this test to evaluate the activity of Factor VII. tests that measure how long it takes the blood to
clot, specifically two tests known as activated partial
A. Bleeding Time thromboplastin time (aPTT) and prothrombin time
B. Thrombin Time (PT). Individuals with factor VII deficiency have a
C. Prothrombin Time normal aPTT and a prolonged PT.
D. Activated Partial Thromboplastin Time

2. A patient with a hypercoagulable state is given this anticoagulant which When given by parenteral injection, the heparin
potentiates the effect of antithrombin III. analogue had almost the same potentiating effect
on antithrombin III as mucous heparin
A. Thrombomodulin
B. Protein C
C. Heparin
D. Antithromboplastin

“Courage is found in unlikely places.”


― J.R.R. Tolkien 71
3. Student X has normal defecation which is mostly due to the activity of Sympathetic supplies rectum to the rectum is from
the: lumbar splanchnic nerves and superior and inferior
hypogastric plexuses.
A. Pelvic Splanchnic Nerves
B. Vagus Nerves Parasympathetic supplies are S4 via the PELVIC
C. Sympathetic nerves SPLANCHNIC NERVES and inferior hypogastric
D. Prevertebral Ganglia plexuses.

4. A patient with bilious vomiting showed this compensatory mechanism: Pancreatic and biliary drainage most often occurs
after surgery, and usually the volume is low, so,
A. Decreased rate and depth of breathing despite loss of a HCO3−-rich fluid, significant
B. Increased secretion of metabolic acidosis does not occur. In the rare setting
C. Increased activity of beta intercalated cells of collecting ducts in which drainage volume exceeds 1 to 2 L/d,
D. Increased generation of hydrogen ions from food however, metabolic acidosis will develop and be
maintained by concomitant volume depletion

5. A patient with which condition will not exhibit an increase in hematocrit? Factors that may cause a rise in Hct value such as:
6. Polycythemia Vera
A. High altitude dwelling 7. Heart or kidney problems
B. Dehydration 8. Intake of anabolic steroids
C. Pregnancy 9. Dehydration
D. Polycythemia Vera 10. Diarrhea
11. Lung problems
12. Burns
13. Smoking
14. High altitude

15. Checking a patient’s arterial blood gas, you take note that this factor
increases the affinity of hemoglobin to oxygen

A. Increase in pCO2
B. Increase in pH
C. Increase in 2,3 OPC
D. Increase in temperature

“Courage is found in unlikely places.”


― J.R.R. Tolkien 72
16. Seeing Korean Food on your Grab App made your mouth watery and is
due to the activation of which preganglionic neuron?

A. Submandibular ganglion
B. Inferior salivatory nucleus
C. Otic ganglion
D. Superior salivatory nucleus

17. A newly wed male argues that the newborn baby with the blood type O
is not his. The father is a type B homozygous while the mother is a type
AB, what is the probability that this newborn will have that blood type?

A. 75%
B. 50%
C. 0%
D. 25%

18. Your asthmatic patient complained that everytime he uses his reliever Relievers:
medication he experiences a feeling of incomplete voiding. What kind of SABAs
drug is his reliever medication? Anticholinergics

A. Sympathomimetic and Parasympatholytic


B. Parasympatholytic
C. Parasympathomimetic
D. Sympathomimetic

19. A patient complains palpitations, while examining him you place the
diaphragm of the stethoscope at which best location if you want to hear
the sounds produced by closing the mitral valve?

A. 4 iC5, LMCL
B. 4 iC5, LPSM
C. 5 iC5 LMCL
D. 5 iC5, LPSM

“Courage is found in unlikely places.”


― J.R.R. Tolkien 73
20. For a left-handed person, an ischemic event to this area presents a
problem in the regulation of speaking: left inferior frontal gyrus houses the broca’s area
broca’s area is responsible in expressing spoken and
A. right superior temporal gyrus written language
B. right inferior frontal gyrus
C. left superior temporal gyrus
D. left inferior frontal gyrus

21. A patient complained with a high grade fever for 5 days with a history of
dengue 2 years ago. The doctor assured that his current condition is not
dengue due to what dengue blot findings?

A. IgG positive, IgM negative


B. IgG positive, IgM positive
C. IgG negative, IgM positive
D. IgG negative, IgM negative

22. While playing basketball, a blunt injury to the neck occurred after which
a tingling sensation in the clavicular area was felt, the most probable level
of injury is at:

A. C3
B. C5
C. C1
D. C7

23. After eating a heavy breakfast, the distention of the stomach will: The gastrocolic reflex is a physiological reflex that
controls the motility of the lower gastrointestinal
A. Trigger gastrocolic reflex tract following a meal. As a result of the gastrocolic
B. Decrease salivary secretion reflex, the colon has increased motility in response
C. Cause glucose absorption to the stretch of the stomach with the ingestion of
D. Inhibit gastric emptying food.

24. Why would tetany not happen in cardiac muscles? The refractory period of cardiac muscle is
dramatically longer than that of skeletal muscle. This
A. They have a long hyperpolarized period prevents tetanus from occurring and ensures that
B. They have a long absolute refractory period each contraction is followed by enough time to allow
C. They have a the heart chamber to refill with blood before the
D. They have more mitochondria than skeletal muscles next contraction.

“Courage is found in unlikely places.”


― J.R.R. Tolkien 74
25. Among water soluble hormones, which of the following is NOT a 2nd
messenger?

A. Acetylcholine
B. Cyclic GMP
C. Calcium
D. Cyclic AMP

26. This muscle reflex metatarsophalangeal joint of the big toe and also Flexor hallucis longus - flexion of all the joints of the
support the medial longitudinal arch great toe (hallux). When the foot is off the ground,
this muscle, together with flexor digitorum longus,
A. Flexor hallucis longus flexes the toes at the distal phalanges.
B. Lumbricals (no)
C. Flexor hallucis brevis The lumbrical muscles, with the help of the
D. Adductor hallucis oblique head interosseous muscles, simultaneously flex the
metacarpophalangeal joints while extending both
interphalangeal joints of the digit on which it inserts.
The lumbricals are used during an upstroke in
writing.

The primary function of the flexor hallucis brevis is


flexion of the great toe at the metatarsophalangeal
joint. This muscle aids the flexor hallucis longus in
the toe-off phase of locomotion, increasing the final
push-off from the ground during activities such as
walking, running and jumping.

The Adductor hallucis (Adductor obliquus hallucis)


arises by two heads—oblique and transverse and is
responsible for adducting the big toe.

27. Which electrolyte can cause cardiac arrest in diastole when it is in excess? Extremely high levels of potassium in the blood
(severe hyperkalemia) can lead to cardiac arrest
A. Potassium and death.
B. Calcium
C. Sodium Hyperkalemia- Associated cardiac arrhythmias
D. Chloride include sinus bradycardia, sinus arrest, slow
idioventricular rhythms, ventricular tachycardia,
ventricular fibrillation and asystole
Hypernatremia- Primarily neurologic: change in
sensorium is the most common manifestation
Hyperchloremia- Effects on the cardiovascular
system include direct impairment of myocardial
contraction (especially at a pH < 7.2), tachycardia,
and increased risk of ventricular fibrillation or heart
failure with pulmonary edema. Patients may report
dyspnea upon exertion or, in severe cases, at rest.
Hypercalcemia- Bradycardia, AV block, arrhythmias
and shortened QT-interval

“Courage is found in unlikely places.”


― J.R.R. Tolkien 75
28. The penile urethra in on what specific structure?

A. Corpus cavernosum
B. Corpus spinosum
C. Corpus spongiosum
D. Corpus striatum

The pendulous urethra is invested in the corpus


spongiosum of the penis in the pendulous portion of
the penis. The urethra is located concentrically
within the corpus spongiosum. In the distal urethra
lies the fossa navicularis, a small dilation of the
urethra just proximal to the urethral meatus.

A scaphoid (navicular) fracture is a break in one of


the small bones of the wrist. This type of fracture
29. Most fractured carpal bone to suspect if one will fall from an outstretched occurs most often after a fall onto an outstretched
hand. hand. Symptoms of a scaphoid fracture typically
include pain and tenderness in the area just below
A. Hamate the base of the thumb.
B. Scaphoid
C. Capitate
D. Pisiform

30. For a left-handed person, an ischemic event to this area presents a


problem in the regulation of speaking: left inferior frontal gyrus houses the broca’s area
broca’s area is responsible in expressing spoken and
E. right superior temporal gyrus written language
F. right inferior frontal gyrus
G. left superior temporal gyrus
H. left inferior frontal gyrus

31. Under normal circumstances, which neurotransmitter provides inhibition Under normal circumstances, glycine provides
of muscle tone that balances the excitation of muscle tone provided by inhibition of muscle tone that balances the excitation
other neurotransmitters? of muscle tone provided by other neurotransmitters

A. Glutamine
B. Glucagon
C. Glycine
D. Glutamate

32. Which of the following does not have muscarinic receptors? Muscarinic receptors are divided into five main
subtypes M1, M2, M3, M4, and M5.
A. Constrictor Pupillae Muscle
B. Arterial Muscle (????) While each of the subtypes exists within the central
C. Heart nervous system, they are encoded by separate genes
D. Sweat Glands and localized to different tissue types.

“Courage is found in unlikely places.”


― J.R.R. Tolkien 76
The M1 receptor is primarily found in the cerebral
cortex, gastric, and salivary glands.

M2 receptors are diffusely located in smooth muscle


and cardiac tissue.

M3 receptors are also present on smooth muscle,


gastric, and salivary glands.

M4 and M5 receptors are not as well characterized


but appear within the hippocampus and substantia
nigra.

The wide distribution of receptors functions to


mediate the parasympathetic division of the
autonomic nervous system, maintaining internal
homeostasis.

Muscarinic receptors are G-coupled protein


receptors involved in the parasympathetic nervous
system. The only exception to these receptors is the
sweat glands, which possess muscarinic receptors
but are part of the sympathetic nervous system.

33. Which of the following will lead to an increased stroke volume?

A. A highly tachycardic heart


B. An increase in aortic pressure
C. A more negative intrathoracic pressure
D. A decrease in venous return

34. Under basal condition, majority of the caloric needs of human heart carbohydrates make up 45 to 65 percent of your
comes from: total daily calories

A. Ketones
B. Fats
C. Carbohydrates
D. Amino Acids

Part 2 (26-50)
Questions Ratio

“Courage is found in unlikely places.”


― J.R.R. Tolkien 77
35. How is the DNA Template read during transcription? The antisense strand of DNA is read by RNA polymerase
from the 3' end to the 5' end during transcription (3' →
A. From ‘3 to ‘5 end 5'). The complementary RNA is created in the opposite
B. From C-terminal to N-terminal direction, in the 5' → 3' direction, matching the sequence
C. From 5’ to 3’ end of the sense strand with the exception of switching uracil
D. From N-Terminal to C-Terminal for thymine

36. Which hormone is produced using two mitochondrial enzymes?

A. Estradiol
B. Aldosterone
C. Testosterone
D. Cortisol

37. Which among the coenzyme of the pyruvate dehydrogenase Thiamin pyrophosphate (TPP) is a cofactor for a number
transketolase, a-ketoglutarate dehydrogenase: of enzymes, such as transketolase, pyruvate
dehydrogenase, and α-ketoglutarate dehydrogenase.
A. Vit B7
B. Vit B5
C. Vit B3
D. Vit B1

38. In fibrinolysis, plasminogen is activated to plasmin and inhibits


what factor?

A. Factor X
B. Factor XI
C. Factor XIII
D. Factor XI

39. There could be a premature termination of protein synthesis There are 3 STOP codons in the genetic code - UAG, UAA,
when this codon is encountered by ribosome and UGA

A. CUG
B. GNU
C. UCC
D. UAG

40. If one mole of Fructose-1.6-biphosphate undergoes complete 38 ATP


oxidation using the malate shuttle, how many moles of ATP will
Since the malate-aspartate shuttle regenerates NADH
be produced (NET|?
inside the mitochondrial matrix, it is capable of
A. 38 maximizing the number of ATPs produced in glycolysis
(3/NADH), ultimately resulting in a net gain of 38 ATP
B. 28
molecules per molecule of glucose metabolized.
C. 22
D. 34

“Courage is found in unlikely places.”


― J.R.R. Tolkien 78
41. RNA Polymerase is involved in all the following steps in Transcription involves four steps:
transcription except:

A. Initiation Initiation. The DNA molecule unwinds and separates to


B. post-transcriptional modification
form a small open complex. RNA polymerase binds to the
C. elongation
D. recognition of promoter site promoter of the template strand.

Elongation. RNA polymerase moves along the template


strand, synthesising an mRNA molecule. In prokaryotes
RNA polymerase is a holoenzyme consisting of a number
of subunits, including a sigma factor (transcription factor)
that recognises the promoter. In eukaryotes there are
three RNA polymerases: I, II and III. The process includes
a proofreading mechanism.

Termination. In prokaryotes there are two ways in which


transcription is terminated. In Rho-dependent
termination, a protein factor called "Rho" is responsible
for disrupting the complex involving the template strand,
RNA polymerase and RNA molecule. In Rho-independent
termination, a loop forms at the end of the RNA molecule,
causing it to detach itself. Termination in eukaryotes is
more complicated, involving the addition of additional
adenine nucleotides at the 3' of the RNA transcript (a
process referred to as polyadenylation).

Processing. After transcription the RNA molecule is


processed in a number of ways: introns are removed and
the exons are spliced together to form a mature mRNA
molecule consisting of a single protein-coding sequence.
RNA synthesis involves the normal base pairing rules, but
the base thymine is replaced with the base uracil.

“Courage is found in unlikely places.”


― J.R.R. Tolkien 79
42. What is the complication of excessive ketogenesis? level of ketones in the blood reaches an excess, such that
the buffering capacity of the plasma is exhausted due to
A. Metabolic alkalosis the depletion of bicarbonates, it can lead to metabolic
B. Metabolic acidosis acidosis with a drop in blood pH, a condition known as
C. Respiratory alkalosis ketoacidosis that can occur in many metabolically
D. Respiratory acidosis deranged situations

43. The binding of epinephrine to which receptor inhibits adenylate The beta-adrenergic receptor is coupled to adenylyl
cyclase from producing cAMP? cyclase via Gs and binding of epinephrine to this receptor
leads to increased cyclic AMP synthesis. Also, when
A. a1 adrenergic epinephrine binds to alpha-2 adrenergic receptors,
B. Vasopressin-V2 adenylyl cyclase activity is inhibited, because that
C. Vasopressin-V1 receptor is coupled to via Gi, an inhibitory G protein.
D. a2 adrenergic

44. What is the major end-product of beta oxidation of fatty acids? Beta-oxidation is the process by which long chain fatty
acyl CoA is degraded. The products of beta-oxidation are:
A. Propionyl CoA acetyl CoA. FADH2, NADH and H.
B. Acetyl CoA
C. Butyryl CoA
D. Acetoacetyl CoA

45. Which of the following hormones is not polypeptide?

A. FSH List of peptide hormones in humans


B. Aldosterone
adrenocorticotropic hormone (ACTH)
C. ACTH
D. Hcg amylin
angiotensin
atrial natriuretic peptide (ANP)
calcitonin
cholecystokinin (CCK)
gastrin
ghrelin
glucagon
growth hormone
follicle-stimulating hormone (FSH)
insulin
leptin
melanocyte-stimulating hormone (MSH)
oxytocin
parathyroid hormone (PTH)
prolactin
renin

“Courage is found in unlikely places.”


― J.R.R. Tolkien 80
somatostatin
thyroid-stimulating hormone (TSH)
thyrotropin-releasing hormone (TRH)
vasopressin
vasoactive intestinal peptide

46. Which structural feature is found in a single stranded DNA The covalent structure of RNA differs from that of DNA in
molecule? two respects. The sugar units in RNA are riboses rather
than deoxyriboses. Ribose contains a 2′-hydroxyl group
A. Each purine and pyrimidine pairs with its not present in deoxyribose.
complementary base
B. With sugar and phosphate as backbone and nitrogen
bases in the center which holds the strand together
C. Each 3’-5’ phosphodiester bond contains one
phosphate group linking 2 deoxyribose sugars
D. It can have one end with a 5’-phosphate group and the
other end has a 2’-hydroxyl group

47. If G-6-P-D is deficient:

A. Hydrogen peroxide is detoxified to water


B. Glutathione is oxidized
C. Glutathione remains in its reduced form
D. RBC Membrane is intact

48. The “B” Gene in ABO blood group codes for what enzyme? The ABO gene is located on chromosome 9 and has three
alleles, A, B and O. The A allele encodes a
A. GelNA1 transferase glycosyltransferase that adds N-acetylgalactosamine to
B. Fucosyltransferase the glycoprotein H antigen that is expressed on all normal
C. G-6-RD red cells. The B allele encodes a different
D. Gal Transferase glycosyltransferase that adds d-galactose

“Courage is found in unlikely places.”


― J.R.R. Tolkien 81
49. The committed step in extramitochondrial lipogenesis involves The first step in fatty acid synthesis is to activate acetyl
the formation of: CoA by the addition of a carbon dioxide using acetyl Coa
carboxylase. This reaction and is the flux of first
A. acetyl CoA committed step of FA biosynthesis. Acetyl CoA
B. malonyl CoA carboxylase is responsible for producing Malonyl CoA.
C. propionyl CoA
D. succinyl CoA

50. What happens if an amino acid undergoes catabolism? Aspartic acid is converted to oxaloacetate thru
TRANSAMINATION. Transamination reactions combine
A. Decarboxylation converts glutamic acid to GABA reversible amination and deamination, and they mediate
B. Serine is converted to redistribution of amino groups among amino acids. It is a
C. All choices biologically important process by which living cells
D. Aspartic Acid is converted to oxaloacetate thru non- reversibly transfer the amino group from an amine (e.g.,
oxidative deamination γ-aminobutyrate) or α-amino acid (e.g., aspartate) to an
α-keto carboxylic acid (e.g., α-ketoglutarate).
Serine is converted to pyruvate thru DEHYDRATION
deamination. (serine dehydratase)These enzymes are
called dehydratases because dehydration precedes
deamination. Serine loses a hydrogen ion from its α-
carbon atom and a hydroxide ion group from its β-carbon
atom to yield aminoacrylate.

51. The cellular elements of blood within the circulation is suspended Plasma. The watery fluid portion of blood (90 percent
in its fluid portion called: water) in which the corpuscular elements are suspended.
It transports nutrients as well as wastes throughout the
A. Serum body.
B. Plasma
C. Conguiant
D. Supernatant

52. Insulin lowers the blood glucose level by all the following
mechanism EXCEPT:

A. promoting the mobilization of glucose transporters


B. promoting glycogenesis
C. increasing lipogenesis
D. increasing the synthesis of proteins

“Courage is found in unlikely places.”


― J.R.R. Tolkien 82
53. A deep laceration at the anterior axillary will traumatize the The anterior axillary fold is formed by the lateral edge of
structure the pectoralis major muscle;

A. Subscapularis muscle
B. coracobrachialis
C. Pectoralis major

54. Erection of the penis is a response to the activity of the


The parasympathetic nervous system is primarily
A. ilioinguinal nerves involved in sustaining and maintaining an erection, which
B. perineal branch of S4 is derived from S2-S4 nerve roots. Sexual stimulation
C. parasympathetic nerves causes the release of neurotransmitters from cavernosal
D. sympathetic nerves nerve endings and relaxation factors from endothelial
cells lining the sinusoids.

55. The muscles of the anterior thigh are mainly innervated by this femoral nerve
nerve
The muscles in the anterior compartment of the thigh are
A. Obturator innervated by the femoral nerve (L2-L4), and as a general
B. Internal Iliac rule, act to extend the leg at the knee joint. There are
three major muscles in the anterior thigh – the pectineus,
C. Femoral
sartorius and quadriceps femoris.
D. Sciatic

56. Ejaculation is the response to the activity of the Ejaculation is a process involving
A. perineal branch of S4 sympathetic and parasympathetic
B. parasympathetic nerves effects during different stages –
C. sympathetic nerves emission and ejection. Some conditions
D. obturator nerves of ejaculation dysfunction are
associated with autonomic nerves.
However, the exact effects of
autonomic nerves on ejaculation are
not well defined
57. The uterus receives its blood supply from what artery The ovarian artery supplies oxygenated blood to the
ovary, fallopian tube, and uterus. In the myometrium, the
A. Uterine and ovarian artery uterine artery further branches out to the arcuate artery,
B. superior vesical artery radial artery, spiral artery, and basal artery. The
C. middle rectal artery descending branch supplies blood to uterine cervix and
D. uterine artery vagina.

58. The artery that supplies the gallbladder usually arises from the The arterial supply to the gallbladder is via the cystic
artery – a branch of the right hepatic artery (which itself
A. Superior mesenteric artery is derived from the common hepatic artery, one of the
B. Gastroduodenal artery three major branches of the coeliac trunk).
C. Cardiac Artery

“Courage is found in unlikely places.”


― J.R.R. Tolkien 83
D. Right hepatic artery

59. Cancer of the Prostate can metastasize to the skull via the The predilection of prostate cancer to the skull may be
due to Batson's venous plexus that connects prostate
A. vertebral venous plexus with the skull through epidural and dural veins.
B. pampiniform plexus Therefore, there should be deliberate attempt at
C. portal vein assessing possibility of skull metastasis in all patients with
D. inferior vena cava advanced prostate cancer.

Part 3 (51-75)
Questions Ratio

60. Which is the characteristic feature of Tay Sach’s disease and The accumulation of lipid in retinal ganglion cells that leads to a
other metabolic disease affecting the neurons chalk-white appearance of the fundus called 'cherry red spot' is
the hallmark of Tay-Sachs disease. It is also seen in others
A. None of the choices neurometabolic diseases as well as in central retinal artery
B. Onion peel appearance in the neurons occlusion.
C. The waxy White appearance or the bundle

D. A cherry red used in the macula

61. Coarctation of the aorta is common in which syndromes Turner’s- Heart abnormalities are common in individuals with Turn
People should be
A. Tumer’s monitored for problems with their Aorta and high blood
B. Klinefelter’s pressure.
C. Dump
D. Noonan’s Coarctation of the aorta- Aortic narrowing near insertion of
ductus arteriousus (juxtaductal). Associated with bicuspid aortic
valve, other heart defects, and Turner syndrome.

USMLE Step 1 2018, Cardio-Pathology, Page 294, Section III.

62. Metastatic calcification Metastatic calcification is caused by an increased calcium


phosphate product in the blood, and may result from
A. Can occur with normal serum levels of calcium hypercalcemia or hyperphosphatemia, or both. It is commonly
B. Commonly develops in aging of damaged heart associated with hyperparathyroidism, sarcoidosis, metastatic
valves disease, and myeloma
C. Is nosocomial in areas of mass
D. Can be associated with metastatic malignancy

63. Which of the following cell type of the innate immune Several types of cells of the immune system perform
system does not perform phagocytosis phagocytosis, such as neutrophils, macrophages, dendritic cells,
and B lymphocytes.
A. Basophils
B. Eosinophils The professional phagocytes are the monocytes, macrophages,
C. Macrophages neutrophils, tissue dendritic cells, and mast cells.
D. Neutrophils

“Courage is found in unlikely places.”


― J.R.R. Tolkien 84
64. Review of a series of surgical pathology resorts indicates Stage I. This stage is usually a small cancer or tumor that has not
that a certain type of neoplasm is diagnosed as a grade I on grown deeply into nearby tissues. It also has not spread to the
the scale of IV. Clinically, some of the patients with this lymph nodes or other parts of the body. It is often called early-
neoplasm are found to have stage I disease. WHich of the stage cancer.
following is the best interpretation of a neoplasm with these
designations?

A. May spread via lymphatic and bloodstream


B. Well-differentiated and localized
C. Unlikely to be malignant
D. Aching from epithelium

65. Angelman syndrome is due to: Angelman syndrome is a genetic disorder. It's usually caused by
problems with a gene located on chromosome 15 called the
A. Inversion ubiquitin protein ligase E3A (UBE3A) gene.
Angelman syndrome results when a person inherits two copies of
B. Dynamic inheritance chromosome 15 from his or her father (paternal copies) instead
C. Uniparental disomy of one copy from each parent. This phenomenon is called
paternal uniparental disomy .
D. Mitochondrial disorder Most cases of Angelman syndrome are not inherited, particularly
those caused by a deletion in the maternal chromosome 15 or
by paternal uniparental disomy . These genetic changes occur as
random events during the formation of reproductive cells (eggs
and sperm) or in early embryonic development.

66. Transudation may occur due to the following except: http://www.edcentral.com.au/education/primary-exam-


resources/pathologynewmcqs/1001-013-pathology-mcq-acem-
A. Decreased intravascular oncotic pressure primary-fluid-and-haemodynamic-derang-1/file
B. Inflammation
C. Increased intravascular hydrostatic pressure
D. Lymphatic Obstruction

“Courage is found in unlikely places.”


― J.R.R. Tolkien 85
67. A 49-year-old woman has a routine of physical examination.
A 4 cm diameter non-tender mass is palpated in her right
breast. The mass appears fixed to the chest wall. Another
2 cm non-tender is palpable in the left axilla. A chest
radiograph reveals multiple 0.5 to 2 cm nodules in both
lungs. Which of the following TNM classifications best
indicates the stage of her disease?

A. T2 N1 M0
B. T3 N0 M0
C. T1 N0 M1
D. T4 N1 M1

68. Regarding a wound created by a surgical incision: Macrophages begin as circulating monocytes that are attracted
to the wound site beginning about 24 hours after injury
A. The initial leucocytic infiltrate has largely cleared
by week 2
B. Epithelial spurs fuse to close the wound at around
day 5

C. Macrophages appear at the wound within 24


hours

D. Granulation tissue formation occurs as Pro first


step in wound healing

69. Which of the following regarding arterial thrombi is TRUE? mural thrombi most commonly located in descending aorta
Break off and embolize or result in total occlusion of VEIN not
A. It typically creates a long cast of vessel lumen with artery
a propagational long tail
B. Most common sights are cerebral, coronary is
temporal arteries
C. The tail of thrombi
D. Mural thrombi is most commonly found in the
femoral artery

“Courage is found in unlikely places.”


― J.R.R. Tolkien 86
70. Chronic inflammation is characterized by all of the below
EXCEPT:

A. Tissue destruction
B. Infiltration with neutrophils
C. Angiogenesis
D. Fibrosis

Flocculent densities in the matrix of mitochondria have become


quite important in cell pathology since, when prominent, they
71. Which of the following changes is associated with indicate irreversible cell injury.
irreversible cell damage?

A. Cellular swelling
B. Glycogen depletion
C. Loss of microvilli
D. Flocculent densities in mitochondria

72. Histamine is involved in acute inflammatory responses and The richest sources of histamine are the mast cells that are
is released from mast cells, Which of the following normally present in the connective tissue adjacent to blood
statements is incorrect? vessels. It is also found in blood basophils and platelets.

A. It causes increased permeability of arterialies Histamine is stored in mast cell granules and is released by mast
B. It is found in blood basophils platelets and most cell degranulation in response to a variety of stimuli, including
cells (1) physical injury, such as trauma, cold, or heat, by unknown
C. It may be released from physical trauma mechanisms;
D. It causes constriction of large vessels (2) binding of antibodies to mast cells, which
underlies immediate hypersensitivity (allergic) reactions
(3) products of complement called anaphylatoxins (C3a and C5a).

Histamine causes dilation of arterioles and increases the


permeability of venules. Histamine is considered to be the
principal mediator of the immediate transient phase of increased
vascular permeability, producing interendothelial gaps in
venules, as discussed earlier. Its vasoactive effects are mediated
mainly via binding to receptors, called H1 receptors, on
microvascular endothelial cells.

Robbin’s 9th ed. Chapter 3 Acute Inflammation p 83

“Courage is found in unlikely places.”


― J.R.R. Tolkien 87
73. With regard to mechanisms of cellular adaptation, which of
the following statements is true?

A. The myocardium adapts to increased workload by


hypertrophy and hyperplasia

B. TNF always acts as a inhibitor of hyperplasia

C. Hyperplasia refers to an increase in cell size

D. Barett’s metaplasia is squamous to columnar in


the distal esophagus

74. Regarding mediators of inflammation: TNF regulates energy balance by promoting lipid and protein
mobilization and by suppressing appetite. Therefore, sustained
A. TNF contributes to cachexia of disease production of TNF contributes to cachexia, a pathologic state
B. The systemic acute phase response is induced by characterized by weight loss and anorexia that accompanies
MIF-1 and RANTES chemokines some chronic
infections and neoplastic diseases
C. TNF is a chemokines with chemoattractant
properties TNF and IL-1 serve critical roles in leukocyte recruitment by
promoting adhesion of leukocytes to endothelium and their
D. TNF and IL-1 are produced mainly by activated migration through vessels. These cytokines are produced mainly
hydrocytes by activated macrophages and dendritic cells; TNF is also
produced by T lymphocytes and mast cells, and IL-1 is produced
by some epithelial cells as well.

TNF is a cytokine not a chemokine. Chemokines =


chemoattractants

The C-C chemokines, which include


monocyte chemoattractant protein (MCP-1), eotaxin,
macrophage inflammatory protein­1α (MIP­1α), and RANTES
(regulated and normal T-cell expressed and secreted), generally
attract monocytes, eosinophils, basophils and lymphocytes, but
are not as potent chemoattractants for neutrophils.

Robbin’s 9th ed. Chapter 3 Acute Inflammation p 86

75. Which of the following statements regarding disseminated


Excess circulating thrombin results from the excess activation of
intravascular coagulation (DIC) is FALSE? the coagulation cascade. The excess thrombin cleaves fibrinogen,
which ultimately leaves behind multiple fibrin clots in the
A. Fibrinogen degradation products IFDPN increases due
circulation. These excess clots trap platelets to become larger
to capal fibrinolysis
clots, which leads to microvascular and macrovascular
B. Multiple thrombi develop in the microcirculation
thrombosis. This lodging of clots in the microcirculation, in the
C. Bleeding disease is due to thrombocytopenia large vessels, and in the organs is what leads to the ischemia,
D. Rapid consumption of platelets, prothrombin and impaired organ perfusion, and end-organ damage that occurs
firbrinogen occurs. with DIC.[citation needed]
Coagulation inhibitors are also consumed in this process.
Decreased inhibitor levels will permit more clotting so that a
positive feedback loop develops in which increased clotting leads
to more clotting. At the same time, thrombocytopenia occurs and
this has been attributed to the entrapment and consumption of
platelets. Clotting factors are consumed in the development of

“Courage is found in unlikely places.”


― J.R.R. Tolkien 88
multiple clots, which contributes to the bleeding seen with
DIC.[citation needed]
Simultaneously, excess circulating thrombin assists in the
conversion of plasminogen to plasmin, resulting in fibrinolysis.
The breakdown of clots results in an excess of FDPs, which have
powerful anticoagulant properties, contributing to hemorrhage.
The excess plasmin also activates the complement and kinin
systems. Activation of these systems leads to many of the clinical
symptoms that patients experiencing DIC exhibit, such as shock,
hypotension, and increased vascular permeability. The acute
form of DIC is considered an extreme expression of the
intravascular coagulation process with a complete breakdown of
the normal homeostatic boundaries. DIC is associated with a poor
prognosis and a high mortality rate

76. Regarding acute inflammation:


2 phase:
A. Increase permeability leads to protein depleted
plasma leaking into the tissue 1. Vascular Phase
B. Initial vasoconstriction is the result of histamine
and nitric oxide In the vascular phase, small blood vessels adjacent to the
C. Stasis injury dilate (vasodilatation) and blood flow to the area
D. Initial formation of increases. The endothelial cells initially swell, then contract
to increase the space between them, thereby increasing the
permeability of the vascular barrier. This process is
regulated by chemical mediators.

Exudation of fluid leads to a net loss of fluid from the


vascular space into the interstitial space, resulting in
oedema (tumour).

The formation of increased tissue fluid acts as a medium for


which inflammatory proteins (such as complement and
immunoglobulins) can migrate through. It may also help to
remove pathogens and cell debris in the area through
lymphatic drainage.

2. Cellular Phase

The predominant cell of acute inflammation is the


neutrophil. They are attracted to the site of injury by the
presence of chemotaxins, the mediators released into the
blood immediately after the insult.

The migration of neutrophils occurs in four stages:


● Margination – cells line up against the endothelium
● Rolling – close contact with and roll along the
endothelium
● Adhesion – connecting to the endothelial wall
● Emigration – cells move through the vessel wall to the
affected area

“Courage is found in unlikely places.”


― J.R.R. Tolkien 89
Once in the region, neutrophils recognise the foreign body
and begin phagocytosis, the process whereby the pathogen
is engulfed and contained with a phagosome. The
phagosome is then destroyed via oxygen-independent (e.g.
lysozymes) or oxygen-dependent (e.g. free radical
formation) mechanisms.

77. Irreversible cellular injury is characterized by all EXCEPT:

A. decreased activity NA+ / K+ ATPase causing


cellular swelling
B. the presence of reaction to specimen
C. progressive loss of phospholipids
D. damage to the cellular cytoskeleton (???)

78. A substance that is not known to accumulate intracellularly Initially ink is taken up by keratinocytes, and phagocytic cells
is: (including fibroblasts, macrophages and mast cells).

A. Hemosiderin
Tattooing is a form of localized, exogenous pigmentation of the
B. melanin skin.
C. pus
D. tattoo ink (???)

79. Common risk factors for arterial thrombo-emboli include Risk factors for thromboembolism, the major cause of arterial
the following EXCEPT: embolism, include disturbed blood flow (such as in atrial
fibrillation and mitral stenosis), injury or damage to an artery
A. Cardiomyopathy (???) wall, and hypercoagulability (such as increased platelet count).
B. Rheumatic heart disease
C. AMI
D. Paradoxical embolism from venous thrombi

“Courage is found in unlikely places.”


― J.R.R. Tolkien 90
80. Habitat of Giardia lamblia: Giardia duodenalis is a protozoan flagellate (Diplomonadida).
This protozoan was initially named Cercomonas intestinalis by
A. Mainly in the duodenum and jejunum Lambl in 1859. It was renamed Giardia lamblia by Stiles in 1915
B. Colon and rectum in honor of Professor A. Giard of Paris and Dr. F. Lambl of Prague.
C. Distal part of the large intestine However, many consider the name, Giardia duodenalis, (Davaine
D. Liver and biliary tree 1875) to be the correct taxonomic name for this protozoan.

81. A 15-year-old healthy girl with no major medical problems


notes blotchy areas of erythema that are pruritic over the
skin of her arms, legs, and trunk within an hour every time
she eats seafood, followed by diarrhea. These problems
abate within 3 hours, and then physical examination reveals
no abnormal findings. Which of the following immunologic
abnormalities is she most likely to have:

A. Localized anaphylaxis
B. Complement activation
C. Cell-mediated hypersensitivity
D. Hypergammaglobulinemia

“Courage is found in unlikely places.”


― J.R.R. Tolkien 91
82. In primary wound healing:

A. Wound strength is 25% of normal at the end of the


first week
B. Neovascularization is maximal at day five
C. Macrophage infiltration occurs at 24 hours
D. Type I collagen is replaced by Type II collagen

83. Type of specimen used in parasitological examination of CSF- Naegleriafowleri infection. can be diagnosed in the
Naegleria fowleri: laboratory by detecting
N. fowleri organism in CSF thru Lumbar puncture, biopsy or tissue
A. Cerebrospinal fluid specimens.
B. Liver aspirate
C. Urine www.cdc.gov/ diagnosis-hcp
D. Perianal Swab
CSF

USMLE Step 1 2018, Micro/para page 156, Section II

84. Definitive host for Toxoplasma gondii: The only known definitive hosts for Toxoplasma gondii are
members of family Felidae (domestic cats and their relatives).
A. Dog Unsporulated oocysts are shed in the cat's feces . Although
B. Cat oocysts are usually only shed for 1–3 weeks, large numbers may
C. Rat be shed.
D. Man

Part 4 (76-100)
Questions Ratio

“Courage is found in unlikely places.”


― J.R.R. Tolkien 92
85. Alpha 1 adrenoceptor action, EXCEPT:

A. Mydriasis
B. Increase force of contraction of the heart
C. Glycogenolysis
D. Contraction of most vascular smooth muscle

86. Neutralize gastric acid: Magnesium hydroxide- increases peristaltic activity causing osmotic
retention of fluids. It reduces stomach acid by reacting with
A. Magnesium hydroxide hydrochloric acid to form Mg Chloride.
B. Cimetidine
C. Sucralfate

D. Clarithromycin

87. Prototype of muscarinic antagonists: Atropine - nonselective competitive antagonist at all muscarinic
receptors in CNS and periphery, prototype of muscarinic
A. Atropine receptor blocking drugs
B. Physostigmine Hyoscine (Scopolamine) - unknown mechanism in CNS
C. Neostigmine Physostigmine - intermediate acting cholinesterase inhibitors
D. Hyoscine Neostigmine - intermediate acting cholinesterase inhibitors

88. It blocks sodium channel so that there is an increase in Quinidine acts as a blocker of voltage gated Na channels. The
the threshold of excitability quinidine effects on Na channels is known as a use of dependent
block. This means the higher rates, the block increases, while at
A. Hydrochlorothiazide lower rates the block decreases.
B. Verapamil
C. Propranial
D. Quinidine

89. Drugs that mimic the action of acetylcholine Cholinergic drug, any of various drugs that inhibit, enhance, or mimic
the action of the neurotransmitter acetylcholine, the primary
A. Sympatholytic drugs transmitter of nerve impulses within the parasympathetic nervous
B. Parasympathomimetic drugs system
C. Parasympatholytic drugs
D. Sympathomimetic drugs

90. Effect of atropine The anticholinergic effects of atropine can produce tachycardia, pupil
dilation, dry mouth, urinary retention, inhibition of sweating
A. increased GIT motility and secretions (anhidrosis), blurred vision and constipation. However, most of these
B. tachycardia side effects are only manifested with excessive dosing or with
C. Bronchoconstriction repeated dosing.

“Courage is found in unlikely places.”


― J.R.R. Tolkien 93
D. High doses produce sedation

91. The new drug studied in patients with the target Phase 0
disease to determine its efficacy and the doses to be
Phase 0 trials are the first clinical trials done among people. They aim
used in any follow-on trials:
to learn how a drug is processed in the body and how it affects the
A. Phase 1 Clinical Trials body. In these trials, a very small dose of a drug is given to about 10
B. Phase 2 clinical trials to 15 people.
C. Phase 3 Clinical Trials
Phase I
D. Phase 4 Clinical Trials
Phase I trials aim to find the best dose of a new drug with the fewest
side effects. The drug will be tested in a small group of 15 to 30
patients. Doctors start by giving very low doses of the drug to a few
patients. Higher doses are given to other patients until side effects
become too severe or the desired effect is seen. The drug may help
patients, but Phase I trials are to test a drug’s safety. If a drug is found
to be safe enough, it can be tested in a phase II clinical trial.

Phase II
Phase II trials further assess safety as well as if a drug works. The drug
is often tested among patients with a specific type of cancer. Phase II
trials are done in larger groups of patients compared to Phase I trials.
Often, new combinations of drugs are tested. Patients are closely
watched to see if the drug works. However, the new drug is rarely
compared to the current (standard-of-care) drug that is used. If a drug
is found to work, it can be tested in a phase III clinical trial.

Phase III
Phase III trials compare a new drug to the standard-of-care drug.
These trials assess the side effects of each drug and which drug works
better. Phase III trials enroll 100 or more patients.

Often, these trials are randomized. This means that patients are put
into a treatment group, called trial arms, by chance. Randomization is
needed to make sure that the people in all trial arms are alike. This
lets scientists know that the results of the clinical trial are due to the
treatment and not differences between the groups. A computer
program is often used to randomly assign people to the trial arms.

There can be more than two treatment groups in phase III trials. The
control group gets the standard-of-care treatment. The other groups
get a new treatment. Neither you nor your doctor can choose your
group. You will also not know which group you’re in until the trial is
over.

Every patient in a phase III study is watched closely. The study will be
stopped early if the side effects of the new drug are too severe or if
one group has much better results. Phase III clinical trials are often
needed before the FDA will approve the use of a new drug for the
general public.

Phase IV
Phase IV trials test new drugs approved by the FDA. The drug is tested
in several hundreds or thousands of patients. This allows for better
research on short-lived and long-lasting side effects and safety. For

“Courage is found in unlikely places.”


― J.R.R. Tolkien 94
instance, some rare side effects may only be found in large groups of
people. Doctors can also learn more about how well the drug works
and if it’s helpful when used with other treatments.

92. The actions of the body on the drug The action of drugs on the human body is called pharmacodynamics,
and what the body does with the drug is called pharmacokinetics
A. Pharmacogenomic
B. Pharmacodynamic
C. pharmacosystematic
D. Pharmacokinetic

93. The principal action of alpha adrenergic blocking agent The alpha-2 receptor acts as an allosteric inhibitor through Gi
in function, leading to an inhibition of adenylyl cyclase, decreasing the
formation of cAMP. It also leads to a reduced amount of cytoplasmic
A. Central vasoconstriction calcium, which decreases neurotransmitter release and central
B. Peripheral vasodilation vasodilation
C. Central vasodilation (???)
D. Peripheral vasoconstriction

94. Alpha 1 agonist:

A. Phenylephrine
B. Dobutamine
C. Propranolol

D. Prazosin

95. Cholinesterase regenerator

A. Atropine
B. Scopolamine
C. Pralidoxime

D. Ipratropium

“Courage is found in unlikely places.”


― J.R.R. Tolkien 95
96. Beta 1 agonist

A. Chrolodine
B. Dopamine
C. Dobutamine
D. Propanolol

97. Adrenaline is conjugated and oxidized in the: It is rapidly conjugated and oxidized in GI mucosa and liver.

A. Spleen
B. Small Intestine
C. Pancreas
D. Liver

Phase III
98. Full scale evaluation of treatment comparing it with Phase III trials compare a new drug to the standard-of-care drug.
standard treatment is done: These trials assess the side effects of each drug and which drug works
better. Phase III trials enroll 100 or more patients.
A. Phase 4 Clinical Trial
B. Phase 3 Clinical Trial
C. Phase 1 Clinical Trial
D. Phase 2 Clinical Trial

99. Reduces smooth muscle and secretory activity of the Anticholinergic agents such as dicyclomine (Bentyl, Axcan
gut Scandipharm) and hyoscyamine (Levsin, Levbid, NuLev, all by Schwarz
Pharma) are commonly used in the United States to treat pain
A. Ipratropium associated with functional intestinal disorders. These agents are
B. Atropine smooth muscle relaxants that block the muscarinic effects of
C. Dicytlomine acetylcholine on the gastrointestinal tract, thereby relaxing smooth
D. Scopolamine muscle and potentially reducing spasm and abdominal pain, slowing
intestinal motility, and decreasing diarrhea

Atropine reduces secretions in the mouth and respiratory passages,


relieves the constriction and spasm of the respiratory passages, and
may reduce the paralysis of respiration, which results from actions of
the toxic agent on the central nervous system.

100. Comprises small scale trials on patients used to Phase 0


determine dose level and establish that the treatment
Phase 0 trials are the first clinical trials done among people. They aim
offers some benefit:
to learn how a drug is processed in the body and how it affects the
A. Phase 2 Clinical Trial body. In these trials, a very small dose of a drug is given to about 10
B. Phase 3 Clinical Trial to 15 people.
C. Phase 1 Clinical Trial
Phase I
D. Phase 5 Clinical Trial
Phase I trials aim to find the best dose of a new drug with the fewest
side effects. The drug will be tested in a small group of 15 to 30

“Courage is found in unlikely places.”


― J.R.R. Tolkien 96
patients.

Phase II
Phase II trials further assess safety as well as if a drug works. The drug
is often tested among patients with a specific type of cancer. Phase II
trials are done in larger groups of patients compared to Phase I trials.

Phase III
Phase III trials compare a new drug to the standard-of-care drug.
These trials assess the side effects of each drug and which drug works
better. Phase III trials enroll 100 or more patients.

101. Side effects of Methyldopa:

A. Sedation and Vertigo


B. Nausea
C. Sedation, vertigo, and nausea
D. vertigo
E. sedation

102. Reduce gastric acid secretions: Histamine H2-receptor antagonists and proton pump inhibitors are
the main classes of drug used to inhibit gastric acid secretion. The
A. Clarithromycin former act by reversibly blocking the action of histamine, which is
B. Magnesium Hydroxide released from other mucosal cells in anticipation of a meal or when
C. Dicyclomine food enters the stomach.

D. Scopolamine
???
103. Atypical lymphocytosis is most likely to be found in Infectious Mononucleosis
which one of the following diseases? These atypical lymphocytes (ie, Downey cells) actually are CD8
cytotoxic T cells. B-cell infection caused by EBV leads to the
A. Chronic Hepatitis C transformation of the B cells to immortal plasmacytoid cells,

“Courage is found in unlikely places.”


― J.R.R. Tolkien 97
B. Parvovirus Infection which secrete a wide variety of immunoglobulins (eg,
C. Monoclonal induced by Epstein Barr virus heterophile antibodies).

D. Gastroenteritis

104. Which of the following is a leading cause of pneumonia Pneumonia in infants aged three weeks to three months is most often
primarily in infants? bacterial; Streptococcus pneumoniae is the most common pathogen.
In infants older than four months and in preschool-aged children,
A. Mumps viruses are the most frequent cause of CAP; respiratory syncytial
B. Cytomegalovirus virus (RSV) is the most common.
C. Respiratory syncytial virus
D. Rhinovirus

105. A child develops impetigo on the right thigh. He was Penicillins and cephalosporins are the major antibiotics that inhibit
given an antibiotic that will inhibit cell wall synthesis. bacterial cell wall synthesis.
Which is the most likely antibiotic this child would have
received?

A. Cephalexin
B. Erythromycin
C. Oproticxaxin
D. Ampicillin

106. Which of the following causes a fatal encephalitis for


which a vaccine is available? Encephalitis, or inflammation of the brain, is a treatable but
potentially-serious medical condition that can result from immune
A. Cytomegalovirus
B. Mumps system disorders. This includes disorders triggered by the MMR
C. Rhinovirus vaccine. According to the CDC:
D. Rabies
“Measles inclusion body encephalitis, or severe brain swelling caused
by the measles virus, is a complication of getting infected with the
wild-type measles virus. While rare, this disorder almost always
happens in patients with weakened immune systems. The illness
usually develops within 1 year after initial measles infection and has
a high death rate.”

107. What is a mass of fungal filaments called? Mycelium, plural mycelia, the mass of branched, tubular filaments
(hyphae) of fungi. The mycelium makes up the thallus, or
A. Hyphae undifferentiated body, of a typical fungus.
B. Mycelium
C. Yeast Hyphae each of the branching filaments that make up the mycelium
D. Seplum of a fungus.

Yeasts are eukaryotic, single-celled microorganisms classified as


members of the fungus kingdom.

“Courage is found in unlikely places.”


― J.R.R. Tolkien 98
108. Stabilize the mast cells so that release of histamine is
Cromolyn belongs to a class of medications called mast cell stabilizers. It
inhibited:
works by preventing mast cells from releasing the substances that can
A. Terbutaline
cause these skin and gut problems.
B. Salbutamol
C. Cromolyn sodium
D. Ipratropium

109. Which one of the following viruses is the leading cause Infections known to produce congenital defects have been described
of congenital malformations? with the acronym TORCH (Toxoplasma, others, rubella,
cytomegalovirus [CMV], herpes). The "others" category has rapidly
A. Rabies expanded to include several viruses known to cause neonatal disease.
B. Rhinovirus
C. Cytomegalovirus
D. Mumps

WINX CLUB:
BASIC PART 1

“Courage is found in unlikely places.”


― J.R.R. Tolkien 99
contains the auditory cortex that is responsible in
1. Where is the lesion located for a patient with inability to transmitting sounds
determine sounds heard in the absence of ear abnormalities?
The primary auditory cortex (A1) is located on the superior
temporal gyrus in the temporal lobe and receives point-to-
a. Temporal lobe point input from the ventral division of the medial
geniculate complex; thus, it contains a precise tonotopic
b. Superior temporal gyrus map.

c. Occipital lobe The Auditory Cortex - Neuroscience - NCBI Bookshelf


(nih.gov)
d. Superior parietal lobe

2. Where are the nicotinic receptors in the myoneural junction The sarcolemma at the junction has invaginations called
located? postjunctional folds, which increase its surface area facing
the synaptic cleft. These postjunctional folds form the
a. Terminal ends of the motor neuron motor end plate, which is studded with nicotinic
acetylcholine receptors (nAChRs) at a density of 10,000
b. T-tubule

c. Epimysium

d. Motor end-plate

3. Student X has normal defecation which is mostly due to the Sympathetic supplies rectum to the rectum is from lumbar
activity of the: splanchnic nerves and superior and inferior hypogastric
plexuses.
a. Pelvic Splanchnic Nerves
Parasympathetic supplies are S4 via the PELVIC
b. Sympathetic Nerves SPLANCHNIC NERVES and inferior hypogastric plexuses.

c. Vagus Nerves

d. Prevertebral ganglia

“Courage is found in unlikely places.”


― J.R.R. Tolkien 100
4. Seeing Korean food on your Grab app made your mouth
watery and is due to the activation of which preganglionic
neuron?

a. Otic ganglion

b. Inferior salivatory nucleus

c. Submandibular ganglion

d. Superior salivatory nucleus

5. While explaining anemia to a 32 year old patient, you


explain that this bone is no longer a site for hematopoiesis: Hematopoiesis is the process by which immature cells
develop into mature blood cells. In adults the site of
o Ribs hematopoiesis is located in the bone marrow of the skull,
o Sternum ribs, sternum vertebral column, pelvis, proximal ends of
o Vertebra femur. FEMUR GRADUALLY DECREASED IN THE
o Femur HEMATOPOIESIS AT AROUND 20-30 YEARS OLD.

6. As part of the neurologic exam, the cranial nerves Metencephalon


were tested and you elicited the corneal reflex which helps you
access which region of the CNS?

a. Mesencephalon

b. Myelencephalon

c. Metencephalon

d. Diencephalon

“Courage is found in unlikely places.”


― J.R.R. Tolkien 101
7. A newly wed male argues that the newborn baby with blood
type O is not his. The father is a type B homozygous and the 25%
mother is a type AB, what is the probability that this newborn
will have that blood type?

a. 0%

b. 25%

c. 50%

d. 75%

8. Why would tetany not happen in cardiac muscles? The refractory period of cardiac muscle is dramatically
longer than that of skeletal muscle. This prevents tetanus
a. They have a more calcium ions from occurring and ensures that each contraction is
followed by enough time to allow the heart chamber to
b. They have a long absolute refractory period refill with blood before the next contraction.

c. They have more mitochondria than


skeletal muscles

d. They have a long hyperpolarized period

9. For a left-handed person, an ischemic event to this area


presents a problem in the regulation of speaking: ● left inferior frontal gyrus houses the broca’s area
● broca’s area is responsible in expressing spoken and
a. Left superior temporal gyrus written language

b. Left inferior frontal gyrus

c. Right superior temporal gyrus

d. Right inferior frontal gyrus

“Courage is found in unlikely places.”


― J.R.R. Tolkien 102
10. A patient complaining of frequent bruising followed up
with a bleeding time result that is prolonged, this may be due Underlying medical disorders may cause a bleeding
to: phenotype due to thrombocytopenia .

a. Vit. K deficiency

b. Thrombocytopenia

c. Hemophilia

d. Protein C deficiency

11. After eating a heavy breakfast, the distention of the


stomach will: The gastrocolic reflex is a physiological reflex that controls
the motility of the lower gastrointestinal tract following a
o Trigger gastrocolic reflex meal. As a result of the gastrocolic reflex, the colon has
increased motility in response to the stretch of the stomach
o Inhibit gastric emptying with the ingestion of food.

o Decrease salivary secretion

o Cause glucose absorption

12. While playing basketball, a blunt injury to the neck Fractures of the clavicle are frequent injuries and are
occurred after which a tingling sensation in the clavicular area responsible for 2% to ... from the C3 and C4 nerve roots of
was felt. The most probable level of injury is at: the superficial cervical plexus

a. C1

b. C3

c. C5

d. C7

“Courage is found in unlikely places.”


― J.R.R. Tolkien 103
13. A patient with a failing heart was prescribed cardiac Answer: Increase myocardial contractile force
glycosides in order to:
Cardiac glycosides increase the force of contraction of the
a. Increase the excretion of fluids muscle of the heart by inhibiting the activity of an enzyme
(ATPase) that controls the movement of Calcium, Sodium
b. Increase myocardial contractile force and Potassium into the heart muscle.

c. Increase the storage of fluids in the venous


system

d. Decrease the absorption of sodium

14. Checking a patient’s arterial blood gas, you take note that Increase in pH
this factor increases the affinity of hemoglobin to oxygen: The following physiological factors influence the affinity of
haemoglobin for oxygen:
a. Increase in pH
● The partial pressure of CO2
b. Increase in pCO2
○ Increasing CO2 shifts the curve to the right
c. Increase in 2,3 DPG ○ Hyperventilation and hypocapnia shifts the
curve to the left
d. Increase in temperature ● pH, independent of CO2
○ Decreasing pH (acidosis) shifts the curve to the
right
○ Alkalosis shifts the curve to the left
● The concentration of 2,3-DPG inside the
erythrocytes
○ Increased 2,3-DPG (eg. in response to hypoxia
or erythropoietin) shifts the curve to the right
○ Decreased 2,3-DPG (eg. as a red cell storage
lesion ) shifts the curve to the left
● The presence of unusual haemoglobin species
○ Methaemoglobin, carboxyhaemoglobin and
foetal haemoglobin shift the curve to the left
○ Sulfhaemoglobin shifts the curve to the right
● Temperature
○ Hyperthermia shifts the curve right
○ Hypothermia shifts it left

“Courage is found in unlikely places.”


― J.R.R. Tolkien 104
15. A patient complains of palpitations. While examining him,
you place the diaphragm of the stethoscope at which best 5th ICS, LMCL
location if you want to hear the sound produced by closing of
the mitral valve?

a. 4th ICS, LMCL

b. 4th ICS, LPSM

c. 5th ICS, LPSM

d. 5th ICS, LMCL

16. Lymph from the skin of the lateral side of the foot drains
into ____? Popliteal Nodes

a. Internal iliac node The popliteal lymphatic nodes are small in size, usually
between five and seven in number, and are often found
b. Inguinal node embedded in fat reserves in the popliteal fossa. They
receive lymph from the lateral superficial vessels.
c. Subsartorial nodes

d. Popliteal node

17.Most fractured carpal bone to suspect if one will fall from > Scaphoid fractures are by far the most common of the
an outstretched hand? carpal fractures, and account for 10 percent of all hand
fractures and about 55 percent of all carpal fractures
a. Pisiform
>The scaphoid is usually fractured secondary to
b. Capitate hyperextension of the wrist, often from falls onto the
outstretched hand. The scaphoid is wedged between the
c. Hamate radius and the surrounding carpal bones, particularly the
capitate.
d. Scaphoid

“Courage is found in unlikely places.”


― J.R.R. Tolkien 105
18. The penile urethra is embedded in what specific structure? > The urethra is located concentrically within the corpus
spongiosum.
a. Corpus spongiosum
>The penile urethra runs through the penis, within the
b. Corpus cavernosum corpus spongiosum

c. Corpus striatum

d. Corpus spinosum

19. Which part of the intestines is movable? > The transverse colon is the longest and most movable
part of the colon
a. Rectum

b. Sigmoid

c. Transverse colon

d. Ascending colon

20. Largest and strongest bone of the foot. > Calcaneus: The largest bone of the foot, it is commonly
referred to as the heel of the foot. It points upward, while
a. Calcaneus -1st the remaining bones of the feet point downward.

b. Cuboid

c. Cuneiform

d. Talus -2nd

“Courage is found in unlikely places.”


― J.R.R. Tolkien 106
21. Children aged 12 months through age 12 years can be given Vaccines utilise this adaptive immunity and memory to
2 doses of varicella vaccine subcutaneously, separated by at expose the body to the antigen without causing disease, so
least 3 months and results in faster and higher antibody that when then live pathogen infects the body, the
production brought by which attribute of B cells? response is rapid and the pathogen is prevented from
causing disease.
a. Memory

b. Clonal proliferation

c. Antigenicity

d. Specificity

22. A patient with acute gastroenteritis with no signs of lower concentration to high concentration (active,uphill)
dehydration was given oral rehydration solution. Glucose with
sodium in the ORS when absorbed in the GIT exemplifies what
type of transport process?

a. Primary active transport

b. Passive transport

c. Secondary active transport

d. Vesicular transport

“Courage is found in unlikely places.”


― J.R.R. Tolkien 107
23. Patient X obtained abrasions from falling off his bicycle
after avoiding a pedestrian. Which is NOT expected from the migration of neutrophils into the site of injury
resulting inflammatory process?

a. Migration of large number of granulocytes


and monocytes into the injured tissue

b. Vasoconstriction of local blood vessels


decreasing local blood flow

c. Clotting of fluid in the interstitial space due


to leakage of fibrinogen

d. Increased permeability of the capillaries


with leakage of fluid into the interstitium

24. A patient with bilious vomiting showed this compensatory


mechanism:

a. Increased secretion of H+ ion by the


proximal tubules
b. Increased generation of hydrogen ions
from food
c. Increased activity of beta intercalated cells
of collecting ducts
d. Decreased rate and depth of breathing

25. Which of the following factors decrease the cardiac output Afterload The resistance against which the ventricle must
in normal physiologic states? pump. Excessive afterload = difficult to pump blood →
reduced CO/SV
a. Venous Return
b. Afterload
c. Effective filling time
d. Effective filling pressure

“Courage is found in unlikely places.”


― J.R.R. Tolkien 108
26. The presence of this manifestation in your patient made
you pinpoint the location of his brain tumor in the medial side
of the medulla oblongata:

a. Impaired gag reflex


b. Inability to close an eye
c. Deviation of the neck on one side
d. Paralysis of the tongue

27. Hypokalemia was found in a hypertensive patient on


prolonged hydrochlorothiazide who had difficulty ambulating.
This finding can explain the muscle weakness due to:

a. Hyperpolarization of the resting membrane


potential requiring greater stimulus intensity
to initiate an action potential

b. Hyperpolarization of the resting membrane


potential requiring lesser stimulus intensity
to initiate an action potential

c. Hypopolarization of the resting membrane


potential requiring greater stimulus intensity
to initiate an action potential

d. Hypopolarization of the resting membrane


potential requiring lesser stimulus intensity
to initiate an action potential

“Courage is found in unlikely places.”


― J.R.R. Tolkien 109
28. Which of the following will lead to an increase in stroke
volume?

a. An increase in aortic pressure

b. A decrease in venous return

c. A highly tachycardic heart

d. A more negative intrathoracic pressure

29. Which of the following is NOT associated with the


development of a thromboembolic condition?

a. Normal bleeding time and clotting time

b. Platelet count of 150,000-300,000/ul

c. Smooth endothelial surface

d. Adequate levels of calcium on the blood

30. After eating a hamburger, most of its chemical digestion majority of chemical digestion occurs in the SI; this is
will occur where in the GI tract? where chyme will be mixed with other digestive juice from
the pancreas and bile form the liver
a. Esophagus

b. Stomach

c. Small intestines

d. Large intestines

“Courage is found in unlikely places.”


― J.R.R. Tolkien 110
31. Weakest area or part of the shoulder joint.
a. Anterior
b. Posterior
c. Superior
d. Inferior

32. Part of the brain that separates frontal from parietal lobe The central sulcus runs posterior-medial to anterior-lateral
of cerebrum: and separates the frontal lobe from the parietal lobe

a. Precentral gyrus

b. Central sulcus of Rolando

c. Lateral Sylvian fissure

d. Longitudinal fissure

33. As the facial nerve emerges from the stylomastoid Ans: Posterior auricular nerve
foramen, it immediately gives out a branch that goes behind
the ear to innervate the occipitalis and auricularis posterior
muscles. What do you call that nerve branch?

a. Great auricular nerve


b. Posterior auricular nerve
c. Marginal mandibular nerve
d. Auriculotemporal nerve

“Courage is found in unlikely places.”


― J.R.R. Tolkien 111
34. What vertebra that is identified by the presence of Ans: Lumbar
mammillary process?

a. Atlas

b. Cervical

c. Thoracic

d. Lumbar

35. The antero-posterior diameter of pelvic inlet that can be Ans: Diagonal conjugate
measured through vaginal exam:
The anteroposterior diameter from the sacrococcygeal
a. Diagonal conjugate joint to the lower border of the symphysis pubis is
estimated through measuring the diagonal conjugate
b. Obstetrics conjugate which is normally about 15in.(13cm).

c. Sagittal conjugate

d. Transverse conjugate

36. Which of the following nerves is NOT a branch of CN VII: Ans: Mental

a. Buccal Facial nerve runs forward within the substance of the


parotid gland, it divides into its five terminal branches:
b. Mental
● Temporal
c. Mandibular ● Zygomatic
● Buccal
d. Temporal ● Mandibular
● Cervical

“Courage is found in unlikely places.”


― J.R.R. Tolkien 112
37. The following are unpaired cranial bones, EXCEPT:
Ans: Temporal (Paired bone)
a. Frontal
b. Temporal
c. Sphenoid (Unpaired bones)
d. Occipital ● Fontal
● Ethmoid
● Sphenoid
● Occipital

38. Which of the following cranial nerves does NOT originate Ans: Trochlear (Midbrain) CN IV
from the pons?

a. Trochlear

b. Trigeminal

c. Abducens

d. Facial

“Courage is found in unlikely places.”


― J.R.R. Tolkien 113
39. This muscle flexes the metatarsophalangeal joint of the big Flexor hallucis brevis - felexes the metatarsopharyngeal
toe and also supports the medial longitudinal arch joint of the big toe, supports the medial longitudinal arch.

a. Flexor hallucis longus Flexor hallucis longus - flexes the distal phalanx of the big
toe, plantar flexes the foot, and supports the medial
b. Flexor hallucis brevis longitudinal arch
Adductor hallucis oblique head - flexes the big toe, supports
c. Adductor hallucis Oblique head the transverse arch
Lumbricals - flex MCP joints and extends interpharyngeal
d. Lumbricals joints of fingers.

40. Which of the following is not part of the Levator ani? Ans: Coccygeus

a. Puborectalis Parts of Levator ani


● Iliococcygeus
b. Coccygeus ● Pubococcygeus
● Puborectalis
c. Iliococcygeus

d. Pubococcygeus

41. A patient followed up for the reading of his tuberculin skin B. Tuberculin-Type Hypersensitivity
test, which you interpret as a positive response. This is due to
which type of hypersensitivity reaction? Delayed hypersensitivity to antigens of microorganisms
occurs in many infectious diseases and it has been used as
a. Immediate an aid in diagnosis. It is typified by the tuberculin reaction.
When a small amount of tuberculin is injected into the
b. Delayed epidermis of a patient previously exposed to
Mycobacterium tuberculosis,there is little immediate
c. Antibody-mediated reaction. Gradually, however, induration and redness
develop and reach a peak in 24–72 hours. Mononuclear
d. Immune-complex cells accumulate in the subcutaneous tissue, and there are
CD4 Th1 cells in abundance. A positive skin test indicates
that the person has been infected with the agent but does
not imply the presence of current disease. However, a
recent change of skin test response from negative to
positive suggests recent infection and possible current
activity. A positive skin test response assists in diagnosis.
For example, in leprosy, a positive skin test indicates
tuberculoid disease with active cell-mediated immunity,
whereas a negative test suggests lepromatous leprosy with
weak cell-mediated immunity.
Jawetz 27th edition Page 142

“Courage is found in unlikely places.”


― J.R.R. Tolkien 114
42. When testosterone produced by the Leydig cells of the Forms of chemical signaling
testes acts on adjacent seminiferous tubules to stimulate Autocrine - a cell targets itself
spermatogenesis, what type of chemical signaling is Paracrine- a cell targets nearby cell
exemplified? Endocrine- a cell targets a distant cell through the
bloodstream
a. Autocrine Signaling across the gap junctions- a cell targets a cell
connected by gap junctions
b. Endocrine (Juxtacrine- involves between cells)

c. Juxtracrine

d. Paracrine

journals.sagepub.com/doi/pdf/10.1177/092623389017002
19
Page 419

“Courage is found in unlikely places.”


― J.R.R. Tolkien 115
43. Under normal circumstances, which neurotransmitter Under normal circumstances, glycine provides inhibition
provides inhibition of muscle tone that balances the excitation of muscle tone that balances the excitation of muscle tone
of muscle tone provided by other neurotransmitters? provided by other neurotransmitters

a. Glycine

b. Glutamine

c. Glutamate

d. Glucagon

44. Which electrolyte can cause cardiac arrest in diastole when Extremely high levels of potassium in the blood (severe
it is in excess? hyperkalemia) can lead to cardiac arrest and death.

a. Calcium Hyperkalemia- Associated cardiac arrhythmias include sinus


bradycardia, sinus arrest, slow idioventricular rhythms,
b. Chloride ventricular tachycardia, ventricular fibrillation and asystole
Hypernatremia- Primarily neurologic: change in sensorium
c. Potassium ? is the most common manifestation
Hyperchloremia- Effects on the cardiovascular system
d. Sodium include direct impairment of myocardial contraction
(especially at a pH < 7.2), tachycardia, and increased risk of
ventricular fibrillation or heart failure with pulmonary
edema. Patients may report dyspnea upon exertion or, in
severe cases, at rest.
Hypercalcemia- Bradycardia, AV block, arrhythmias and
shortened QT-interval

45. Motor fibers that form the corticospinal tract are mostly
coming from the:

a. Premotor area

b. Primary motor area

c. Primary somatosensory area

d. Supplementary motor area

Reference:
https://www.neuroscientificallychallenged.com/blog/know
-your-brain-corticospinal-tract

“Courage is found in unlikely places.”


― J.R.R. Tolkien 116
46. Educating your patient why he is experiencing heartburn Achalasia is a condition of impaired esophageal
after eating meals, you explained that this is because of the contractions resulting from degeneration of myenteric
inability of the lower esophageal sphincter to contract (auerbach) plexuses in the esophagus. The lower
effectively when food enters the stomach, also called as: esophageal sphincter fails to relax during swallowing.

a. Dysphagia

b. Achalasia

c. Gastric reflux

d. Hirschsprung disease Reference: BRS Gross Anatomy 9th edition

47. This blocks the active sites on actin for myosin cross-bridge
formation when calcium is not present:

a. Actinin and titin

b. Troponin-tropomyosin complex

c. Sarcolipin and myoregulin Reference: BRS Physiology 7th edition

d. Sarcoplasmic reticulum and the T-tubules

48. Your asthmatic patient complained that every time he uses


his reliever medication he experiences a feeling of incomplete Improves symptoms of urinary incontinence.
voiding. What kind of drug is his reliever medication?

a. Sympathomimetic

b. Parasympathomimetic

c. Parasympatholytic

d. Sympathomimetic and parasympatholytic

“Courage is found in unlikely places.”


― J.R.R. Tolkien 117
49. A patient with which condition will NOT exhibit an increase Factors that may cause a rise in Hct value such as:
in hematocrit? 1. Polycythemia Vera
2. Heart or kidney problems
a. Pregnancy 3. Intake of anabolic steroids
4. Dehydration
b. Dehydration 5. Diarrhea
6. Lung problems
c. Polycythemia Vera 7. Burns
8. Smoking
d. High altitude dwelling 9. High altitude

50. Among water soluble hormones, which of the following is Binding of Water-Soluble Hormones
NOT a 2nd messenger? Water-soluble hormones cannot diffuse through the cell
membrane. These hormones must bind to a surface cell-
membrane receptor. The receptor then initiates a cell-
a. Cyclic AMP signaling pathway within the cell involving G proteins,
adenylyl cyclase, the secondary messenger cyclic AMP
b. Cyclic GMP (cAMP), and protein kinases. In the final step, these protein
kinases phosphorylate proteins in the cytoplasm. This
c. Calcium activates proteins in the cell that carry out the changes
specified by the hormone.
d. Acetylcholine

BASICS PART 2

“Courage is found in unlikely places.”


― J.R.R. Tolkien 118
1. What is the major end- product of beta oxidation
of fatty acid? *Long-chain acyl-CoA enters the fatty acid β-oxidation pathway, which results in
the production of one acetyl-CoA from each cycle of fatty acid β-oxidation.

o Priopionyl CoA *Acetly CoA carboxylated is the rate limiting steps.


o Butyroyl CoA
o Acetoacetyl CoA
o Acetyl CoA
Source: medical review series

2. Which of the following is NOT a steroid hormone?

o Tetraiodothyronine (T4)
o Aldosterone
o Estriol
o Dihydrotestosterone

3. What is the primary biochemical lesion in Familial hypercholesterolemia (FH) results from deficiency or defective function
homozygotes with familial hypercholesterolemia? of LDLRs FH follows autosomal codominant inheritance, with a very severe
phenotype in homozygotes (HoFH), and a dramatic but less severe phenotype in
heterozygotes (HeFH)
o Loss of feed back inhibition of HG reductase
o Loss of apolipoprotein B
o Increased production of LDL from VLDL
o Functional deficiency of plasma membrane
receptors for LDL Source :https://www.sciencedirect.com/topics/medicine-and-
dentistry/familial-hypercholesterolemia

4. Which hormone is produced using two


mitochondrial enzymes?

o Testosterone
o Estradiol
o Aldosterone
o Cortisol

5. What is the complication of excessive level of ketones in the blood reaches an excess, such that the buffering capacity
ketogenesis? of the plasma is exhausted due to the depletion of bicarbonates, it can lead to
metabolic acidosis with a drop in blood pH, a condition known as ketoacidosis
that can occur in many metabolically deranged situations
o Metabolic acidosis
o Metabolic alkalosis

“Courage is found in unlikely places.”


― J.R.R. Tolkien 119
o Respiratory acidosis
o Respiratory alkalosis

6. What happens during each cycle of β-oxidation?

o One carbon atom is removed from carbonyl


end of the fatty acid
o One carbon atom is removed from the
methyl end of the fatty acid
o two carbon atoms are removed from
carbonyl end of the fatty acidT
o Two carbon atoms are removed from the
methyl end of the fatty acid

7. The “B” gene in ABO blood group codes for what


enzyme?

o G-6-PD
o Gal transferase
o Fucosyl transferase
o GalNAc transferase

8. Deficiency of this/these enzyme(s) can lead to


hyperammonemia:

o Carbamoyl phosphate synthetase 2


o Cystathionine synthetase
o Saccharopine dehydrogenase
o No correct answer among the choices

9. In fibrinolysis, plasminogen is activated to plasmin


and inhibits what factor?

o Factor X
o Factor XI
o Factor XII
o Factor XIII

“Courage is found in unlikely places.”


― J.R.R. Tolkien 120
10. Which of the following post-translational
modification is involved in regulation of the
activity of covalently modulated enzymes?

o Phosphorylation of serine
o Carboxylation of glutamic acid
o Glycosylation of tyrosine
o Removal of leader peptide

11. The binding of epinephrine to which receptor


inhibits adenylate cyclase from producing cAMP?

o Vasopressin-V2
o Vasopression-V1
o α2 adrenergic
o α1 adrenergic

Source: Katzung p. 99

12. Which of the following enzymes catalyzes the


conversion of cholesterol to cholic acid?

o 7-alpha hydroxylase
o 9-beta hydroxylase
o 17-alpha hydroxylase
o 19-beta hydroxylase

Source: Harper’s p. 273

13. The product of the reversed glutamate


dehydrogenase reaction:

o α-ketoglutarate
o oxaloacetate
o ammonia
o glutamic acid Source: Harper’s p. 291

“Courage is found in unlikely places.”


― J.R.R. Tolkien 121
14. There could be premature of termination of
protein synthesis when this codon is encountered
by the ribosome:

o UAG
o UCC
o CUG
o GAU

Source: Harper’s p. 423

15. Which of these steps in beta oxidation is rate-


limiting?

o Carnitine transport of the fatty acid


o Thiolytic cleavage of ketoacyl CoA
o Oxidation of fatty acyl CoA
o Activation of free fatty acid to fatty acyl CoA
Source: researchgate.net

16. What happens if the energy requirement of the If the energy requirement of the cell is increased: meaning it has decreased
cell is increased? energy, stimulation of glycolysis is required.

o Citrate synthase is stimulated


o G-6-P-D is stimulated
o Phosphofructokinase 2 is stimulated
o All choices

17. Action of the levator ani muscles when defecating


The act of defecation involves a coordinated reflex that results in the emptying
of the descending colon, sigmoid colon, rectum and anal canal. It is assisted by
o Are completely inactive a rise intra abdominal pressure brought by contraction of the muscles of
o Relax portion of puborectalis with the anal anterior abdominal wall. the tonic contraction of the internal and external
sphincters sphicters including puborectalis muscle, is now voluntarily inhibited and the
o Do not support the sigmoid colon feces are evacuated through anal canal.
o Do not support the uterus and vagina

18. The committed step in extramitochondrial The first step in fatty acid synthesis is to activate acetyl CoA by the addition of a
lipogenesis involves the formation of: carbon dioxide using acetyl Coa carboxylase. This reaction and is the flux of first
committed step of FA biosynthesis. Acetyl CoA carboxylase is responsible for
producing Malonyl CoA.
o Acetyl CoA
o Priopionyl CoA
o Succinyl CoA
o Malonyl CoA

“Courage is found in unlikely places.”


― J.R.R. Tolkien 122
19. The artery that supplies the gallbladder usually Cystic artery a branch of the right hepatic artery in the cystohepatic triangle.
arises from the _____ Hepatic artery is direct branch of celiac trunk or artery.

o Celiac artery
o Gastroduodenal artery
o Inferior mesenteric artery
o Superior mesenteric artery

20. These are the two starting materials needed in the Heme synthesis starts in mitochondria with the condensation of succinyl Coa
pathway in order for heme to be synthesized: with amino acid glycine, activated by pyrodoxical phosphate.

o Succinyl-CoA and leucine


o Succinyl-CoA and lysine
o Succinyl-CoA and glycine
o Succinyl-CoA and tyrosine

21. The uterus receives its blood supply from what


artery? uterine arteries are the main blood vessels that supply blood to the uterus. They
give off branches that supply different portions of the uterus and plays an
important role in maintaining blood supply during physiological processes,
o Uterine artery
o Uterine and ovarian arteries
o Middle rectal artery
o Superior vesical artery https://www.ncbi.nlm.nih.gov/books/NBK482267/

22. All of the following would cause a shift to the right


on the oxygen dissociation curve, EXCVEPT decrease in pH (acidity) shifts the dissociation curve to the right

decrease in pH (acidity) shifts the dissociation curve to the right


o Hypothermia
o Hyperthermia higher temperatures which will cause a rightward shift
o Decreased pH
o Increased 2,3 DPG https://www.statpearls.com/ArticleLibrary/viewarticle/26495

“Courage is found in unlikely places.”


― J.R.R. Tolkien 123
23. The following are 1st layer muscles of the sole,
EXCEPT:

o Abductor hallucis muscle


o Abductor digiti minimi muscle
o Flexor digitorum brevis muscle
o Quadratus plantae muscle

24. Anemia secondary to malaria is classified under? The most common extrinsic causes of hemolytic anemia includes:
25. Hyperactive immune system (autoimmune hemolytic anemia)
26. Infections
o Extrinsic hemolysis 27. Certain medications
o Acute blood loss 28. Abnormal blood vessels
o Intrinsic hemolysis 29. Trapping in the spleen
o Disorder of erythropoiesis 30. Some cancers, including leukemia and lymphoma

31. TRUE of the Subtalar joint: The joints move in a triaxial plane, which allows for the motions of
flexion/extension, inversion/eversion, and adduction/abduction.

o Consists of calcaneocuboid
o Also known as “Chopart’s joint”
o Synovial joint that can evert/invert foot
o Transverse tarsal joint

“Courage is found in unlikely places.”


― J.R.R. Tolkien 124
32. Which is true regarding degeneracy of the genetic
code?

o Involves the 1st base of the codon


o Involves the 3rd base of the codon
o Involves the N-terminal end of the
polypeptide
o Involves the C-terminal end of the
polypeptide

“Courage is found in unlikely places.”


― J.R.R. Tolkien 125
33. Common action of Gastrocnemius, Soleus and
Plantaris muscles on the foot

o Dorsiflexion
o Inversion
o Eversion
o Plantarflexion

34. Which of the following regarding functions of Naive lymphocytes expressing the chemokine receptor CCR7 continuously
immune cells under natural immunity is migrate into lymph nodes (LN) from the blood via specialized high endothelial
CORRECT? venules (HEV), while antigen-bearing dendritic cells (DC), also expressing CCR7,
carry antigen from the periphery into the LN via afferent lymphatics.

o Basophils produce reactive oxygen species to


kill pathogens it phagocytosed
o Dendritic cells secrete chemokines that
orchestrate the migration of other immune
cells within the lymph nodes
o Neutrophils during inflammation potentiates
activation of B and T cells by the expression
of co-stimulatory molecules
o Natural Killer cells produce and release
cationic content of granules to induce
inflammation

“Courage is found in unlikely places.”


― J.R.R. Tolkien 126
35. The levator ani muscle receives its innervation
from the

o Inferior rectal nerve


o Obturator nerve
o Hypogastric plexuses
o Pudendal nerve and the perineal branch of S4

36. If G-6-P-D is deficient:

o Hydrogen peroxide is detoxified to water


o RBC membrane is intact
o Glutathione remains in its reduced form
o Glutathione is oxidized

37. Cancer of the prostate can metastasize to the skull The predilection of prostate cancer to the skullmay be due to Batson's venous
via the plexus that connects prostate with the skull throughepidural and dural veins [9].
Therefore, there should be deliberate attempt at assessing possibility of skull
metastasis in all patients with advanced prostate cancer.

“Courage is found in unlikely places.”


― J.R.R. Tolkien 127
o Inferior vena cava
o Portal vein
o Vertebral venous plexus
o Pampiniform plexus

38. Formation of the peptide bond is inhibited by this tiamulin


antibiotic:
Thereby, tiamulin directly inhibits peptide bond formation. Comparison of the
tiamulin binding site with other PTC targeting drugs, like chloramphenicol,
o Penicillin clindamycin and streptogramins, may facilitate the design of modified or
o Cephalexin hybridized drugs that extend the applicability of this class of antibiotics.
o Erythromycin
o Chloramphenicol

39. Ejaculation is the response to the activity of the Erections occur in response to tactile, olfactory, and visual stimuli. The ability to
achieve and maintain a full erection depends not only on the penile portion of
o Obturator nerves the process but also on the status of the peripheral nerves, the integrity of the
o Sympathetic nerves vascular supply, and biochemical events within the corpora. The autonomic
o Parasympathetic nerves nervous system is involved in erection, orgasm, and tumescence. The
o Perineal branch of S4 parasympathetic nervous system is primarily involved in sustaining and
maintaining an erection, which is derived from S2-S4 nerve roots.

40. Hereditary spherocytosis is secondary to a defect Decreased surface area may produced by two different mechanisms: 1)
in what red cell membrane protein? Defectsof spectrin, ankyrin, or protein 4.2 lead to reduced density of the
membraneskeleton, destabilizing the overlying lipid bilayer and releasing band
3-containing microvesicles.
o Ankyrin
o Anion exchange protein
o Glycophorin
o Spectrin

41. True regarding the biliary tree?

o Union of the major and minor pancreatic


ducts will form the Ampulla of Vater
o Union of the right and left hepatic ducts will
form the common bile duct
o Union of the cystic duct and common bile
duct will form the common hepatic duct
o Union of the common hepatic duct and cystic
duct will form the common bile duct

“Courage is found in unlikely places.”


― J.R.R. Tolkien 128
42. Proteins with garbled amino acid sequence are Missense mutation if the mRNA codon codes for a different amino acid, or a
produced when there is: nonsense mutation if the mRNA codon becomes a stop codon.
Transition mutations occur when a pyrimidine base (i.e., thymine [T] or cytosine
[C]) substitutes for another pyrimidine base
o Missense mutation
o Deletion frameshift mutation Transversion mutations occur when a purine base substitutes for a pyrimidine
base, or vice versa
o Transition point mutation
o Transversion point mutation

43. Which of the following is NOT a boundary of the


Calot triangle?

o Cystic artery
o Gastroduodenal artery
o Common hepatic duct
o Inferior surface of the liver

44. Which structural feature is found in a single C - The covalent structure of RNA differs from that of DNA in two respects. The
stranded DNA molecule? sugar units in RNA are riboses rather than deoxyriboses. Ribose contains a 2′-
hydroxyl group not present in deoxyribose.

o Each purine and pyrimidine pairs with its


complementary base
o Each 3’-5’ phosphodiester bond contains one
phosphate group linking 2 deoxyribose
sugars.
o It can have one end with a 5’-phosphate
group and the other end has a 2’ -hydroxyl
group
o With sugar and phosphate as backbone and
nitrogen bases in the center which hold the
strands together

45. Erection of the penis is a response to the activity


of the B – PARA SYMPHATETIC PENILE ERCTION AND SYMPHATETIC FLACCID STAGE

o Sympathetic nerves
o Parasympathetic nerves

“Courage is found in unlikely places.”


― J.R.R. Tolkien 129
o Ilioinguinal nerves
o Perineal branch of S4

46. A 35-year-old man with severe A -The defect makes the body unable to remove low density lipoprotein (LDL, or
hypercholesterolemia has a family history of bad) cholesterol from the blood. This results in a high level of LDL in the blood.
deaths at a young age from heart disease and This makes you more likely to have narrowing of the arteries from
stroke. Which of the following genes is likely to be atherosclerosis at an early age.
defective?

o LDL receptor
o Lipoprotein lipase
o Apolipoprotein E
o LCAT

47. The antero-posterior diameter of pelvic inlet that


can be measured through vaginal exam

o Obstetrics conjugate
o Diagonal conjugate
o Sagittal conjugate
o Transverse conjugate

48. A 7-year-old child was brought to the clinic for


nutritional assessment. He has winged scapulae
and old man facie which can be found among
patients with:

o Kwashiorkor
o Marasmus
o Obesity
o 1 malnutrition

“Courage is found in unlikely places.”


― J.R.R. Tolkien 130
49. The internal anal sphincter receives its nerve
supply from the

o Obturator nerve
o Hypogastric plexuses
o Vagus nerve
o Pudendal nerve and the perineal branch of S4

50. How is the DNA template read during The antisense strand of DNA is read by RNA polymerase from the 3' end to the
transcription? 5' end during transcription (3' → 5'). The complementary RNA is created in the
opposite direction, in the 5' → 3' direction, matching the sequence of the sense
strand with the exception of switching uracil for thymine
o From 5’ to 3’ end
o From 3’ to 5’ end
o From C-terminal to N-terminal
o From N-terminal to C-terminal

51. A deep laceration at the anterior axillary fold will The anterior axillary fold is formed by the lateral edge of the pectoralis major
traumatize this structure: muscle;

o Pectoralis major
o Latissimus dorsi
o Coracobrachialis
o Subscapularis muscle

52. This is a hemoglobinopathy whereby the iron in a similar reaction also occurs during the spontaneous oxidation of haemoglobin
ferrous (Fe2+) state is converted to the ferric (Fe3+) to methaemoglobin
state:

o HbS
o α Thalassemia
o Beta Thalassemia
o Methemoglobinemia

“Courage is found in unlikely places.”


― J.R.R. Tolkien 131
53. Insulin lowers the blood glucose level by all of the
following mechanisms, EXCEPT:

o Increasing lipogenesis
o Increasing the synthesis of proteins
o Promoting glycogenolysis
o Promoting the mobilization of glucose
transporters

54. What happens if an amino acid undergoes


catabolism?

o Aspartic acid is converted to oxaloacetate


thru non-oxidative deamination
o Serine is converted to pyruvate thru
oxidative deamination
o Decarboxylation converts glutamic acid to
GABA
o All choices

55. Which of the following hormones is not a


polypeptide? aldosterone is a steroid while all remaining are polypeptide

o ACTH
o FSH
o hCG
o Aldosterone

“Courage is found in unlikely places.”


― J.R.R. Tolkien 132
56. If one mole of Fructose-1,6-biphosphate
undergoes complete oxidation using the malate
shuttle, how many moles of ATP will be produced
(NET)?

o 22
o 28
o 30
o 34

PART 3: BASIC QUESTIONS

Vitamin C, also known as ascorbic acid, is required for the synthesis of


1. Regarding cutaneous wound healing which statement is collagen. It is also a highly effective antioxidant protecting cells from
correct: damage by free radicals. Studies have shown that the vitamin can help
speed the healing process of wounds.

Glucocorticoids (GCs) have been widely used as strong anti-inflammatory


A. Glucocorticoids promote collagen synthesis and reagents. However, despite their beneficial effects, side effects, such as
fibrosis of tissues impairing wound
B. Complications from wound contraction are most
commonly seen on the face and neck
C. Early movement of wounds can stimulate the process
of angiogenesis and healing
D. Vitamin c deficiency impairs collagen synthesis in
healing tissues

Metastatic calcification is caused by an increased calcium phosphate


2. Metastatic calcification: product in the blood, and may result from hypercalcemia or
hyperphosphatemia, or both. It is commonly associated with
A. Commonly develops in aging or damaged heart valves hyperparathyroidism, sarcoidosis, metastatic disease, and myeloma
B. Can occur with normal serum levels of calcium
C. Is encountered in areas of necrosis
D. Can be associated with metastatic malignancy

“Courage is found in unlikely places.”


― J.R.R. Tolkien 133
3. A substance that is not known to accumulate intracellularly
is:

A. Hemosiderin
B. Pus
C. Melanin
D. Tattoo ink

4. A 44 year old woman notes a lump in her left breast while Lymph node metastases. Axillary lymph node status is the most
taking a shower. The physician palpates a 3 cm firm, irregular, important prognostic factor for invasive carcinoma in the absence of
non-movable mass in the upper outer quadrant of her left distant metastases. The clinical assessment of lymph node status is
breast on physical examination. A fine needle aspiration of this unreliable due to both false positives (e.g., palpable reactive nodes) and
mass is performed, and cytologically the cells are consistent false negatives (e.g., lymph nodes with small metastatic
with infiltrating ductal carcinoma. The mass is removed with deposits). Therefore, biopsy is necessary for accurate assessment. With
lumpectomy along with an axillary lymph node dissection. no nodal involvement, the 10-year disease-free survival rate is close to
Which of the following findings will best predict a better 70% to 80%; the rate falls to 35% to 40% with one to three positive nodes,
prognosis for this patient? and to 10% to 15% when more than 10 nodes are positive. Lymphatic
vessels in most breast carcinomas drain first to one or two sentinel nodes,
which can be identified with radiotracer or colored dyes. If a biopsy
restricted to the sentinel nodes is negative for metastasis, it is unlikely
A. No metastases in the sampled lymph nodes that other more distant nodes will be involved and the patient can be
B. Tumor cells strongly estrogen receptor positive spared the morbidity of a complete axillary dissection.
C. One relative who had similar type of breast cancer
D. Flow cytometry analysis with aneuploidy and a high S-
phase Robbins and Cotran Pathologic Basis of Disease, p. 1066

5. Amoebic dysentery: · Diarrhea with belly cramps


· Fever
A. Odorless · Nausea and vomiting
B. Bright red blood on the stool · Blood or mucus in the diarrhea
C. Few pus cells on the stool
D. Non-adherent stool consistency

6. Biological vector: ● Phlebotomus - sandfly fever and leishmaniasis


-Phlebotomus (old world) and Lutzomyia (new world) sandfly bites
A. Phlebotomus transmit leishmaniasis to humans in most of the tropical and subtropical
B. Glossina world
C. Musca domestica ● Glossina - tsetse fly - trypanosomiasis
D. Anopheles ● Anopheles - malaria

7. In malaria, the infective stage to man from the insect ● The human-infective stage are sporozoites from the salivary
vector is: gland of a mosquito.
● The sporozoites grow and multiply in the liver to become
A. Gametocytes merozoites.
B. Schizonts ● These merozoites invade the erythrocytes (RBCs) to form

“Courage is found in unlikely places.”


― J.R.R. Tolkien 134
C. Sporozoites trophozoites, schizonts and gametocytes, during which the
D. Merozoites symptoms of malaria are produced.

8. In primary wound healing:

A. Neovascularization is maximal at day five - secondary


healing (4th day)
B. Macrophage infiltration occurs at 24 hours
C. Type I collagen is replaced by type III collagen
D. Wound strength is 25% of normal at the end of first
week

9. A substance that is not known to accumulate intracellularly


is:

A. Pus
B. Melanin
C. Hemosiderin
D. Tattoo ink

10. A host harboring sexually immature or larval stage of a 1. Definitive host:- Depending on the parasitic species, it is either
parasite and in which no fertilization takes place in it: a host which harbors the adult stage of a parasite or most highly
developed form of the parasite occurs; or sexually mature
A. Carrier host stages of a parasite and fertilization takes place in it, e.g., man
B. Intermediate host is the definitive host of Taenia saginata. When the mature or
C. Reservoir host most highly developed form is not obvious the definitive host is
D. Definitive host the mammalian host, e.g., human is the definitive host for
trypanosomes that cause African trypanosomiasis.
2. Intermediate host:- Is a host harboring sexually immature or
larval stage of a parasite and in which no fertilization takes
place in it. E.g. Cow is the intermediate host for Taenia saginata
Parasitology 14 Amplifier host- Intermediate hosts in which
parasites under go multiplication.
3. Reservoir host:- A wild or domestic animal which harbors a
parasite and acts as sources of infection to humans.
4. Carrier host:- A host harboring and disseminating a parasite but
exhibiting no clinical sign.
5. Accidental (Incidental) host:- Infection of a host other than the
normal host species. A parasite may or may not continue full
development in this host.

“Courage is found in unlikely places.”


― J.R.R. Tolkien 135
11. A host harboring and disseminating a parasite but (see above question)
exhibit g no clinical sign:

A. Reservoir host
B. Carrier host
C. Intermediate host
D. Definitive host

12. Regarding acute inflammation:


2 phase:
A. Increased permeability leads to protein depleted
plasma leaking into the tissue 1. Vascular Phase
B. Initial vasoconstriction is the result of histamine and
nitric oxide In the vascular phase, small blood vessels adjacent to the injury
C. Initial formation of endothelial gaps lasts for only 15-30 dilate (vasodilatation) and blood flow to the area increases. The
minutes endothelial cells initially swell, then contract to increase the space
D. Stasis occurs due to vasodilation and the larger caliber between them, thereby increasing the permeability of the vascular
of vessels barrier. This process is regulated by chemical mediators.

Exudation of fluid leads to a net loss of fluid from the vascular space
into the interstitial space, resulting in oedema (tumour).

The formation of increased tissue fluid acts as a medium for which


inflammatory proteins (such as complement and immunoglobulins)
can migrate through. It may also help to remove pathogens and cell
debris in the area through lymphatic drainage.

2. Cellular Phase

The predominant cell of acute inflammation is the neutrophil. They


are attracted to the site of injury by the presence of chemotaxins,
the mediators released into the blood immediately after the insult.

The migration of neutrophils occurs in four stages:


● Margination – cells line up against the endothelium
● Rolling – close contact with and roll along the endothelium
● Adhesion – connecting to the endothelial wall
● Emigration – cells move through the vessel wall to the affected
area

Once in the region, neutrophils recognise the foreign body and begin
phagocytosis, the process whereby the pathogen is engulfed and
contained with a phagosome. The phagosome is then destroyed via
oxygen-independent (e.g. lysozymes) or oxygen-dependent (e.g.
free radical formation) mechanisms.

13. All are true about karyotyping, EXCEPT: The study of chromosomes—karyotyping—is the basic tool of the
cytogeneticist. The usual procedure to examine chromosomes is to arrest
A. G-banding is most common staining method employed dividing cells in metaphase with mitotic spindle inhibitors (e.g., N-
using Giemsa diacetyl-N-methylcolchicine [Colcemid]) and then to stain the
B. Chromosomes are arranged in pairs and increasing chromosomes. In a metaphase spread, the individual chromosomes take

“Courage is found in unlikely places.”


― J.R.R. Tolkien 136
order of length the form of two chromatids connected at the centromere. A karyotype is
C. After staining, chromosomes are analyzed under a obtained by arranging each pair of autosomes according to length,
microscope and photographed followed
D. Dividing cells are arrested in metaphase by sex chromosomes. A variety of staining methods have been developed
that allow identification of individual chromosomes on the basis of a
distinctive and reliable pattern of alternating light and dark bands. The
one most commonly used involves a Giemsa stain and is hence called G
banding. A normal male karyotype with G banding is illustrated in Figure
5-17. With standard G banding, approximately 400 to 800 bands per
haploid set can be detected. The resolution obtained by banding can be
markedly improved by obtaining the cells in prophase. The individual
chromosomes appear markedly elongated, and as many as 1500 bands

Robbins and Cotran Pathologic Basis of Disease, p.158

14. Klinefelter’s syndrome is characterized by: Klinefelter syndrome can rarely be diagnosed before puberty, particularly
because the testicular abnormality does not develop before early
A. Webbed hands puberty. Most patients have a distinctive body habitus with an increase
B. Short stature in length between the soles and the pubic bone, which creates the
C. Cardiac defects appearance of an elongated body. Also characteristic are eunuchoid body
D. Subnormal intelligence habitus with abnormally long legs; small atrophic testes often associated
with a small penis; and lack of such secondary male characteristics as
deep voice, beard, and male distribution of pubic hair. Gynecomastia may
be present. The mean IQ is somewhat lower than normal, but
mental retardation is uncommon. There is increased incidence of type 2
diabetes and the metabolic syndrome that gives rise to insulin resistance.
Curiously, mitral valve prolapse is seen in about 50% of adults with
Klinefelter syndrome. There is also an increased incidence of
osteoporosis and fractures due to sex hormonal imbalance.

Robbins and Cotran Pathologic Basis of Disease, p.165

15. Myelogenous leukemias are caused by the cancerous Many diseases alter the production of blood cells. The marrow is the
production of innate (non-specific) immune system cells: in which ultimate source of most cells of the innate and adaptive immune system
tissue is such production most likely to occur? and responds to infectious or inflammatory challenges by increasing its
output of granulocytes under the direction of specific growth factors and
A. Thymus cytokines. By contrast, many other disorders are associated
B. Spleen with defects in hematopoiesis that lead to deficiencies of one or more
C. Lymph nodes types of blood cells. Primary tumors of hematopoietic cells are among the
D. Bone marrow most important diseases that interfere with marrow function, but certain
genetic diseases, infections, toxins, and nutritional deficiencies, as well as
chronic inflammation from any cause, can also decrease the production
of blood cells by the marrow.

Robbins and Cotran Pathologic Basis of Disease, p.581

16. Histamine is involved in acute inflammatory responses and is The richest sources of histamine are the mast cells that are normally
released from mast cells. Which of the following statements is present in the connective tissue adjacent to blood vessels. It is also found
incorrect? in blood basophils and platelets.

A. It causes increased permeability of arterioles Histamine is stored in mast cell granules and is released by mast cell
B. It causes constriction of large vessels degranulation in response to a variety of stimuli, including
C. It may be released by physical trauma/ (1) physical injury, such as trauma, cold, or heat, by unknown
D. It is found in blood basophils, platelets and mast cells mechanisms;
(2) binding of antibodies to mast cells, which
underlies immediate hypersensitivity (allergic) reactions

“Courage is found in unlikely places.”


― J.R.R. Tolkien 137
(3) products of complement called anaphylatoxins (C3a and C5a).

Histamine causes dilation of arterioles and increases the permeability of


venules. Histamine is considered to be the principal mediator of the
immediate transient phase of increased vascular permeability, producing
interendothelial gaps in venules, as discussed earlier. Its vasoactive
effects are mediated mainly via binding to receptors, called H1 receptors,
on microvascular endothelial cells.

Robbin’s 9th ed. Chapter 3 Acute Inflammation p 83

TNF regulates energy balance by promoting lipid and protein mobilization


17. Regarding mediators of inflammation: and by suppressing appetite. Therefore, sustained production of TNF
contributes to cachexia, a pathologic state characterized by weight loss
A. TNF contributes to cachexia of disease and anorexia that accompanies some chronic
B. TNF and IL-1 are produced mainly by activated infections and neoplastic diseases
leukocytes
C. TNF is a chemokine with chemoattractant properties TNF and IL-1 serve critical roles in leukocyte recruitment by promoting
D. The systemic acute phase response is induced by MIP-1 adhesion of leukocytes to endothelium and their migration through
and RANTES chemokineS vessels. These cytokines are produced mainly by activated macrophages
and dendritic cells; TNF is also produced by T lymphocytes and mast cells,
and IL-1 is produced by some epithelial cells as well.

TNF is a cytokine not a chemokine. Chemokines = chemoattractants

The C-C chemokines, which include


monocyte chemoattractant protein (MCP-1), eotaxin, macrophage
inflammatory protein­1α (MIP­1α), and RANTES (regulated and normal
T-cell expressed and secreted), generally attract monocytes, eosinophils,
basophils and lymphocytes, but are not as potent chemoattractants for
neutrophils.

Robbin’s 9th ed. Chapter 3 Acute Inflammation p 86

18. Leishmania species are transmitted by: Leishmania organisms occur in two forms: extracellular, flagellate
promastigotes (length, 10–20 μm) in the sandfly vector and intracellular,
A. Tsetse flies nonflagellate amastigotes (length, 2–4 μm; in vertebrate hosts, including
B. Sandflies humans.
C. Triatomid bugs

“Courage is found in unlikely places.”


― J.R.R. Tolkien 138
D. House flies Harrison’s 20th ed. p 1594

19. Transduction may occur due to the following, EXCEPT:


B. Inflammation
A. Lymphatic obstruction
B. Inflammation Reception: A cell detects a signaling molecule from the outside of the cell.
C. Increased intravascular hydrostatic pressure A signal is detected when the chemical signal (also known as a ligand)
D. Decreased intravascular oncotic pressure binds to a receptor protein on the surface of the cell or inside the cell.

2. Transduction: When the signaling molecule binds the receptor it


changes the receptor protein in some way. This change initiates the
process of transduction. Signal transduction is usually a pathway of
several steps. Each relay molecule in the signal transduction pathway
changes the next molecule in the pathway.

3. Response: Finally, the signal triggers a specific cellular response

20. Complement: Complement pathway is activated by C3 cleavage

A. Pathway is inhibited by C3-cleavage Complement proteins are present in inactive forms in the plasma, and
B. C3 is the most abundant protein in the complement many of them are activated to become proteolytic enzymes that degrade
family other complement proteins, thus forming an enzymatic cascade capable
C. Activation by the classical pathway involves microbial of tremendous amplification. The critical step in complement activation
surface antigens is the proteolysis of the third (and most abundant) component, C3.
D. Proteins are usually stored in an active state in Cleavage of C3 can occur by one of
lysosomal molecules three pathways: The classical, alternative and lectin pathway.

Pathway

1. Classical which is triggered by fixation of C1 to


antibody (IgM or IgG) that has
combined with antigen

2. which can be triggered by


Alternative microbial surface molecules (e.g.,
endotoxin, or LPS),
complex polysaccharides, cobra
venom, and other substances, in the
absence of antibody

3. Lectin plasma mannose-binding


lectin binds to Carbohydrates on
microbes and directly activates C1.

Robbin’s 9th ed. Chapter 3 Inflammation and Repair p 84

21. Which of the following changes is associated with irreversible Flocculent densities in the matrix of mitochondria have become quite
cell damage? important in cell pathology since, when prominent, they indicate
irreversible cell injury.
A. Cellular swelling

“Courage is found in unlikely places.”


― J.R.R. Tolkien 139
B. Loss of microvilli Collan Y, McDowell E, Trump BF. Studies on the pathogenesis of ischemic
C. Glycogen depletion cell injury. VI. Mitochondrial flocculent densities in autolysis. Virchows
D. Flocculent densities in mitochondria Arch B Cell Pathol Incl Mol Pathol. 1981;35(3):189-99. doi:
10.1007/BF02889159. PMID: 6116311.

22. Common risk factors for arterial thrombo-emboli include the


following, EXCEPT:

A. AMI
B. Rheumatic heart disease
C. Paradoxical embolism from venous thrombi
D. Cardiomyopathy

23. When the parasite benefited from the host Symbiosis is a close relationship between two species in which at least
while the host neither benefited or harmed: one species benefits.
Commensalism is a symbiotic relationship in which one species benefits
A. Symbiosis while the other species is not affected.
B. Commensalism Mutualism is a symbiotic relationship in which both species benefit.
C. Mutualism Parasitism is a symbiotic relationship in which one species (the parasite)
D. Parasitism benefits while the other species (the host) is harmed.

24. Monocytes move from the systemic circulatory system into Some monocytes and dendritic cells remain in the general blood
general connective tissues, where they differentiate into what circulation, but most of them enter body tissues. In tissues, monocytes
phagocytic cell type? develop into much larger phagocytic cells known as macrophages

A. T cell
B. B cell
C. Neutrophil
D. Macrophage

25.Regarding collagen: Ratio 22. Collagen consists of amino acids bound together to form a triple
helix of elongated fibril known as a collagen helix. It is mostly found in
A. Non-fibrillar collagen is the most ubiquitous protein in connective tissue such as cartilage, bones, tendons, ligaments, and skin.
the body
B. Collagen is formed in a triple helix structure
C. Collagen is formed in cells as long fibrils
D. Type V, non-fibrillar collagen, is seen in basement
membranes

“Courage is found in unlikely places.”


― J.R.R. Tolkien 140
26. A 48 year old woman has a routine physical examination. A 4
cm diameter non tender mass is palpated in her right breast. The
mass appears fixed to the chest wall. Another 2 cm non-tender
mass is palpable in the left axilla. A chest radiograph reveals
multiple 0.5 to 2 cm nodules in both lungs. Which of the following
TNM classification best indicated the stage of her disease?

A. T1 N0 M1
B. T2 N1 M0
C. T3 N0 M0
D. T4 N1 M1

27. Habitat of Giardia lamblia: Giardia duodenalis is a protozoan flagellate (Diplomonadida). This
protozoan was initially named Cercomonas intestinalis by Lambl in 1859.
A. Distal part of the large intestine It was renamed Giardia lamblia by Stiles in 1915 in honor of Professor A.
B. Liver and biliary tree Giard of Paris and Dr. F. Lambl of Prague. However, many consider the
C. Colon and rectum name, Giardia duodenalis, (Davaine 1875) to be the correct taxonomic
D. Mainly in the duodenum and jejunum name for this protozoan.

“Courage is found in unlikely places.”


― J.R.R. Tolkien 141
Ans. Doudenum

28. ANGEL’S MAN syndrome is due to: Angelman syndrome is a genetic disorder. It's usually caused by problems
with a gene located on chromosome 15 called the ubiquitin protein ligase
A. Inversion E3A (UBE3A) gene.
B. Uniparental disomy Angelman syndrome results when a person inherits two copies of
C. Mitochondrial disorder chromosome 15 from his or her father (paternal copies) instead of one
D. Digenic inheritance copy from each parent. This phenomenon is called paternal uniparental
disomy .
Most cases of Angelman syndrome are not inherited, particularly those
caused by a deletion in the maternal chromosome 15 or by paternal
uniparental disomy . These genetic changes occur as random events
during the formation of reproductive cells (eggs and sperm) or in early
embryonic development.

29. Apoptosis occurs in all BUT one of the following situations: In tumors there is loss of apoptotic events.

A. Tumors
B. Menstruation
C. Bacterial infection
D. Embryogenesis

30. Histamine exerts its effect during inflammation by: Histamine exerts its effects by binding to histamine receptors on cells'
surfaces. H1 receptors are more common.
A. Vasoconstriction of post capillary sphincters
B. Acting on H2 receptors on mast cells Chemical mediators producing endothelial contraction include:
C. Constriction of large arteries histamine, leukotrienes, bradykinin, platelet activating factor, and the
D. Causing venular endothelial contraction C3a and C5a components from complement activation. Mediators of this
process over a longer term include tumor necrosis factor and interleukin-
1.

31. Metastatic calcification:

A. Is encountered in areas of necrosis


B. Can be associated with metastatic malignancy
C. Can occur with normal serum levels of calcium

“Courage is found in unlikely places.”


― J.R.R. Tolkien 142
D. Commonly develops in aging or damaged heart valves

32. Concerning vascular leakage with acute inflammation:

A. Chemically mediated endothelial contraction is usually


reversible
B. Immediate sustained response is due to widening of
intercellular gaps
C. Endothelial cell contraction preferentially occurs in
arterioles 20-60um in diameter
D. Tumor necrosis factor (TNF) affects leakage by direct
endothelial cell necrosis

33. In the mononuclear phagocyte system: ● Macrophages have a long life span, ranging from months to
years
A. The life span of tissue macrophages is about three days ● Kupffer cells ( aka stellate sinusoidal macrophages or Kupffer-
B. Aggregation of macrophages in the kidney are known as Browicz cells) are macrophages found in the sinusoids of the
Kupffer cells liver.
C. The half-life of blood monocytes is about one month ● The life span of a circulating monocyte is fairly brief and most
D. Chemotactic stimuli for monocytes include C5a and undergo apoptosis after about 24h
certain growth factors

34. The parasite derives food and protection from the host Commensalism does not involve physiologic interaction or dependency
without causing harm to the host: between the two partners. Literally, the term means “eating at the same
table”. In other words, commensalism is a type of symbiosis in which
A. Opportunistic parasite spatial proximity allows the commensal to feed on substances captured
B. Commensal and opportunistic parasite or ingested by the host.
C. Commensal
D. Pathogenic parasite

35. The following are all actions of bradykinin, EXCEPT:

A. Dilation of blood vessels


B. Increased vascular permeability
C. Smooth muscle contraction
D. Activation of complement pathway

36. Type of specimen used for parasitological examination of Microscopic examination of sputum is used in identifying Paragonimus
Paragonimus westermani: westermani eggs, Strongyloides stercolaris larvae, Ascaris lumbricoides
larvae, hookworm larvae, and rarely Entamoeba histolytica. Sputum
A. Sputum should be obtained from the lower respiratory passages rather than a
B. Bone marrow aspirate sample consisting of saliva.
C. Rectal scrapin
D. Stool

“Courage is found in unlikely places.”


― J.R.R. Tolkien 143
37. Most common cause of calcium oxalate stone formation:

A. Hyperuricosuria
B. Hyperoxaluria
C. Hypercalciuria without hypercalcemia
D. Hypercalcemia and hypercalciuria

38. A diabetic patient has an ulcer on their great toe. Which


statement is false regarding it’s healing?

A. It will heal by secondary intention


B. The amount of acute inflammation is small, slowing the
healing
C. Wound contraction required actin containing
fibroblasts
D. Substantial scar formation is likely

39. Parasites that can complete its life cycle in a single host: Life cycles of parasites can be further divided into two categories: direct
(monoxenous) and indirect (heteroxenous). Direct life cycles spend most
A. Taenia of their adult lives in one host, known as the parasitic stage, with their
B. Leishmania progeny transmitted from one host to another, known as the free-living
C. Plasmodium stage.
D. Giardia

40. The route of infection is through inhalation of contaminated strongyloides: MOT through contact with soil that is contaminated with
air: free living larvae
Trichomonas vaginalis:sexual contact
A. Strongyloides stercoralis Toxoplasmosis: contaminated food, zoonotic, mother to child
B. Trichomonas vaginalis (congenital)
C. Toxoplasma gondii Enterobius vermicularis: fecal oral, inhalation
D. Enterobius vermicularis

41. Which of the following regarding arterial thrombi is TRUE? mural thrombi most commonly located in descending aorta
Break off and embolize or result in total occlusion of VEIN not artery
A. Mural thrombi is most commonly found on the femoral
artery
B. The tail of thrombi often breaks off and embolized to

“Courage is found in unlikely places.”


― J.R.R. Tolkien 144
distal sites
C. Most common sites are cerebral, coronary and femoral
arteries
D. It typically creates a long cast of vessel lumen with a
propagational long tail

42. Regarding cutaneous wound healing which statement is


correct?

A. Vitamin C deficiency impairs collagen synthesis in


healing tissues
B. Early movement of wounds can stimulate the process of
angiogenesis and healing
C. Glucocorticoids promote collagen synthesis and fibrosis
of tissues
D. Complication from wound contraction are most
commonly seen on the face and neck

Schistosomiasis is a tropical disease caused by a parasite. The parasite is


43. Source of exposure to cercarial signs of human blood fluke: transmitted through contact with fresh water contaminated with the
parasite’s larvae

A. Contaminated water People become infected with schistosomes if they come into contact with
B. Contaminated soil the larval? forms of the parasite, for example, when washing or playing
C. Blood sucking arthropods in contaminated fresh water.
D. Insufficiently cooked meat of pork or beef

44. Which of the following is the least important risk factor for the
development of thrombus?

A. Acute myocardial infarction


B. Smoking
C. Antithrombin III deficiency
D. Disseminated malignancy

“Courage is found in unlikely places.”


― J.R.R. Tolkien 145
45. A 15 year old healthy girl with no major medical problems
notes blotchy areas of erythema that are pruritic over the skin of
her arms, legs and trunk within an hour every time she eats
seafood, followed by diarrhea. These problems abate within 3
hours, and then physical examination reveals no abnormal
findings. Which of the following immunologic abnormalities is she
most likely to have?

A. Complement activation
B. Hypergammaglobulinemia
C. Cell-mediated hypersensitivity
D. Localized anaphylaxis

46 Permanent association between two different organisms so


dependent on each other that their life part is impossible:

A. Symbiosis
B. Parasitism
C. Commensalism
D. Mutualism

Excess circulating thrombin results from the excess activation of the


47. Which of the following statement regarding disseminated coagulation cascade. The excess thrombin cleaves fibrinogen, which
intravascular coagulation is FALSE? ultimately leaves behind multiple fibrin clots in the circulation. These
excess clots trap platelets to become larger clots, which leads to
A. Bleeding diathesis is solely due to thrombocytopenia?? microvascular and macrovascular thrombosis. This lodging of clots in the
Not sure microcirculation, in the large vessels, and in the organs is what leads to
B. Rapid consumption of platelets, prothrombin and the ischemia, impaired organ perfusion, and end-organ damage that
fibrinogen occurs occurs with DIC.[citation needed]
C. Fibrinogen degradation products (FDPs) increases due
to rapid fibrinolysis Coagulation inhibitors are also consumed in this process. Decreased
D. Multiple thrombi develop in the microcirculation inhibitor levels will permit more clotting so that a positive feedback loop
develops in which increased clotting leads to more clotting. At the same
time, thrombocytopenia occurs and this has been attributed to the
entrapment and consumption of platelets. Clotting factors are consumed
in the development of multiple clots, which contributes to the bleeding
seen with DIC.[citation needed]
Simultaneously, excess circulating thrombin assists in the conversion of
plasminogen to plasmin, resulting in fibrinolysis. The breakdown of clots
results in an excess of FDPs, which have powerful anticoagulant
properties, contributing to hemorrhage. The excess plasmin also
activates the complement and kinin systems. Activation of these systems
leads to many of the clinical symptoms that patients experiencing DIC
exhibit, such as shock, hypotension, and increased vascular permeability.
The acute form of DIC is considered an extreme expression of the

“Courage is found in unlikely places.”


― J.R.R. Tolkien 146
intravascular coagulation process with a complete breakdown of the
normal homeostatic boundaries. DIC is associated with a poor prognosis
and a high mortality rate

48. Coarctation of the aorta is common in which syndrome:


Turner’s- Heart abnormalities are common in individuals with Turner.
A. Turner’s should be
B. Noonan’s monitored for problems with their Aorta and high blood pressure.
C. Down’s
D. Klinefelter’s Coarctation of the aorta- Aortic narrowing near insertion of ductus
arteriousus (juxtaductal). Associated with bicuspid aortic valve, other
heart defects, and Turner syndrome.

USMLE Step 1 2018, Cardio-Pathology, Page 294, Section III.

49. With regard to mechanisms of cellular adaptation, which of


the following statements is TRUE? Barrett’s esophagus- consists of change in the normally Squamous lining
of the lower esophagus to Columnar epithelium (metaplasia). Unless
A. TNF always acts as an inhibitor of hyperplasia there is severe esophagitis, this change can be recognized during an
B. Barrett’s metaplasia is squamous to columnar endoscopy.
epithelium in the distal esophagus
C. Hyperplasia refers to an increase in cell size WebMD
D. The myocardium adapts to increased workload by
hypertrophy and hyperplasia Metaplasia- Reprogramming of Stem cells, replacement of one cell type
by another that can adapt to a new stress. Usually due to an irritant, such
as gastric acid from GERD that will lead to ( Barretts esophagus).

USMLE Step 1 2018, Patho- Cellular Injury, Page 206, Section II.

50. Type of specimen used in parasitological examination of


Naegleria fowleri: CSF- Naegleriafowleri infection. can be diagnosed in the laboratory by
detecting
A. Urine N. fowleri organism in CSF thru Lumbar puncture, biopsy or tissue
B. Cerebrospinal fluid specimens.
C. Perianal swab
D. Liver aspirate www.cdc.gov/ diagnosis-hcp

CSF

USMLE Step 1 2018, Micro/para page 156, Section II

51. A 51 year old man has worked for 10 years in a factory


producing plastic pipe but not following safety standards. He has Vinyl Chloride- Exposure is associated with an increased risk of a rare
noted weight loss, nausea, and vomiting worsening over the past form of liver CA (Hepatic Angiosarcoma). As well as Brain and Lung CA,
5 months. On examination, he is afebrile. There is generalized Lymphoma and Leukemia.
muscle wasting. Laboratory studies show the serum alkaline
phosphatase is 405 U/L with AST 47 U/L. ALT 35 U/L, and total Hepatic angiosarcoma- malignant tumor of endothelial origin; associated
bilirubin 1.2 mg/dL. An abdominal CT scan reveals a 12 cm right with exposure to arsenic and Vinyl Chloride.
liver lobe mass. Liver biopsy reveals a neoplasm composed of
spindle cells forming irregular vascular channels. With USMLE Step 1 2018, GIT- Patho Page 386, Section III.
immunohistochemical staining the cells demonstrate vimentin
positivity and cytokeratin negativity. Exposure to which of the

“Courage is found in unlikely places.”


― J.R.R. Tolkien 147
following substances most likely led to development of this
neoplasm

A. • Cyclophosphamide
B. • Benzene
C. • Radon
D. • Vinyl chloride

52. Review of a series of surgical pathology reports indicates that If a grading system for a tumor type is not specified, the following system
a certain type of neoplasm is diagnosed as grade I on a scale of I is generally used (1):
to IV. Clinically, some of the patients with this neoplasm are found
to have stage I disease. Which of the following is the best GX: Grade cannot be assessed (undetermined grade)
interpretation of a neoplasm with these designation? G1: Well differentiated (low grade)
G2: Moderately differentiated (intermediate grade)
A. • Arising from epithelium G3: Poorly differentiated (high grade)
B. • Well-differentiated and localized G4: Undifferentiated (high grade)
C. • May spread via lymphatics and bloodstream
D. • Unlikely to be malignant National Cancer Institute/ USA.gov

53. Most common site of lodgment of arterial thromboemboli Arterial emboli commonly affect the brain and extremities.
is: Less frequently they can wreak havoc in mesenteric and renal
arteries.
A. • Kidney
B. • Brain https://www.ncbi.nlm.nih.gov/pmc/articles
C. • Lower limb
D. • Upper limb

PART 3: BASIC QUESTIONS

Vitamin C, also known as ascorbic acid, is required for the synthesis of


1. Regarding cutaneous wound healing which statement is collagen. It is also a highly effective antioxidant protecting cells from
correct: damage by free radicals. Studies have shown that the vitamin can help
speed the healing process of wounds.

Glucocorticoids (GCs) have been widely used as strong anti-inflammatory


A. Glucocorticoids promote collagen synthesis and reagents. However, despite their beneficial effects, side effects, such as
fibrosis of tissues impairing wound
B. Complications from wound contraction are most
commonly seen on the face and neck
C. Early movement of wounds can stimulate the process
of angiogenesis and healing
D. Vitamin c deficiency impairs collagen synthesis in
healing tissues

Metastatic calcification is caused by an increased calcium phosphate


2. Metastatic calcification: product in the blood, and may result from hypercalcemia or
hyperphosphatemia, or both. It is commonly associated with
A. Commonly develops in aging or damaged heart valves hyperparathyroidism, sarcoidosis, metastatic disease, and myeloma
B. Can occur with normal serum levels of calcium
C. Is encountered in areas of necrosis
D. Can be associated with metastatic malignancy

“Courage is found in unlikely places.”


― J.R.R. Tolkien 148
3. A substance that is not known to accumulate intracellularly
is:

A. Hemosiderin
B. Pus
C. Melanin
D. Tattoo ink

4. A 44 year old woman notes a lump in her left breast while Lymph node metastases. Axillary lymph node status is the most
taking a shower. The physician palpates a 3 cm firm, irregular, important prognostic factor for invasive carcinoma in the absence of
non-movable mass in the upper outer quadrant of her left distant metastases. The clinical assessment of lymph node status is
breast on physical examination. A fine needle aspiration of this unreliable due to both false positives (e.g., palpable reactive nodes) and
mass is performed, and cytologically the cells are consistent false negatives (e.g., lymph nodes with small metastatic
with infiltrating ductal carcinoma. The mass is removed with deposits). Therefore, biopsy is necessary for accurate assessment. With
lumpectomy along with an axillary lymph node dissection. no nodal involvement, the 10-year disease-free survival rate is close to
Which of the following findings will best predict a better 70% to 80%; the rate falls to 35% to 40% with one to three positive nodes,
prognosis for this patient? and to 10% to 15% when more than 10 nodes are positive. Lymphatic
vessels in most breast carcinomas drain first to one or two sentinel nodes,
which can be identified with radiotracer or colored dyes. If a biopsy
restricted to the sentinel nodes is negative for metastasis, it is unlikely
A. No metastases in the sampled lymph nodes that other more distant nodes will be involved and the patient can be
B. Tumor cells strongly estrogen receptor positive spared the morbidity of a complete axillary dissection.
C. One relative who had similar type of breast cancer
D. Flow cytometry analysis with aneuploidy and a high S-
phase Robbins and Cotran Pathologic Basis of Disease, p. 1066

5. Amoebic dysentery: · Diarrhea with belly cramps


· Fever
A. Odorless · Nausea and vomiting
B. Bright red blood on the stool · Blood or mucus in the diarrhea
C. Few pus cells on the stool
D. Non-adherent stool consistency

6. Biological vector: ● Phlebotomus - sandfly fever and leishmaniasis


-Phlebotomus (old world) and Lutzomyia (new world) sandfly bites
A. Phlebotomus transmit leishmaniasis to humans in most of the tropical and subtropical
B. Glossina world
C. Musca domestica ● Glossina - tsetse fly - trypanosomiasis
D. Anopheles ● Anopheles - malaria

7. In malaria, the infective stage to man from the insect ● The human-infective stage are sporozoites from the salivary
vector is: gland of a mosquito.
● The sporozoites grow and multiply in the liver to become
A. Gametocytes merozoites.
B. Schizonts ● These merozoites invade the erythrocytes (RBCs) to form

“Courage is found in unlikely places.”


― J.R.R. Tolkien 149
C. Sporozoites trophozoites, schizonts and gametocytes, during which the
D. Merozoites symptoms of malaria are produced.

8. In primary wound healing:

A. Neovascularization is maximal at day five - secondary


healing (4th day)
B. Macrophage infiltration occurs at 24 hours
C. Type I collagen is replaced by type III collagen
D. Wound strength is 25% of normal at the end of first
week

9. A substance that is not known to accumulate intracellularly


is:

A. Pus
B. Melanin
C. Hemosiderin
D. Tattoo ink

10. A host harboring sexually immature or larval stage of a 1. Definitive host:- Depending on the parasitic species, it is either
parasite and in which no fertilization takes place in it: a host which harbors the adult stage of a parasite or most highly
developed form of the parasite occurs; or sexually mature
A. Carrier host stages of a parasite and fertilization takes place in it, e.g., man
B. Intermediate host is the definitive host of Taenia saginata. When the mature or
C. Reservoir host most highly developed form is not obvious the definitive host is
D. Definitive host the mammalian host, e.g., human is the definitive host for
trypanosomes that cause African trypanosomiasis.
2. Intermediate host:- Is a host harboring sexually immature or
larval stage of a parasite and in which no fertilization takes
place in it. E.g. Cow is the intermediate host for Taenia saginata
Parasitology 14 Amplifier host- Intermediate hosts in which
parasites under go multiplication.
3. Reservoir host:- A wild or domestic animal which harbors a
parasite and acts as sources of infection to humans.
4. Carrier host:- A host harboring and disseminating a parasite but
exhibiting no clinical sign.
5. Accidental (Incidental) host:- Infection of a host other than the
normal host species. A parasite may or may not continue full
development in this host.

“Courage is found in unlikely places.”


― J.R.R. Tolkien 150
11. A host harboring and disseminating a parasite but (see above question)
exhibit g no clinical sign:

A. Reservoir host
B. Carrier host
C. Intermediate host
D. Definitive host

12. Regarding acute inflammation:


2 phase:
A. Increased permeability leads to protein depleted
plasma leaking into the tissue 1. Vascular Phase
B. Initial vasoconstriction is the result of histamine and
nitric oxide In the vascular phase, small blood vessels adjacent to the injury
C. Initial formation of endothelial gaps lasts for only 15-30 dilate (vasodilatation) and blood flow to the area increases. The
minutes endothelial cells initially swell, then contract to increase the space
D. Stasis occurs due to vasodilation and the larger caliber between them, thereby increasing the permeability of the vascular
of vessels barrier. This process is regulated by chemical mediators.

Exudation of fluid leads to a net loss of fluid from the vascular space
into the interstitial space, resulting in oedema (tumour).

The formation of increased tissue fluid acts as a medium for which


inflammatory proteins (such as complement and immunoglobulins)
can migrate through. It may also help to remove pathogens and cell
debris in the area through lymphatic drainage.

2. Cellular Phase

The predominant cell of acute inflammation is the neutrophil. They


are attracted to the site of injury by the presence of chemotaxins,
the mediators released into the blood immediately after the insult.

The migration of neutrophils occurs in four stages:


● Margination – cells line up against the endothelium
● Rolling – close contact with and roll along the endothelium
● Adhesion – connecting to the endothelial wall
● Emigration – cells move through the vessel wall to the affected
area

Once in the region, neutrophils recognise the foreign body and begin
phagocytosis, the process whereby the pathogen is engulfed and
contained with a phagosome. The phagosome is then destroyed via
oxygen-independent (e.g. lysozymes) or oxygen-dependent (e.g.
free radical formation) mechanisms.

13. All are true about karyotyping, EXCEPT: The study of chromosomes—karyotyping—is the basic tool of the
cytogeneticist. The usual procedure to examine chromosomes is to arrest
A. G-banding is most common staining method employed dividing cells in metaphase with mitotic spindle inhibitors (e.g., N-
using Giemsa diacetyl-N-methylcolchicine [Colcemid]) and then to stain the
B. Chromosomes are arranged in pairs and increasing chromosomes. In a metaphase spread, the individual chromosomes take

“Courage is found in unlikely places.”


― J.R.R. Tolkien 151
order of length the form of two chromatids connected at the centromere. A karyotype is
C. After staining, chromosomes are analyzed under a obtained by arranging each pair of autosomes according to length,
microscope and photographed followed
D. Dividing cells are arrested in metaphase by sex chromosomes. A variety of staining methods have been developed
that allow identification of individual chromosomes on the basis of a
distinctive and reliable pattern of alternating light and dark bands. The
one most commonly used involves a Giemsa stain and is hence called G
banding. A normal male karyotype with G banding is illustrated in Figure
5-17. With standard G banding, approximately 400 to 800 bands per
haploid set can be detected. The resolution obtained by banding can be
markedly improved by obtaining the cells in prophase. The individual
chromosomes appear markedly elongated, and as many as 1500 bands

Robbins and Cotran Pathologic Basis of Disease, p.158

14. Klinefelter’s syndrome is characterized by: Klinefelter syndrome can rarely be diagnosed before puberty, particularly
because the testicular abnormality does not develop before early
A. Webbed hands puberty. Most patients have a distinctive body habitus with an increase
B. Short stature in length between the soles and the pubic bone, which creates the
C. Cardiac defects appearance of an elongated body. Also characteristic are eunuchoid body
D. Subnormal intelligence habitus with abnormally long legs; small atrophic testes often associated
with a small penis; and lack of such secondary male characteristics as
deep voice, beard, and male distribution of pubic hair. Gynecomastia may
be present. The mean IQ is somewhat lower than normal, but
mental retardation is uncommon. There is increased incidence of type 2
diabetes and the metabolic syndrome that gives rise to insulin resistance.
Curiously, mitral valve prolapse is seen in about 50% of adults with
Klinefelter syndrome. There is also an increased incidence of
osteoporosis and fractures due to sex hormonal imbalance.

Robbins and Cotran Pathologic Basis of Disease, p.165

15. Myelogenous leukemias are caused by the cancerous Many diseases alter the production of blood cells. The marrow is the
production of innate (non-specific) immune system cells: in which ultimate source of most cells of the innate and adaptive immune system
tissue is such production most likely to occur? and responds to infectious or inflammatory challenges by increasing its
output of granulocytes under the direction of specific growth factors and
A. Thymus cytokines. By contrast, many other disorders are associated
B. Spleen with defects in hematopoiesis that lead to deficiencies of one or more
C. Lymph nodes types of blood cells. Primary tumors of hematopoietic cells are among the
D. Bone marrow most important diseases that interfere with marrow function, but certain
genetic diseases, infections, toxins, and nutritional deficiencies, as well as
chronic inflammation from any cause, can also decrease the production
of blood cells by the marrow.

Robbins and Cotran Pathologic Basis of Disease, p.581

16. Histamine is involved in acute inflammatory responses and is The richest sources of histamine are the mast cells that are normally
released from mast cells. Which of the following statements is present in the connective tissue adjacent to blood vessels. It is also found
incorrect? in blood basophils and platelets.

A. It causes increased permeability of arterioles Histamine is stored in mast cell granules and is released by mast cell
B. It causes constriction of large vessels degranulation in response to a variety of stimuli, including
C. It may be released by physical trauma/ (1) physical injury, such as trauma, cold, or heat, by unknown
D. It is found in blood basophils, platelets and mast cells mechanisms;
(2) binding of antibodies to mast cells, which
underlies immediate hypersensitivity (allergic) reactions

“Courage is found in unlikely places.”


― J.R.R. Tolkien 152
(3) products of complement called anaphylatoxins (C3a and C5a).

Histamine causes dilation of arterioles and increases the permeability of


venules. Histamine is considered to be the principal mediator of the
immediate transient phase of increased vascular permeability, producing
interendothelial gaps in venules, as discussed earlier. Its vasoactive
effects are mediated mainly via binding to receptors, called H1 receptors,
on microvascular endothelial cells.

Robbin’s 9th ed. Chapter 3 Acute Inflammation p 83

TNF regulates energy balance by promoting lipid and protein mobilization


17. Regarding mediators of inflammation: and by suppressing appetite. Therefore, sustained production of TNF
contributes to cachexia, a pathologic state characterized by weight loss
A. TNF contributes to cachexia of disease and anorexia that accompanies some chronic
B. TNF and IL-1 are produced mainly by activated infections and neoplastic diseases
leukocytes
C. TNF is a chemokine with chemoattractant properties TNF and IL-1 serve critical roles in leukocyte recruitment by promoting
D. The systemic acute phase response is induced by MIP-1 adhesion of leukocytes to endothelium and their migration through
and RANTES chemokineS vessels. These cytokines are produced mainly by activated macrophages
and dendritic cells; TNF is also produced by T lymphocytes and mast cells,
and IL-1 is produced by some epithelial cells as well.

TNF is a cytokine not a chemokine. Chemokines = chemoattractants

The C-C chemokines, which include


monocyte chemoattractant protein (MCP-1), eotaxin, macrophage
inflammatory protein­1α (MIP­1α), and RANTES (regulated and normal
T-cell expressed and secreted), generally attract monocytes, eosinophils,
basophils and lymphocytes, but are not as potent chemoattractants for
neutrophils.

Robbin’s 9th ed. Chapter 3 Acute Inflammation p 86

18. Leishmania species are transmitted by: Leishmania organisms occur in two forms: extracellular, flagellate
promastigotes (length, 10–20 μm) in the sandfly vector and intracellular,
A. Tsetse flies nonflagellate amastigotes (length, 2–4 μm; in vertebrate hosts, including
B. Sandflies humans.
C. Triatomid bugs

“Courage is found in unlikely places.”


― J.R.R. Tolkien 153
D. House flies Harrison’s 20th ed. p 1594

19. Transduction may occur due to the following, EXCEPT:


B. Inflammation
A. Lymphatic obstruction
B. Inflammation Reception: A cell detects a signaling molecule from the outside of the cell.
C. Increased intravascular hydrostatic pressure A signal is detected when the chemical signal (also known as a ligand)
D. Decreased intravascular oncotic pressure binds to a receptor protein on the surface of the cell or inside the cell.

2. Transduction: When the signaling molecule binds the receptor it


changes the receptor protein in some way. This change initiates the
process of transduction. Signal transduction is usually a pathway of
several steps. Each relay molecule in the signal transduction pathway
changes the next molecule in the pathway.

3. Response: Finally, the signal triggers a specific cellular response

20. Complement: Complement pathway is activated by C3 cleavage

A. Pathway is inhibited by C3-cleavage Complement proteins are present in inactive forms in the plasma, and
B. C3 is the most abundant protein in the complement many of them are activated to become proteolytic enzymes that degrade
family other complement proteins, thus forming an enzymatic cascade capable
C. Activation by the classical pathway involves microbial of tremendous amplification. The critical step in complement activation
surface antigens is the proteolysis of the third (and most abundant) component, C3.
D. Proteins are usually stored in an active state in Cleavage of C3 can occur by one of
lysosomal molecules three pathways: The classical, alternative and lectin pathway.

Pathway

2. Classical which is triggered by fixation of C1 to


antibody (IgM or IgG) that has
combined with antigen

2. which can be triggered by


Alternative microbial surface molecules (e.g.,
endotoxin, or LPS),
complex polysaccharides, cobra
venom, and other substances, in the
absence of antibody

3. Lectin plasma mannose-binding


lectin binds to Carbohydrates on
microbes and directly activates C1.

Robbin’s 9th ed. Chapter 3 Inflammation and Repair p 84

21. Which of the following changes is associated with irreversible Flocculent densities in the matrix of mitochondria have become quite
cell damage? important in cell pathology since, when prominent, they indicate
irreversible cell injury.
A. Cellular swelling

“Courage is found in unlikely places.”


― J.R.R. Tolkien 154
B. Loss of microvilli Collan Y, McDowell E, Trump BF. Studies on the pathogenesis of ischemic
C. Glycogen depletion cell injury. VI. Mitochondrial flocculent densities in autolysis. Virchows
D. Flocculent densities in mitochondria Arch B Cell Pathol Incl Mol Pathol. 1981;35(3):189-99. doi:
10.1007/BF02889159. PMID: 6116311.

22. Common risk factors for arterial thrombo-emboli include the


following, EXCEPT:

A. AMI
B. Rheumatic heart disease
C. Paradoxical embolism from venous thrombi
D. Cardiomyopathy

23. When the parasite benefited from the host Symbiosis is a close relationship between two species in which at least
while the host neither benefited or harmed: one species benefits.
Commensalism is a symbiotic relationship in which one species benefits
A. Symbiosis while the other species is not affected.
B. Commensalism Mutualism is a symbiotic relationship in which both species benefit.
C. Mutualism Parasitism is a symbiotic relationship in which one species (the parasite)
D. Parasitism benefits while the other species (the host) is harmed.

24. Monocytes move from the systemic circulatory system into Some monocytes and dendritic cells remain in the general blood
general connective tissues, where they differentiate into what circulation, but most of them enter body tissues. In tissues, monocytes
phagocytic cell type? develop into much larger phagocytic cells known as macrophages

A. T cell
B. B cell
C. Neutrophil
D. Macrophage

25.Regarding collagen: Ratio 22. Collagen consists of amino acids bound together to form a triple
helix of elongated fibril known as a collagen helix. It is mostly found in
A. Non-fibrillar collagen is the most ubiquitous protein in connective tissue such as cartilage, bones, tendons, ligaments, and skin.
the body
B. Collagen is formed in a triple helix structure
C. Collagen is formed in cells as long fibrils
D. Type V, non-fibrillar collagen, is seen in basement
membranes

“Courage is found in unlikely places.”


― J.R.R. Tolkien 155
26. A 48 year old woman has a routine physical examination. A 4
cm diameter non tender mass is palpated in her right breast. The
mass appears fixed to the chest wall. Another 2 cm non-tender
mass is palpable in the left axilla. A chest radiograph reveals
multiple 0.5 to 2 cm nodules in both lungs. Which of the following
TNM classification best indicated the stage of her disease?

A. T1 N0 M1
B. T2 N1 M0
C. T3 N0 M0
D. T4 N1 M1

27. Habitat of Giardia lamblia: Giardia duodenalis is a protozoan flagellate (Diplomonadida). This
protozoan was initially named Cercomonas intestinalis by Lambl in 1859.
A. Distal part of the large intestine It was renamed Giardia lamblia by Stiles in 1915 in honor of Professor A.
B. Liver and biliary tree Giard of Paris and Dr. F. Lambl of Prague. However, many consider the
C. Colon and rectum name, Giardia duodenalis, (Davaine 1875) to be the correct taxonomic
D. Mainly in the duodenum and jejunum name for this protozoan.

“Courage is found in unlikely places.”


― J.R.R. Tolkien 156
Ans. Doudenum

28. ANGEL’S MAN syndrome is due to: Angelman syndrome is a genetic disorder. It's usually caused by problems
with a gene located on chromosome 15 called the ubiquitin protein ligase
A. Inversion E3A (UBE3A) gene.
B. Uniparental disomy Angelman syndrome results when a person inherits two copies of
C. Mitochondrial disorder chromosome 15 from his or her father (paternal copies) instead of one
D. Digenic inheritance copy from each parent. This phenomenon is called paternal uniparental
disomy .
Most cases of Angelman syndrome are not inherited, particularly those
caused by a deletion in the maternal chromosome 15 or by paternal
uniparental disomy . These genetic changes occur as random events
during the formation of reproductive cells (eggs and sperm) or in early
embryonic development.

29. Apoptosis occurs in all BUT one of the following situations: In tumors there is loss of apoptotic events.

A. Tumors
B. Menstruation
C. Bacterial infection
D. Embryogenesis

30. Histamine exerts its effect during inflammation by: Histamine exerts its effects by binding to histamine receptors on cells'
surfaces. H1 receptors are more common.
A. Vasoconstriction of post capillary sphincters
B. Acting on H2 receptors on mast cells Chemical mediators producing endothelial contraction include:
C. Constriction of large arteries histamine, leukotrienes, bradykinin, platelet activating factor, and the
D. Causing venular endothelial contraction C3a and C5a components from complement activation. Mediators of this
process over a longer term include tumor necrosis factor and interleukin-
1.

31. Metastatic calcification:

A. Is encountered in areas of necrosis


B. Can be associated with metastatic malignancy
C. Can occur with normal serum levels of calcium

“Courage is found in unlikely places.”


― J.R.R. Tolkien 157
D. Commonly develops in aging or damaged heart valves

32. Concerning vascular leakage with acute inflammation:

A. Chemically mediated endothelial contraction is usually


reversible
B. Immediate sustained response is due to widening of
intercellular gaps
C. Endothelial cell contraction preferentially occurs in
arterioles 20-60um in diameter
D. Tumor necrosis factor (TNF) affects leakage by direct
endothelial cell necrosis

33. In the mononuclear phagocyte system: ● Macrophages have a long life span, ranging from months to
years
A. The life span of tissue macrophages is about three days ● Kupffer cells ( aka stellate sinusoidal macrophages or Kupffer-
B. Aggregation of macrophages in the kidney are known as Browicz cells) are macrophages found in the sinusoids of the
Kupffer cells liver.
C. The half-life of blood monocytes is about one month ● The life span of a circulating monocyte is fairly brief and most
D. Chemotactic stimuli for monocytes include C5a and undergo apoptosis after about 24h
certain growth factors

34. The parasite derives food and protection from the host Commensalism does not involve physiologic interaction or dependency
without causing harm to the host: between the two partners. Literally, the term means “eating at the same
table”. In other words, commensalism is a type of symbiosis in which
A. Opportunistic parasite spatial proximity allows the commensal to feed on substances captured
B. Commensal and opportunistic parasite or ingested by the host.
C. Commensal
D. Pathogenic parasite

35. The following are all actions of bradykinin, EXCEPT:

A. Dilation of blood vessels


B. Increased vascular permeability
C. Smooth muscle contraction
D. Activation of complement pathway

36. Type of specimen used for parasitological examination of Microscopic examination of sputum is used in identifying Paragonimus
Paragonimus westermani: westermani eggs, Strongyloides stercolaris larvae, Ascaris lumbricoides
larvae, hookworm larvae, and rarely Entamoeba histolytica. Sputum
A. Sputum should be obtained from the lower respiratory passages rather than a
B. Bone marrow aspirate sample consisting of saliva.
C. Rectal scrapin
D. Stool

“Courage is found in unlikely places.”


― J.R.R. Tolkien 158
37. Most common cause of calcium oxalate stone formation:

A. Hyperuricosuria
B. Hyperoxaluria
C. Hypercalciuria without hypercalcemia
D. Hypercalcemia and hypercalciuria

38. A diabetic patient has an ulcer on their great toe. Which


statement is false regarding it’s healing?

A. It will heal by secondary intention


B. The amount of acute inflammation is small, slowing the
healing
C. Wound contraction required actin containing
fibroblasts
D. Substantial scar formation is likely

39. Parasites that can complete its life cycle in a single host: Life cycles of parasites can be further divided into two categories: direct
(monoxenous) and indirect (heteroxenous). Direct life cycles spend most
A. Taenia of their adult lives in one host, known as the parasitic stage, with their
B. Leishmania progeny transmitted from one host to another, known as the free-living
C. Plasmodium stage.
D. Giardia

40. The route of infection is through inhalation of contaminated strongyloides: MOT through contact with soil that is contaminated with
air: free living larvae
Trichomonas vaginalis:sexual contact
A. Strongyloides stercoralis Toxoplasmosis: contaminated food, zoonotic, mother to child
B. Trichomonas vaginalis (congenital)
C. Toxoplasma gondii Enterobius vermicularis: fecal oral, inhalation
D. Enterobius vermicularis

41. Which of the following regarding arterial thrombi is TRUE? mural thrombi most commonly located in descending aorta
Break off and embolize or result in total occlusion of VEIN not artery
A. Mural thrombi is most commonly found on the femoral
artery
B. The tail of thrombi often breaks off and embolized to

“Courage is found in unlikely places.”


― J.R.R. Tolkien 159
distal sites
C. Most common sites are cerebral, coronary and femoral
arteries
D. It typically creates a long cast of vessel lumen with a
propagational long tail

42. Regarding cutaneous wound healing which statement is


correct?

A. Vitamin C deficiency impairs collagen synthesis in


healing tissues
B. Early movement of wounds can stimulate the process of
angiogenesis and healing
C. Glucocorticoids promote collagen synthesis and fibrosis
of tissues
D. Complication from wound contraction are most
commonly seen on the face and neck

Schistosomiasis is a tropical disease caused by a parasite. The parasite is


43. Source of exposure to cercarial signs of human blood fluke: transmitted through contact with fresh water contaminated with the
parasite’s larvae

A. Contaminated water People become infected with schistosomes if they come into contact with
B. Contaminated soil the larval? forms of the parasite, for example, when washing or playing
C. Blood sucking arthropods in contaminated fresh water.
D. Insufficiently cooked meat of pork or beef

44. Which of the following is the least important risk factor for the
development of thrombus?

A. Acute myocardial infarction


B. Smoking
C. Antithrombin III deficiency
D. Disseminated malignancy

“Courage is found in unlikely places.”


― J.R.R. Tolkien 160
45. A 15 year old healthy girl with no major medical problems
notes blotchy areas of erythema that are pruritic over the skin of
her arms, legs and trunk within an hour every time she eats
seafood, followed by diarrhea. These problems abate within 3
hours, and then physical examination reveals no abnormal
findings. Which of the following immunologic abnormalities is she
most likely to have?

A. Complement activation
B. Hypergammaglobulinemia
C. Cell-mediated hypersensitivity
D. Localized anaphylaxis

46 Permanent association between two different organisms so


dependent on each other that their life part is impossible:

A. Symbiosis
B. Parasitism
C. Commensalism
D. Mutualism

Excess circulating thrombin results from the excess activation of the


47. Which of the following statement regarding disseminated coagulation cascade. The excess thrombin cleaves fibrinogen, which
intravascular coagulation is FALSE? ultimately leaves behind multiple fibrin clots in the circulation. These
excess clots trap platelets to become larger clots, which leads to
A. Bleeding diathesis is solely due to thrombocytopenia?? microvascular and macrovascular thrombosis. This lodging of clots in the
Not sure microcirculation, in the large vessels, and in the organs is what leads to
B. Rapid consumption of platelets, prothrombin and the ischemia, impaired organ perfusion, and end-organ damage that
fibrinogen occurs occurs with DIC.[citation needed]
C. Fibrinogen degradation products (FDPs) increases due
to rapid fibrinolysis Coagulation inhibitors are also consumed in this process. Decreased
D. Multiple thrombi develop in the microcirculation inhibitor levels will permit more clotting so that a positive feedback loop
develops in which increased clotting leads to more clotting. At the same
time, thrombocytopenia occurs and this has been attributed to the
entrapment and consumption of platelets. Clotting factors are consumed
in the development of multiple clots, which contributes to the bleeding
seen with DIC.[citation needed]
Simultaneously, excess circulating thrombin assists in the conversion of
plasminogen to plasmin, resulting in fibrinolysis. The breakdown of clots
results in an excess of FDPs, which have powerful anticoagulant
properties, contributing to hemorrhage. The excess plasmin also
activates the complement and kinin systems. Activation of these systems
leads to many of the clinical symptoms that patients experiencing DIC
exhibit, such as shock, hypotension, and increased vascular permeability.
The acute form of DIC is considered an extreme expression of the

“Courage is found in unlikely places.”


― J.R.R. Tolkien 161
intravascular coagulation process with a complete breakdown of the
normal homeostatic boundaries. DIC is associated with a poor prognosis
and a high mortality rate

48. Coarctation of the aorta is common in which syndrome:


Turner’s- Heart abnormalities are common in individuals with Turner.
A. Turner’s should be
B. Noonan’s monitored for problems with their Aorta and high blood pressure.
C. Down’s
D. Klinefelter’s Coarctation of the aorta- Aortic narrowing near insertion of ductus
arteriousus (juxtaductal). Associated with bicuspid aortic valve, other
heart defects, and Turner syndrome.

USMLE Step 1 2018, Cardio-Pathology, Page 294, Section III.

49. With regard to mechanisms of cellular adaptation, which of


the following statements is TRUE? Barrett’s esophagus- consists of change in the normally Squamous lining
of the lower esophagus to Columnar epithelium (metaplasia). Unless
A. TNF always acts as an inhibitor of hyperplasia there is severe esophagitis, this change can be recognized during an
B. Barrett’s metaplasia is squamous to columnar endoscopy.
epithelium in the distal esophagus
C. Hyperplasia refers to an increase in cell size WebMD
D. The myocardium adapts to increased workload by
hypertrophy and hyperplasia Metaplasia- Reprogramming of Stem cells, replacement of one cell type
by another that can adapt to a new stress. Usually due to an irritant, such
as gastric acid from GERD that will lead to ( Barretts esophagus).

USMLE Step 1 2018, Patho- Cellular Injury, Page 206, Section II.

50. Type of specimen used in parasitological examination of


Naegleria fowleri: CSF- Naegleriafowleri infection. can be diagnosed in the laboratory by
detecting
A. Urine N. fowleri organism in CSF thru Lumbar puncture, biopsy or tissue
B. Cerebrospinal fluid specimens.
C. Perianal swab
D. Liver aspirate www.cdc.gov/ diagnosis-hcp

CSF

USMLE Step 1 2018, Micro/para page 156, Section II

51. A 51 year old man has worked for 10 years in a factory


producing plastic pipe but not following safety standards. He has Vinyl Chloride- Exposure is associated with an increased risk of a rare
noted weight loss, nausea, and vomiting worsening over the past form of liver CA (Hepatic Angiosarcoma). As well as Brain and Lung CA,
5 months. On examination, he is afebrile. There is generalized Lymphoma and Leukemia.
muscle wasting. Laboratory studies show the serum alkaline
phosphatase is 405 U/L with AST 47 U/L. ALT 35 U/L, and total Hepatic angiosarcoma- malignant tumor of endothelial origin; associated
bilirubin 1.2 mg/dL. An abdominal CT scan reveals a 12 cm right with exposure to arsenic and Vinyl Chloride.
liver lobe mass. Liver biopsy reveals a neoplasm composed of
spindle cells forming irregular vascular channels. With USMLE Step 1 2018, GIT- Patho Page 386, Section III.
immunohistochemical staining the cells demonstrate vimentin
positivity and cytokeratin negativity. Exposure to which of the

“Courage is found in unlikely places.”


― J.R.R. Tolkien 162
following substances most likely led to development of this
neoplasm

A. • Cyclophosphamide
B. • Benzene
C. • Radon
D. • Vinyl chloride

52. Review of a series of surgical pathology reports indicates that If a grading system for a tumor type is not specified, the following system
a certain type of neoplasm is diagnosed as grade I on a scale of I is generally used (1):
to IV. Clinically, some of the patients with this neoplasm are found
to have stage I disease. Which of the following is the best GX: Grade cannot be assessed (undetermined grade)
interpretation of a neoplasm with these designation? G1: Well differentiated (low grade)
G2: Moderately differentiated (intermediate grade)
A. • Arising from epithelium G3: Poorly differentiated (high grade)
B. • Well-differentiated and localized G4: Undifferentiated (high grade)
C. • May spread via lymphatics and bloodstream
D. • Unlikely to be malignant National Cancer Institute/ USA.gov

53. Most common site of lodgment of arterial thromboemboli Arterial emboli commonly affect the brain and extremities.
is: Less frequently they can wreak havoc in mesenteric and renal
arteries.
A. • Kidney
B. • Brain https://www.ncbi.nlm.nih.gov/pmc/articles
C. • Lower limb
D. • Upper limb

PART 4 - BASIC QUESTIONS

Histamine is formed by decarboxylation of histidine catalyzed by


1. Formed by decarboxylation of histidine: histidine decarboxylase. It is stored preformed in cytoplasmic
A. Ipratropium granules of mast cells and basophils, and within platelets of some
B. Prostaglandin species. Synthesis occurs also in other cells within the gastric
C. Sodium mucosa, nervous tissue, and lymphatic
D. Histamine

2. Non-selective beta adrenergic blockers


A. Propanolol
B. Metoprolol
C. Atenolol
D. Bisoprolol

3. Action of acetylcholine Actions of Acetylcholine


A. Miosis *Muscarinic actions
B. Tachycardia -Heart
C. Vasoconstriction

“Courage is found in unlikely places.”


― J.R.R. Tolkien 163
D. High blood pressure -Depresses the SA node and thereby reduces the heart rate &
force of contraction
Intestinal-peristalsis is enhanced, sphincters are relaxed, resulting in Decreases rate of diastolic depolarization and increase
rapid forward propulsion of intestinal contents (M1 and M3 receptors) refractory period ( through M2 receptors)
Urinary bladder-detrusor contracts and trigonal sphincter relaxes Blood vessels
thus promotes voiding of urine. (M3 receptors) -Ach relaxes the vascular smooth muscles and dilates the blood
-Bronchial smooth muscle-contraction of bronchial muscle resulting in vessels of the skin and mucous membrane
bronchospasm (M3 receptors) -The BP falls due to a fall in total peripheral resistance and reflex
Secretory glands tachycardia (arteries have no parasympathetic innervations, but
-Enhances the secretion of all glands: saliva, lacrimal, nasopharyngeal, have M3 receptors which release EDRF)
trachea-bronchial, gastric ( M1 receptor) and intestinal secretions are Muscarinic Actions
increased. ( M3 receptors) -Smooth muscle Ach increases the tone of all other (non-vascular)
Eye smooth muscle tone
-Constriction of pupil (miosis) by contracting the circular muscles of
the iris
-Drainage of aqueous humor is facilitated and intraocular pressure
falls.
-Ciliary muscle contracts resulting in spasm of accommodation

4. Direct mode of action of adrenergic drugs Drugs that mimic the actions of epinephrine or norepinephrine have
A. Inhibit the enzyme cholinesterase traditionally been termed sympathomimetic drugs. The
B. Interact & activate adrenoreceptors sympathomimetics can be grouped by mode of action and by the
C. Displacement of stored catecholamines from the spectrum of receptors that they activate. Some of these drugs (eg,
adrenergic nerve endings norepinephrine and epinephrine) are direct agonists; that is, they
D. Inhibition of reuptake of catecholamines already directly interact with and activate adrenoceptors. Others are
released indirect agonists because their actions are dependent on their
Sympathomimetic drugs (also known as adrenergic drugs and ability to enhance the actions of endogenous catecholamines. These
adrenergic amines) are stimulant compounds which mimic the effects indirect agents may have either of two different mechanisms: (1)
of endogenous agonists of the sympathetic nervous system. they may displace stored catecholamines from the adrenergic nerve
ending (eg, the mechanism of action of tyramine), or they may
decrease the clearance of released norepinephrine either by (2a)
inhibiting reuptake of catecholamines already released (eg, the
mechanism of action of cocaine and tricyclic

5. Adrenoceptor antagonist EXCEPT Prazosin, a specific α1-adrenergic receptor antagonist, has been
A. Methyldopa used alone or with other medications to treat cardiovascular
B. Labetalol diseases such as hypertension and congestive heart failure.
C. Prazosin
An adrenergic antagonist is a drug that inhibits the function of
D. Propranolol
adrenergic receptors. There are five adrenergic receptors, which are
E.
divided into two groups. The first group of receptors are the beta (β)
Methyldopa is in the alpha-2 adrenergic receptor agonist family of
adrenergic receptors. There are β1, β2, and β3 receptors. The
medication. It works by stimulating the brain to decrease the activity second group contains the alpha (α) adrenoreceptors. There are
of the sympathetic nervous system only α1 and α2 receptors. Adrenergic receptors are located near the
heart, kidneys, lungs, and gastrointestinal tract.[1] There are also α-
adreno receptors that are located on vascular smooth muscle.[2]
Antagonists reduce or block the signals of agonists. They can be
drugs, which are added to the body for therapeutic reasons, or
endogenous ligands. The α-adrenergic antagonists have different
effects from the β-adrenergic antagonists.

Alpha blockers

● Phentolamine
● phenoxybenzamine

“Courage is found in unlikely places.”


― J.R.R. Tolkien 164
● Tamsulosin

Beta blockers

● Propranolol
● Nebivolol
● Atenolol
● Oxprenolol
● Metoprolol
● Timolol
● Pindolol
● Nadolol
● Pindolol
● Esmolol
● Acebutolol
● Sotalol
● Talinolol
● Betaxolol

Mixed action

● Labetalol
● Carvedilol

6. Reduce smooth muscle and secretory activity of the gut Anticholinergic agents such as dicyclomine (Bentyl, Axcan
A. Dicyclomine Scandipharm) and hyoscyamine (Levsin, Levbid, NuLev, all by
B. Atropine Schwarz Pharma) are commonly used in the United States to treat
C. Scopolamine pain associated with functional intestinal disorders. These agents
D. Ipratropium are smooth muscle relaxants that block the muscarinic effects of
acetylcholine on the gastrointestinal tract, thereby relaxing smooth
The alkaloid L-(-)-scopolamine [L-(-)-hyoscine] competitively inhibits muscle and potentially reducing spasm and abdominal pain,
muscarinic receptors for acetylcholine and acts as a nonselective slowing intestinal motility, and decreasing diarrhea
muscarinic antagonist, producing both peripheral antimuscarinic
properties and central sedative, antiemetic, and amnestic effects. Atropine reduces secretions in the mouth and respiratory passages,
relieves the constriction and spasm of the respiratory passages, and
Ipratropium is an acetylcholine antagonist via blockade of muscarinic may reduce the paralysis of respiration, which results from actions
cholinergic receptors. Blocking cholinergic receptors decreases the of the toxic agent on the central nervous system.
production of cyclic guanosine monophosphate (cGMP). This decrease
in the lung airways will lead to decreased contraction of the smooth
muscles.

7. Angiotensin-converting enzyme inhibitor ACE Inhibitors compete with angiotensin I for binding at the
angiotensin-converting enzyme, blocking the conversion of
A. Enalapril angiotensin I to angiotensin II.
B. Metoprolol
C. Losartan Effect: reduce angiotensin I level; reduce vasoconstriction and
D. Amlodipine aldosterone secretion; increase bradykinin

Drugs: “PRIL” Captopril, Enalapril, Fosinopril, Lisinopril

4. . Exert antimicrobial action against H. pylori: Clarithromycin is a macrolide antibiotic that inhibits protein
synthesis by binding to and slowing the actions of the bacterial
A. Magnesium hydroxide ribosome.
B. Sucralfate

“Courage is found in unlikely places.”


― J.R.R. Tolkien 165
C. Clarithromycin The most important antibiotics in H.Pylori treatment are
D. Cimetidine clarithromycin, metronidazole, and amoxicillin.

5. Effect of atropine Atropine is an anticholinergic drug.


MOA: inhibits the actions of acetylcholine on structures innervated
A. Tachycardia by postganglionic sites (smooth/cardiac muscle, SA/AV nodes);
B. Bronchoconstriction competitive antagonism at all M receptors
C. High doses produce sedation
D. Increased GIT motility and secretions Effects: Tachycardia, mydriasis, urinary retention, constipation,
dryness of the mouth

Phase III
6. Full scale evaluation of treatment comparing it with standard
Phase III trials compare a new drug to the standard-of-care drug.
treatment is done:
These trials assess the side effects of each drug and which drug
A. Phase 4 clinical trial
works better. Phase III trials enroll 100 or more patients.
B. Phase 1 clinical trial
C. Phase 2 clinical trial
D. Phase 3 clinical trial

7. Neutralize gastric acid:


Magnesium hydroxide- increases peristaltic activity causing
A. Sucralfate osmotic retention of fluids. It reduces stomach acid by reacting with
B. Magnesium hydroxide hydrochloric acid to form Mg Chloride.
C. Clarithromycin
D. Cimetidine

8. Beta 1 agonist:

A. Dopamine Dobutamine - exerts positive inotropic effect on the myocardium by


B. Propranolol stimulating B1 adrenergic receptors, thereby increasing myocardial
C. Clonidine contractility, stroke volume and cardiac output.
D. Dobutamine

9. Reduce gastric acid secretion:


Cimetidine - inhibits histamine at H2 receptors of the gastric parietal
A. Cimetidine cells resulting in decreased gastric acid secretion, gastric volume and
B. Sucralfate hydrogen ion concentration
C. Clarithromycin
D. Magnesium hydroxide

10. The actions of the body on the drug: Pharmacodynamic - actions of the drug on the body (p. 5)
A. Pharmacokinetic Pharmacokinetic - actions of the body on the drug (p. 5)
B. Pharmacogenetic Pharmacogenomics - study of genetic factors that underlie
C. Pharmacogenomic variation in drug response; modern term for pharmacogenetics
D. Pharmacodynamic (p. 74)

KATZUNG BG, Basic & Clinical Pharmacology 14e

11. Produce localized vasoconstriction on the small blood vessel Phenylpropanolamine - sympathomimetic; found in cold
of the nasal membrane: preparations
Terbutaline - selective B2 agonist; causes bronchodilation (p.
A. Phenylpropanolamine 351)
B. Terbutaline Dextromethorphan - opioid derivative; antitussive; for cough

“Courage is found in unlikely places.”


― J.R.R. Tolkien 166
C. Dextromethorphan suppression (p. 570)
D. Guaifenesin Guaifenesin - expectorant; increases the effective hydration of
the respiratory gland thereby increasing the volume and
reducing the viscosity of tenacious bronchial secretions, thus
facilitating its removal by natural clearance processes.

12. Cholinesterase regenerator: Cholinesterase regenerator compounds- composed of substituted


oximes. These oxime agents include pralidoxime (PAM),
A. Atropine diacetylmonoxime, obidoxime, and others.
B. Pralidoxime
C. Ipratropium
D. .Scopolamine

13. Adverse effect of propranolol The principal toxicities of propranolol result from blockade of
cardiac, vascular, or bronchial beta receptors.
A. Heart block
B. Heart block, hypertension and bronchodilation Propranolol produces significant reduction in blood pressure
C. Hypertension without prominent postural hypotension.
D. Heart block and hypertension
E. Bronchodilatation

14. Antitussive: Opioid derivatives most commonly used as antitussives are


dextromethorphan, codeine, levopropoxyphene, and noscapine;
A. Salbutamol they should be used with caution in patients taking monoamine
B. Dextromethorphan oxidase inhibitors.
C. Guaifenesin
D. Carbocysteine

15. Produce localized vasoconstriction on the small blood vessel


of the nasal membrane:
A. Phenylpropanolamine
B. Terbutaline
C. Dextromethorphan
D. Guaifenesin
Phenylpropanolamine is a sympathomimetic agent that acts as a
nonselective adrenergic receptor agonist and norepinephrine
reuptake inhibitor. It has been used as a decongestant and appetite
suppressant

16. Cholinesterase regenerator: Pralidoxime (2-PAM) Mechanism of Action: regenerates


A. Atropine cholinesterase activity that has become inhibited by an
B. Pralidoxime organophosphate compound (but is most effective when given
C. Ipratropium before chemical “aging” between the organophosphate &
D. Scopolamine cholinesterase enzyme has occurred)

17. Adverse effect of propranolol Principal toxicities of propranolol result from blockade of cardiac,
A. Heart block vascular, or bronchial B receptors .
B. Heart block, hypertension and bronchodilation In mild to moderate hypertension, propranolol produces a
C. Hypertension significant reduction in blood pressure without prominent postural
hypotension.

“Courage is found in unlikely places.”


― J.R.R. Tolkien 167
D. Heart block and hypertension
E. Bronchodilation

18. Antitussive: Albuterol (also known as salbutamol) is used to treat wheezing and
A. Salbutamol shortness of breath caused by breathing problems such as asthma.
B. Dextromethorphan It is a quick-relief medication. Albuterol belongs to a class of drugs
C. Guaifenesin known as bronchodilators.
D. Carbocysteine
Dextromethorphan is in a class of medications called antitussives.
Guaifenesin is an expectorant that helps thin and loosen mucus in the
lungs, making it easier to cough up the mucus

Carbocysteine -A mucolytic

19.What is the single best test to diagnose a kidney or ureteral stone? Urolithiasis, or urinary calculus disease, may affect up to 10% of the
population over the course of a lifetime.47 Calculi are crystalline
A. CT scan of the abdomen and pelvis with and without contrast aggregates of one or more components, most commonly calcium
B. KUB oxalate. They also may contain calcium phosphate, magnesium
C. Rectal ultrasound ammonium phosphate (struvite), uric acid, or cystine. Calcium- and
D. Non contrast computerized tomography (CT) scan of struvite-containing stones often are visible on plain radiographs, but
abdomen and pelvis CT scans will demonstrate all calculi except those composed of
crystalline-excreted indinavir, an antiretroviral medication.48 For
this reason, noncontrast CT scans have become the study of choice
to evaluate for urolithiasis (schwart’s pg 1666)

20. Comprises small scale trials on patients used to determine dose Phase 0
level and establish that the treatment offers some benefits.
A. Phase 2 Clinical Trial Phase 0 trials are the first clinical trials done among people. They
B. Phase 1 Clinical Trial aim to learn how a drug is processed in the body and how it affects
C. Phase 4 Clinical Trial the body. In these trials, a very small dose of a drug is given to about
D. Phase 3 Clinical Trial 10 to 15 people.

Phase I
Phase I trials aim to find the best dose of a new drug with the
fewest side effects. The drug will be tested in a small group of 15
to 30 patients.

Phase II
Phase II trials further assess safety as well as if a drug works. The
drug is often tested among patients with a specific type of cancer.
Phase II trials are done in larger groups of patients compared to
Phase I trials.

Phase III
Phase III trials compare a new drug to the standard-of-care drug.
These trials assess the side effects of each drug and which drug
works better. Phase III trials enroll 100 or more patients.

21. Loop Diuretics: Examples of loop diuretics include:


A. Furosemide ● Bumetanide (Bumex)
B. Spironolactone
C. Thiazides
● Ethacrynic acid (Edecrin)

“Courage is found in unlikely places.”


― J.R.R. Tolkien 168
D.Clonidine ● Furosemide (Lasix)
● Torsemide (Demadex)

22. Cholinergic drugs are also called:

A. Sympatholytic drugs
B. Sympathomimetic drugs
C. Parasympathomimetic drugs
D. Parasympatholytic drugs

23. Alpha 1 adrenoceptor action, EXCEPT:

A. Contraction of most vascular smooth muscle


B. Mydriasis
C. Increase force of contraction of the heart
D. Glycogenolysis

24. Alpha 2 antagonist: Phentolamine is a potent competitive antagonist at both α1 and α2 receptors.
Phentolamine reduces peripheral resistance through blockade of α1 receptors
A. Oxymetazoline and possibly α2 receptors on vascular smooth muscle. Its cardiac stimulation is
B. Phentolamine due to antagonism of presynaptic α2 receptors (leading to enhanced release of
C. Isoproterenol norepinephrine from sympathetic nerves) and sympathetic activation from
D. Clonidine baroreflex mechanisms.
Oxymetazoline- alpha 1& alpha 2 agonist
Isoproterenol-B receptor agonist
Clonidine- alpha 2 adrenergic receptor agonist

“Courage is found in unlikely places.”


― J.R.R. Tolkien 169
25. Indication for Physostigmine

A. Glaucoma
B. Paralytic ileus
C. Postoperative retention of urine
D. Coronary insufficiency

26. Contraindication/s for the use of adrenaline:

A. Hyperthyroidism
B. Hypertension
C. Hyperthyroidism, hypertension, and coronary
artery disease
D. Coronary artery disease
E. Hypertension and coronary artery disease

27.The new drug is studied in patients with the target Phase 0


disease to determine its efficacy and the doses to be used
in any follow-on trials: Phase 0 trials are the first clinical trials done among people. They aim to learn how
a drug is processed in the body and how it affects the body. In these trials, a very
A. Phase 3 Clinical Trials small dose of a drug is given to about 10 to 15 people.
B. Phase 4 Clinical Trials
C. Phase 2 Clinical Trials Phase I
D. Phase 1 Clinical Trials Phase I trials aim to find the best dose of a new drug with the fewest side effects.
The drug will be tested in a small group of 15 to 30 patients.
Phase IV
Phase II
Phase IV trials test new drugs approved by the FDA. The
drug is tested in several hundreds or thousands of Phase II trials further assess safety as well as if a drug works. The drug is often
patients. This allows for better research on short-lived tested among patients with a specific type of cancer. Phase II trials are done in
and long-lasting side effects and safety. larger groups of patients compared to Phase I trials.

Phase III
Phase III trials compare a new drug to the standard-of-care drug. These trials
assess the side effects of each drug and which drug works better. Phase III trials
enroll 100 or more patients.

28. Enhance mucosal defenses: sucralfate is protective against acid injury in rabbit esophagus by enhancing
mucosal defenses through binding of pepsin and bile salts, neutralization of
A. Magnesium Hydroxide hydrogen ions by its content of aluminum hydroxide, and reduction in the
B. Cimetidine permeability of esophageal mucosa
C. Sucralfate

“Courage is found in unlikely places.”


― J.R.R. Tolkien 170
D. Clarithromycin

29. Drugs that mimic the action of acetylcholine Drugs that activate muscarinic receptors in the peripheral nervous system are
called parasympathomimetic drugs because they mimic the effects of
A. Parasympathomimetic drugs acetylcholine on the parasympathetic nervous system.
B. Sympatholytic drugs
C. Parasympatholytic drugs
D. Sympathomimetic drugs

30. Alpha 1 agonist Dobutamine - B1 agonist


Propanolol - non selective beta adrenergic blocker
A. Dobutamine Prazosin - A1 antagonist
B. Phenylephrine
C. Propanolol
D. Prazosin

31.The actions of the drug on the body pharmacogenetic- it is the science of understanding how genetic variability
influences drug treatment outcomes
A. pharmacogenetic
B. pharmacodynamics pharmacokinetics- actions of body on the drug
C. pharmacokinetics
D. pharmacogenomics pharmacogenomics-study on the role of genome in drug response

32. Non-pharmacological approaches to therapy of Non-Pharmacologic Management : Lifestyle Management for hypertension
hypertension:
A. Low sodium chloride diet, weight
Weight reduction Attain & maintain BMI < 25 kg/m2
reduction, and cessation of smoking
B. Low sodium chloride diet and weight Salt and potassium in diet -For adults, reducation of Na+ intake
reduction by 1 g/ day lowers SBP by 3-4 mmHg
C. Cessation of smoking (aim to consume no more than 2.4
D. Low sodium chloride diet g/day of Na+)
E. Weight reduction -Increased K+ intake my lower BP

Adapt Dietary Approaches to Stop -Diet rich in fruits, vegetables, low fat
Hypertension (DASH) type dietary dairy products, whole grains, poultry ,
plan fish , nuts
Diet low in sweet , red meat &
saturated / total fat

Moderation of Alcohol consumption For those who drink alcohol


,consume:
</=2 standard drinks/ day in men
</=1standard drink/day in women

Physical activity Regular aerobic activity ( 3-4 session a


week lasting 40 mins per session)
Aerobic physical activity reduces SBP
by up to 5 mmHg

33. The relationship of the Individual’s genetic make up medical pharmacology-it is the science that study how medicines drugs work in
to his/her response to specific drugs: health and disease and how they are processed by the bodies

“Courage is found in unlikely places.”


― J.R.R. Tolkien 171
A.medical pharmacology pharmacology- it is a branch of medicine and pharmaceutical sciences which is
B. pharmacology concerned with the study of drug medication actions
C. toxicology
D. pharmacogenomics toxicology- it is the study of adverse effects that occur in living organisms due to
chemicals

34. Side effect/s of Methyldopa:

A. Sedation
B. Sedation, vertigo, and nausea
C. Nausea
D. Sedation and vertigo
E. Vertigo

35. Lower blood pressure by depleting the body sodium


and reducing blood volume:

A. Diuretic
B. Calcium channel blocker
C. Beta blocker
D. ACE inhibitor

36. A boy is being worked up for severe acute


rhinosinusitis. Culture from material obtained from
maxillary sinus grew aerobic, oxidase-positive and gram-
negative diplococci.

How should this infection be treated?


A. Cephalexin
B. Vancomycin
C. Ampicillin
D. Amoxicillin-clavulanate

37.2Which of the following causes a fatal encephalitis for


which a vaccine is available? Encephalitis, or inflammation of the brain, is a treatable but potentially-serious
medical condition that can result from immune system disorders. This includes
a. Rabies
B. Mumps disorders triggered by the MMR vaccine. According to the CDC:
C. Rhinovirus
D. Cytomegalovirus “Measles inclusion body encephalitis, or severe brain swelling caused by the
measles virus, is a complication of getting infected with the wild-type measles
virus. While rare, this disorder almost always happens in patients with weakened
immune systems. The illness usually develops within 1 year after initial measles
infection and has a high death rate.”

38. All of the following syndromes are associated with Genetic syndromes associated with childhood obesity include the following:
Obesity EXCEPT:
● Prader-Willi syndrome

“Courage is found in unlikely places.”


― J.R.R. Tolkien 172
A. Hypothyroidism ● Pseudohypoparathyroidism
B. Cushings Syndrome ● Laurence-Moon-Biedl (Bardet-Biedl) syndrome
C. Prader-will Syndrome ● Cohen syndrome
D. Acromegaly ● Down syndrome
E. Insulinoma ● Turner syndrome
Hormonal disorders associated with childhood obesity include the following:

● Growth hormone deficiency


● Growth hormone resistance
● Hypothyroidism
● Leptin deficiency or resistance to leptin action
● Glucocorticoid excess (Cushing syndrome)
● Precocious puberty
● Polycystic ovary syndrome (PCOS)
● Prolactin-secreting tumors

39.Which is the most common side effect of Side effects caused by chemotherapy:
chemotherapy?

A. Nausea with or without vomiting


B. Febrile neutropenia ● Fatigue
C. Diarrhea
D. Alopecia
● Hair loss
E. Mucositis ● Easy bruising and bleeding
● Infection
● Anemia (low red blood cell counts)
● Nausea and vomiting
● Appetite changes
● Constipation
● Diarrhea
● Mouth, tongue, and throat problems such as sores and pain with
swallowing
● Peripheral neuropathy or other nerve problems, such as numbness,
tingling, and pain
● Skin and nail changes such as dry skin and color change
● Urine and bladder changes and kidney problems
● Weight changes
● Chemo brain, which can affect concentration and focus
● Mood changes
● Changes in libido and sexual function

“Courage is found in unlikely places.”


― J.R.R. Tolkien 173
● Fertility problems

40. All of the ff are manifestation of Ascaris lumbricoides Ascaris Lumbricoides Clinical Presentation
infection EXCEPT: ● Fever
● Nonproductive cough
A. Non-productive cough and pleurisy with ● Dyspnea
eosinophilia ● Wheezing
B. Fever, headache, photophobia, nuchal rigidity, ● Passage of worms (from mouth, nares, anus)
and eosinophilia ● Diffuse or epigastric abdominal pain
C. Right upper quadrant pain and fever ● Nausea, vomiting
D. Asymptomatic carriage ● Pharyngeal globus, "tingling throat"
E. Small bowel obstruction ● Frequent throat clearing, dry cough
● Complications - Mechanical small bowel obstruction, volvulus,
intussusception, biliary obstruction, appendicitis, pancreatitis

41. Which one of the following represents the mechanism Azole antifungals consist of two primary classes: imidazoles and triazoles. Both
of action of anti-fungal azoles? classes are fungistatic agents and share similar mechanisms of action. The azoles
interfere with the synthesis and permeability of fungal cell membranes by
A) Inhibition of the activity of the enzyme, chitin inhibiting cytochrome P450-dependent 14-alpha-sterol demethylase
synthase
B) Inhibition of GTP hydrolysis carried out by
elongation factor-2 (EF-2)
C) Inhibition of the activity of the enzyme, 14
alpha-demethylase
D) Inhibition of the activity of the enzyme, B-

“Courage is found in unlikely places.”


― J.R.R. Tolkien 174
glucan synthase

42. Which of the following is a leading cause of Pneumonia in infants aged three weeks to three months is most often bacterial;
pneumonia primarily in infants? Streptococcus pneumoniae is the most common pathogen. In infants older than
four months and in preschool-aged children, viruses are the most frequent cause
A) Rhinovirus of CAP; respiratory syncytial virus (RSV) is the most common.
B) Respiratory Syncytial Virus
C) Mumps
D) Cytomegalovirus

43. Which of the following causes a fatal encephalitis for Rabies is fatal encephalitis of viral origin in humans and some other mammals.
which a vaccine is available?
Mumps has also a vaccine (MMRV).According to WHO the most common
A. Rhinovirus complication of mumps in children is meningitis, sometimes associated with
B. Rabies encephalitis, and in young adults orchitis.
C. Cytomegalovirus
D. Mumps(another question answered this one) Yung isa bandang taas sagot niya Mumps
https://www.ncbi.nlm.nih.gov/pmc/articles/PMC6307015/

44. How is Pneumonic plague infection transmitted? Pneumonic plague occurs when Y. pestis infects the lungs.

A. Ingestion of contaminated water This type of plague can spread from person to person through the air.
B. Inhalation of aerosolized bacteria Transmission can take place if someone breathes in aerosolized bacteria, which
C. Direct skin contact with bubos could happen in a bioterrorist attack.
D. Bite of infected rat

45. A child develops impetigo on the right thigh. He was Penicillins and cephalosporins are the major antibiotics that inhibit bacterial cell
given an antibiotic that will inhibit cell wall synthesis. wall synthesis.
Which is the most likely antibiotic this child would have
received?

A. Erythromycin
B. Cephalexin
C. Ciprofloxacin
D. Ampicillin

46. Reduce gastric acid secretion: > Types of H2 Blockers

a. Sucralfate
b. Cimetidine
● Famotidine
C. Clarithromycin ● Cimetidine
D. Magnesium sulfate ● Ranitidine
● Nizatidine Capsules
>Examples of H2 antagonists commonly used to suppress gastric acid secretion
include cimetidine, ranitidine, famotidine and nizatidine. These drugs,
particularly cimetidine, are among the most widely prescribed drugs in man.
>Sulfacrate: This drug is a sucrose-aluminum complex that dissociates in stomach
acid and forms a physical barrier over an inflamed area, protecting it from acid,
pepsin, and bile salts. It also inhibits pepsin-substrate interaction, stimulates
mucosal prostaglandin production, and binds bile salts. It has no effect on acid
output or gastrin secretion. Sucralfate seems to have trophic effects on the

“Courage is found in unlikely places.”


― J.R.R. Tolkien 175
ulcerated mucosa, possibly by binding growth factors and concentrating them at
an ulcer site.

47. Traditional vaccination for the common cold is Rhinovirus has more than 100 serotypes; it is unlikely that a unifying vaccine will
virtually impossible because there are multiple serotypes be developed. However, the use of antivirals as chemoprophylaxis may have
of which one of the following viruses? practical value. Topical application of interferon in the nose has been shown to be
effective in reducing the incidence of colds in people who are exposed to others
a. Mumps with a fresh cold .This strategy reduced the overall risk of cold by 40% and almost
B. Cytomegalovirus eliminated proven rhinovirus colds in contacts
C. Rhinovirus
d. Respiratory syncytial virus

48. The action of the drug on the body: PharmacoDynamics: action of Drug to the body
PharmacoKinetics: action of Katawan/Body to drugs
a. Pharmacokinetic
b. Pharmacodynamic
C. Pharmacogenetic
D. Pharmacogenomic

49. Classified as a calicivirus this hepatitis and is acquired


via the fecal-oral route. Common reservoir are pigs and
humans

A. Hepatitis E
B. Hepatitis D
C. Hepatitis C
D. Hepatitis B

50. Orchitis, which may cause sterility, is a possible This inflammatory condition, medically called orchitis, affects boys (who have
manifestation of which of the following? been through puberty) or adults. Orchitis generally affects just one testicle but
can affect both testicles in about 1 in 6 men. This is the reason why mumps causes
A. Mumps male infertility
B. Rhinovirus
C. Rabies
D. Cytomegalovirus

51. Non-selective beta adrenergic blockers: Nonselective beta-blockers include propranolol , nadolol , pindolol , labetalol ,
penbutolol , sotalol , carvedilol and timolol
A. Atenolol
B. Bisoprolol
C. Metoprolol
D. Propranolol

“Courage is found in unlikely places.”


― J.R.R. Tolkien 176
52. A child develops impetigo on the right thigh. He was Penicillins and cephalosporins are the major antibiotics that inhibit bacterial cell
given an antibiotic that will inhibit cell wall synthesis. wall synthesis.
Which is the most likely antibiotics this child would have
received? Oral antibiotic therapy can be used for impetigo with large bullae or when topical
therapy is impractical. Amoxicillin/clavulanate, dicloxacillin, cephalexin,
A. Cephalexin clindamycin, doxycycline, minocycline, trimethoprim/sulfamethoxazole, and
B. Erythromycin macrolides are options, but penicillin is not.
C. Ciprofloxacin
D. Ampicillin

“Courage is found in unlikely places.”


― J.R.R. Tolkien 177

You might also like